JEE MAIN (AIEEE) 2002-2013 Question Paper

You might also like

Download as pdf or txt
Download as pdf or txt
You are on page 1of 400

https://www.gofacademy.

in AIEEE 2002 Old Question Paper by GOF Academy


https://www.gofacademy.in AIEEE 2002 Old Question Paper by GOF Academy
https://www.gofacademy.in AIEEE 2002 Old Question Paper by GOF Academy
https://www.gofacademy.in AIEEE 2002 Old Question Paper by GOF Academy
https://www.gofacademy.in AIEEE 2002 Old Question Paper by GOF Academy
https://www.gofacademy.in AIEEE 2002 Old Question Paper by GOF Academy
https://www.gofacademy.in AIEEE 2002 Old Question Paper by GOF Academy
https://www.gofacademy.in AIEEE 2002 Old Question Paper by GOF Academy
https://www.gofacademy.in AIEEE 2002 Old Question Paper by GOF Academy
https://www.gofacademy.in AIEEE 2002 Old Question Paper by GOF Academy
https://www.gofacademy.in AIEEE 2002 Old Question Paper by GOF Academy
https://www.gofacademy.in AIEEE 2002 Old Question Paper by GOF Academy
https://www.gofacademy.in AIEEE 2002 Old Question Paper by GOF Academy
https://www.gofacademy.in AIEEE 2002 Old Question Paper by GOF Academy
https://www.gofacademy.in AIEEE 2002 Old Question Paper by GOF Academy
https://www.gofacademy.in AIEEE 2002 Old Question Paper by GOF Academy
https://www.gofacademy.in AIEEE 2002 Old Question Paper by GOF Academy
https://www.gofacademy.in AIEEE 2002 Old Question Paper by GOF Academy
https://www.gofacademy.in AIEEE 2002 Old Question Paper by GOF Academy
https://www.gofacademy.in AIEEE 2002 Old Question Paper by GOF Academy
https://www.gofacademy.in AIEEE 2002 Old Question Paper by GOF Academy
https://www.gofacademy.in AIEEE 2002 Old Question Paper by GOF Academy
https://www.gofacademy.in AIEEE 2002 Old Question Paper by GOF Academy
https://www.gofacademy.in AIEEE 2002 Old Question Paper by GOF Academy
https://www.gofacademy.in AIEEE 2002 Old Question Paper by GOF Academy
https://www.gofacademy.in AIEEE 2002 Old Question Paper by GOF Academy
https://www.gofacademy.in AIEEE 2002 Old Question Paper by GOF Academy
https://www.gofacademy.in AIEEE 2002 Old Question Paper by GOF Academy
https://www.gofacademy.in AIEEE 2002 Old Question Paper by GOF Academy
https://www.gofacademy.in AIEEE 2002 Old Question Paper by GOF Academy
https://www.gofacademy.in AIEEE 2002 Old Question Paper by GOF Academy
https://www.gofacademy.in AIEEE 2002 Old Question Paper by GOF Academy
https://www.gofacademy.in AIEEE 2002 Old Question Paper by GOF Academy
https://www.gofacademy.in AIEEE 2002 Old Question Paper by GOF Academy
https://www.gofacademy.in AIEEE 2002 Old Question Paper by GOF Academy
https://www.gofacademy.in AIEEE 2002 Old Question Paper by GOF Academy
https://www.gofacademy.in AIEEE 2002 Old Question Paper by GOF Academy
https://www.gofacademy.in AIEEE 2002 Old Question Paper by GOF Academy
https://www.gofacademy.in AIEEE Old Question Paper by GOF Academy
https://www.gofacademy.in AIEEE Old Question Paper by GOF Academy
https://www.gofacademy.in AIEEE Old Question Paper by GOF Academy
https://www.gofacademy.in AIEEE Old Question Paper by GOF Academy
https://www.gofacademy.in AIEEE Old Question Paper by GOF Academy
https://www.gofacademy.in AIEEE Old Question Paper by GOF Academy
https://www.gofacademy.in AIEEE Old Question Paper by GOF Academy
https://www.gofacademy.in AIEEE Old Question Paper by GOF Academy
https://www.gofacademy.in AIEEE Old Question Paper by GOF Academy
https://www.gofacademy.in AIEEE Old Question Paper by GOF Academy
https://www.gofacademy.in AIEEE Old Question Paper by GOF Academy
https://www.gofacademy.in AIEEE Old Question Paper by GOF Academy
https://www.gofacademy.in AIEEE Old Question Paper by GOF Academy
https://www.gofacademy.in AIEEE Old Question Paper by GOF Academy
https://www.gofacademy.in AIEEE Old Question Paper by GOF Academy
https://www.gofacademy.in AIEEE Old Question Paper by GOF Academy
https://www.gofacademy.in AIEEE Old Question Paper by GOF Academy
https://www.gofacademy.in AIEEE Old Question Paper by GOF Academy
https://www.gofacademy.in AIEEE Old Question Paper by GOF Academy
https://www.gofacademy.in AIEEE Old Question Paper by GOF Academy
https://www.gofacademy.in AIEEE Old Question Paper by GOF Academy
https://www.gofacademy.in AIEEE Old Question Paper by GOF Academy
https://www.gofacademy.in AIEEE Old Question Paper by GOF Academy
https://www.gofacademy.in AIEEE Old Question Paper by GOF Academy
https://www.gofacademy.in AIEEE Old Question Paper by GOF Academy
https://www.gofacademy.in AIEEE Old Question Paper by GOF Academy
https://www.gofacademy.in AIEEE Old Question Paper by GOF Academy
https://www.gofacademy.in AIEEE Old Question Paper by GOF Academy
https://www.gofacademy.in AIEEE Old Question Paper by GOF Academy
https://www.gofacademy.in AIEEE Old Question Paper by GOF Academy
https://www.gofacademy.in AIEEE Old Question Paper by GOF Academy
https://www.gofacademy.in AIEEE Old Question Paper by GOF Academy
https://www.gofacademy.in AIEEE Old Question Paper by GOF Academy
https://www.gofacademy.in AIEEE Old Question Paper by GOF Academy
https://www.gofacademy.in AIEEE Old Question Paper by GOF Academy
https://www.gofacademy.in AIEEE Old Question Paper by GOF Academy
https://www.gofacademy.in AIEEE Old Question Paper by GOF Academy
https://www.gofacademy.in AIEEE Old Question Paper by GOF Academy
https://www.gofacademy.in AIEEE Old Question Paper by GOF Academy
https://www.gofacademy.in AIEEE Old Question Paper by GOF Academy
https://www.gofacademy.in AIEEE Old Question Paper by GOF Academy
https://www.gofacademy.in AIEEE Old Question Paper by GOF Academy
https://www.gofacademy.in AIEEE Old Question Paper by GOF Academy
http://notesforfree.com/

PAPER -2004
1. Which one of the following represents the correct dimensions of the coefficient of viscosity?
(A) ML−1T−2 (B) MLT−1
−1 −1
(C) ML T (D) ML−2T−2

1. C.
Dimensions of η (coefficient of viscosity)
MLT −2
= = ML−1T−1
M L ⋅ M0LT −1
0 0

2. A particle moves in a straight line with retardation proportional to its displacement. Its loss of
kinetic energy for any displacement x is proportional to
(A) x2 (B) ex
(C) x (D) logex

2. A.
mk 2
K f − Ki = x
2

/
m
K f − ki ∝ x2 .

co
3. A ball is released from the top of a tower of height h metres. It takes T seconds to reach the
ground. What is the position of the ball in T/3 seconds?
(A) h/9 metres from the ground
(C) 8h/9 metres from the ground
e e.
(B) 7h/9 metres from the ground
(D) 17h/18 metres from the ground.
rfr

3. C.
fo

r r r r
es

4. If A × B = B × A, then the angle between A and B is


(A) π (B) π/3
t

(C) π/2 (D) π/4


no

4. A.
://
tp

5. A projectile can have the same range R for two angles of projection. If T1 and T2 be the time
of flights in the two cases, then the product of the two time of flights is directly proportional to
ht

(A) 1/R2 (B) 1/R


(C) R (D) R2

5. C.
Range is same for complimentary angles.
2u sin θ 2u sin (90 − θ)
T1 = and T2 =
g g
u2 sin 2θ
and R =
g
2u sin θ 2u cos θ 2R
∴ T1T2 = × = .
g g g

6. Which of the following statements is false for a particle moving in a circle with a constant
angular speed?
(A) The velocity vector is tangent to the circle.
(B) The acceleration vector is tangent to the circle.
(C) The acceleration vector points to the centre of the circle.
(D) The velocity and acceleration vectors are perpendicular to each other.

http://notesforfree.com/
http://notesforfree.com/
AIEEE PAPER-04-PH-2

6. B.
The acceleration vector is along the radius of circle.

7. An automobile travelling with speed of 60 km/h, can brake to stop within a distance of 20 cm.
If the car is going twice as fast, i.e 120 km/h, the stopping distance will be
(A) 20 m (B) 40 m
(C) 60 m (D) 80 m

7. D.
If the initial speed is doubled, the stopping distance becomes four times, i.e. 80 m.

8. A machine gun fires a bullet of mass 40 g with a velocity 1200 ms−1. The man holding it can
exert a maximum force of 144 N on the gun. How many bullets can he fire per second at the
most?
(A) one (B) four
(C) two (D) three

8. D.
Change in momentum for each bullet fired is
40

/
= × 1200 = 48 N

m
1000
If a bullet fired exerts a force of 48 N on man’s hand so ρ man can exert maximum force of 144 N,

co
number of bullets that can be fired = 144/48 = 3 bullets.

9. e.
Two masses m1 = 5 kg and m2 = 4.8 kg tied to a string are hanging
e
over a light frictionless pulley. What is the acceleration of the masses
rfr
when lift free to move?
(g = 9.8 m/s2)
fo

(A) 0.2 m/s2 (B) 9.8 m/s2


es

2
(C) 5 m/s (D) 4.8 m/s2 m1
t

9. A. m2
no

 m − m2 
a= 1  g = 0.2 m / s
2
://

 m1 + m2 
tp

10. A uniform chain of length 2 m is kept on a table such that a length of 60 cm hangs freely
ht

from the edge of the table. The total mass of the chain is 4 kg. What is the work done in
pulling the entire chain on the table?
(A) 7.2 J (B) 3.6 J
(C) 120 J (D) 1200 J

10. B.
Work done = mgh = 1.2 × 0.3 × 10 = 3.6 J.

11. A block rests on a rough inclined plane making an angle of 30° with the horizontal. The
coefficient of static friction between the block and the plane is 0.8. If the frictional force on
the block is 10 N, the mass of the block (in kg) is (take g = 10 m/s2)
(A) 2.0 (B) 4.0
(C) 1.6 (D) 2.5

11. A.
m = 2 kg

http://notesforfree.com/
http://notesforfree.com/
AIEEE PAPER-04-PH-3

r
12. A force F = (5iˆ + 3ˆj + 2k)N
ˆ is applied over a particle which displaces it from its origin to the
r ˆ ˆ
point r = (2i − j) m. The work done on the particle in joules is
(A) −7 (B) +7
(C) +10 (D) +13
12. B.
r r
Work done, W = F ⋅ s
r r r
Here s = rf − ri = (2iˆ − ˆj)
W = (5iˆ + 3ˆj + 2k)(2i
ˆ ˆ − ˆj) = 10 − 3 = 7 J.

13. A body of mass m, accelerates uniformly from rest to v1 in time t1. The instantaneous power
delivered to the body as a function of time t is
mv1t mv12 t
(A) (B)
t1 t12
mv1t 2 mv12 t
(C) (D)
t1 t1

13. B.

/
m
r r  v   v  mv 2 t
Power P = F ⋅ v = mav = m  1   1 t  = 2 1

co
 t 1   t1  t1

14.
e.
A particle is acted upon by a force of constant magnitude which is always perpendicular to
the velocity of the particle, the motion of the particle takes place in a plane. It follows that
e
(A) its velocity is constant (B) its acceleration is constant
rfr
(C) its kinetic energy is constant (D) it moves in a straight line.
fo

14. C.
es

When a force of constant magnitude which is always perpendicular to the velocity of the
particle acts on a particle, the work done and hence change in kinetic energy is zero.
t
no

15. A solid sphere is rotating in free space. If the radius of the sphere is increased keeping mass
same which one of the following will not be affected?
://

(A) moment of inertia (B) angular momentum


(C) angular velocity (D) rotational kinetic energy.
tp

15. B.
ht

Let it be assume that in “free space” not only the acceleration due to gravity it acting but also
there are no external torque acting but also there are no external torque acting on the
sphere. If due to internal changes in the system, the radius has increased, then the law of
the conservation of angular momentum holds good.
16. A ball is thrown from a point with a speed ν0 at an angle of projection θ. From the same point
and at the same instant person starts running with a constant speed ν0/2 to catch the ball.
Will the person be able to catch the ball? If yes, what should be the angle of projection?
(A) yes, 60° (B) yes, 30°
(C) no (D) yes, 45°
16. A.
For the person to be able to catch the ball, the horizontal component of the velocity of the
ball should be same as the speed of the person.
v
v 0 cos θ = 0
2
⇒ θ = 60°.

http://notesforfree.com/
http://notesforfree.com/
AIEEE PAPER-04-PH-4

17. One solid sphere A and another hollow sphere B are of same mass and same outer radii.
Their moment of inertia about their diameters are respectively IA and IB such that
(A) IA = IB (B) IA > IB
(C) IA < IB (D) IA/IB = dA/dB
Where dA and dB are their densities.

17. C.
2
Moment of inertia of a uniform density solid sphere, A = MR 2
5
2
And of hollow sphere B = MR 2
3
Since M and R are same, IA < IB.

18. A satellite of mass m revolves around the earth of radius R at a height x from its surface. If g
is the acceleration due to gravity on the surface of the earth, the orbital speed of the satellite
is
gR
(A) gx (B)
R−x
1/ 2
gR2  gR2 

/
(C) (D)  

m
R+x R + x

co
18. D.

GMem
=
Mv 02
and
GMe
=g
e e.
For the satellite, the gravitational force provides the necessary centripetal force i.e.
rfr
(R + X)2
(R + X) R2
1/ 2
 gR2 
fo

∴ v0 =  
R + X
t es

19. The time period of an earth satellite in circular orbit is independent of


no

(A) the mass of the satellite


(B) radius of its orbit
://

(C) both the mass and radius of the orbit


tp

(D) neither the mass of the satellite nor the radius of its orbit.
ht

19. A.
The time period of satellite is given by
(R + h)3
T = 2π
GM
where, R + h = radius of orbit satellite, M = mass of earth.

20. If g is the acceleration due to gravity on the earth’s surface, the gain in the potential energy
of object of mass m raised from the surface of the earth to a height equal to the radius R of
the earth is
1
(A) 2 mgR (B) mgR
2
1
(C) mgR (D) mgR
4

20. B.

http://notesforfree.com/
http://notesforfree.com/
AIEEE PAPER-04-PH-5

21. Suppose the gravitational force varies inversely as the nth power of distance. Then the time
period planet in circular orbit of radius R around the sun will be proportional to
 n +1   n −1
   
2  2 
(A) R (B) R
 n−2 
 
(C) Rn (D) R 2 

21. A.
T ∝ R(n +1) / 2

22. A wire fixed at the upper end stretches by length by applying a force F. The work done in
stretching is
(A) F/2l (B) Fl
(C) 2Fl (D) Fl/2

22. D.
1 2 1 2
Work done = kx = kl where l is the total extensions.
2 2
1 1
= (kl )l = Fl

/
2 2

m
co
23. Spherical balls of radius R are falling in a viscous fluid of viscosity η with a velocity v. The
retarding viscous force acting on the spherical ball is

(B) directly proportional to both radius R and velocity v.


e e.
(A) directly proportional to R but inversely proportional to v.

(C) inversely proportional to both radius R and velocity v.


rfr

(D) inversely proportional to R but directly proportional to velocity v.


fo

23. B.
es

Retarding viscous force = 6πηRv


t
no

24. If two soap bubbles of different radii are connected by a tube,


(A) air flows from the bigger bubble to the smaller bubble till the sizes are interchanged.
://

(B) air flows from bigger bubble to the smaller bubble till the sizes are interchanged
(C) air flows from the smaller bubble to the bigger.
tp

(D) there is no flow of air.


ht

24. C.
 4T 
The pressure inside the smaller bubble will be more  Pi = P0 +
 r 
Therefore, if the bubbles are connected by a tube, the air will flow from smaller bubble to the
bigger.

25. The bob of a simple pendulum executes simple harmonic motion in water with a period t,
while the period of oscillation of the bob is t0 in air. Neglecting frictional force of water and
4
given that the density of the bob is   × 1000 kg/m3. What relationship between t and t0 is
3
true?
(A) t = t0 (B) t = t0/2
(C) t = 2t0 (D) t = 4t0

25. C.

http://notesforfree.com/
http://notesforfree.com/
AIEEE PAPER-04-PH-6

T 1 1
= =
T0  ρ'  1
1 − ρ  1−
  3
T
⇒ =2
T0
or, T = 2T0

26. A particle at the end of a spring executes simple harmonic motion with a period t1, while the
corresponding period for another spring is t2. If the period of oscillation with the two springs
in series is t, then
(A) T = t1 + t2 (B) T 2 = t12 + t 22
(C) T −1 = t1−1 + t 2−1 (D) T −2 = t1−2 + t 2−2

26. B.
t12 + t 22 = T 2

27. The total energy of particle, executing simple harmonic motion is

/
(A) ∝ x (B) ∝ x2

m
(C) independent of x (D) ∝ x1/2

co
27. C.
e.
In simple harmonic motion, as a particle is displaced from its mean position, its kinetic
e
energy is converted to potential energy and vice versa and total energy remains constant.
rfr

The total energy of simple harmonic motion is independent of x.


fo

28. The displacement y of a particle in a medium can be expressed as


es

y = 10−6sin(110t + 20 x + π/4) m, where t is in seconds and x in meter. The speed of the


wave is
t
no

(A) 2000 m/s (B) 5 m/s


(C) 20 m/s (D) 5π m/s.
://

28. B.
tp

ω
v= = 5 ms −1
ht

29. A particle of mass m is attached to a spring (of spring constant k) and has a natural angular
frequency ω0. An external force F(t) proportional to cosωt (ω≠ω0) is applied to the oscillator.
The time displacement of the oscillator will be proportional to
m 1
(A) 2 (B)
ω0 − ω 2
m(ω0 − ω2 )
2

1 m
(C) (D) 2
m(ω0 + ω )
2 2
ω0 + ω2

29. B.
For forced oscillations, the displacement is given by
F /m
x = A sin(ωt + φ) with A = 20
ω0 − ω2

http://notesforfree.com/
http://notesforfree.com/
AIEEE PAPER-04-PH-7

30. In forced oscillation of a particle the amplitude is maximum for a frequency ω1 of the force,
while the energy is maximum for a frequency ω2 of the force, then
(A) ω1 = ω2
(B) ω1 > ω2
(C) ω1 < ω2 when damping is small and ω1 > ω2 when damping is large
(D) ω1 < ω2
30. A.
Both amplitude and energy get maximised when the frequency is equal to the natural
frequency. This is the condition of resonance.
ω1 = ω2
31. One mole of ideal monoatomic gas (γ = 5/30) is mixed with one mole of diatomic gas
(γ = 7/5). What is γ for the mixture? γ denotes the ratio of specific heat at constant pressure,
to that at constant volume.
(A) 3/2 (B) 23/15
(C) 35/23 (D) 4/3
31. A.
Q = Q1 + Q2
n1 + n2

/
n n

m
= 1 + 2
γ m − 1 γ1 − 1 γ 2 − 1

co
3
γm =
2
32.
e e.
If the temperature of the sun were to increase from T to 2T and its radius from R to 2R, then
rfr
the ratio of the radiant energy received on earth to what it was previously will be
(A) 4 (B) 16
fo

(C) 32 (D) 64.


es

32. D.
According to Stefan’s law,
t
no

P ∝ AT4 and A ∝ r2
P ∝ r2T4
://

33. Which of the following statements is correct for any thermodynamic system?
tp

(A) The internal energy changes in all processes.


ht

(B) Internal energy and entropy are state functions.


(C) The change in entropy can never be zero.
(D) The work done in an adiabatic process is always zero.

33. B.
34. Two thermally insulated vessels 1 and 2 are filled with air at temperatures (T1, T2), volume
(V1, V2) and pressure (P1, P2) respectively. If the valve joining two vessels is opened, the
temperature inside the vessel at equilibrium will be
(A) T1 + T2 (B) (T1 + T2)/2
T1T2 (P1V1 + P2 V2 ) T T (P V + P2 V2 )
(C) (D) 1 2 1 1
P1V1T2 + P2 V2 T1 P1V1T1 + P2 T2 T2

34. C.
The number of moles of system remains same
According to Boyle’s law,
P1V1 + P2V2 = P(V1 + V2)
T T (P V + P2 V2 )
∴ T= 1 2 1 1
P1V1T2 + P2 V2 T1

http://notesforfree.com/
http://notesforfree.com/
AIEEE PAPER-04-PH-8

35. A radiation of energy E falls normally on a perfectly reflecting surface. The momentum
transferred to the surface is
(A) E/c (B) 2E/c
(C) Ec (D) E/c2

35. B.
2E
∆Psurface = −∆P = .
c

36. The temperature of two outer surfaces of a composite slab, x 4x


consisting of two materials having coefficients of thermal
conductivity K and 2K and thickness x and 4x, respectively are
T2 and T1 (T2 > T1). The rate of heat transfer through the slab, T2 K 2K T1
 A(T2 − T1 )K 
in a steady state is   f, with f equal to
 x 
(A) 1 (B) ½
(C) 2/3 (D) 1/3

36. D.

/
2T − T1 

m
kA 
∆q = T2 − 2
x  3 

co
kA
= [T2 − T1 ]
3x
e e.
37. A light ray is incident perpendicular to one face of a 90° prism and is
rfr

totally internally reflected at the glass-air interface. If the angle of


fo

reflection is 45°, we conclude that the refractive index n


45°
1
es

(A) n < (B) n > 2 45°


2
t

1
no

(C) n > (D) n < 2


2
://

37. B.
tp

Angle of incidence i > C for total internal reflection.


ht

Here i = 45° inside the medium.


∴ 45° > sin−1(1/n)
⇒ n > √2.

38. A plane convex lens of refractive index 1.5 and radius of curvature 30 cm is silvered at the
curved surface. Now this lens has been used to form the image of an object. At what
distance from this lens an object be placed in order to have a real image of the size of the
object?
(A) 20 cm (B) 30 cm
(C) 60 cm (D) 80 cm
38. A.
1 2 1
= +
F f1 fm
1 1 1  1
and = (1.5 − 1)  − =
f1  ∞ −30  60
and fm = 15 cm.
∴ F = 10 cm.
Object should be placed at 20 cm from the lens.

http://notesforfree.com/
http://notesforfree.com/
AIEEE PAPER-04-PH-9

39. The angle of incidence at which reflected light totally polarized for reflection from air to glass
(refractive index n), is
(A) sin−1(n) (B) sin−1(1/n)
−1
(C) tan (1/n) (D) tan−1(n)

39. D.
Brewster’s law: According to this law the ordinary light is completely polarised in the plane
of incidence when it gets reflected from transparent medium at a particular angle known as
the angle of polarisation.
n = tan ip.

40. The maximum number of possible interference maxima for slit-separation equal to twice the
wavelength in Young’s double-slit experiment is
(A) infinite (B) five
(C) three (D) zero

40. B.
For interference maxima, d sin θ = nλ
Here d = 2λ

/
m
∴ sin θ = n/2 and is satisfied by 5 integral values of n (−2, −1, 0, 1, 2), as the maximum
value of sin θ can only be 1.

co
An electromagnetic wave of frequency ν = 3.0 MHz passes from vacuum into a dielectric
41.
medium with permittivity ε = 4.0. Then
e e.
(A) wavelength is doubled and the frequency remains unchanged
rfr
(B) wavelength is doubled and frequency becomes half
(C) wavelength is halved and frequency remains unchanged
fo

(D) wavelength and frequency both remain unchanged.


es

41. C.
t
no

ε
Refractive index, µ = =2
ε0
://

Speed and wavelength of wave will becomes half, the frequency remaining unchanged
tp

(frequency of a wave depends on the source as due to refraction, it is assumed that the
energy is conserved. hν remains the same)
ht

42. Two spherical conductor B and C having equal radii and carrying equal charges in them
repel each other with a force F when kept apart at some distance. A third spherical
conductor having same radius as that of B but uncharged brought in contact with B, then
brought in contact with C and finally removed away from both. The new force of repulsion,
between B and C is
(A) F/4 (B) 3F/4
(C) F/8 (D) 3F/8.

42. D.
1 (q / 2)(3q / 4) 3F
F' = = .
4πε0 d2 8

43. A charged particle q is shot towards another charged particle Q which is fixed, with a speed
v it approaches Q upto a closest distance r and then returns. If q were given a speed 2v, the
closest distances of approach would be
(A) r (B) 2r
(C) r/2 (D) r/4

http://notesforfree.com/
http://notesforfree.com/
AIEEE PAPER-04-PH-10

43. D.
By principle of conservation of energy
1 KqQ
mv 2 = …(i)
2 r
1 KqQ
Finally, m(2v)2 = 2 …(ii)
2 r
Equation (i) ÷ (ii),
1 r'
=
4 r
r
⇒ r' = .
4
44. Four charges equal to −Q are placed at the four corners of a square and a charge q is at its
centre. If the system is in equilibrium the value of q is
Q Q
(A) − (1 + 2 2) (B) (1 + 2 2)
4 4
Q Q
(C) − (1 + 2 2) (D) (1 + 2 2)
2 2

/
m
44. B.
Q

co
q= + (1 + 2 2)
4
45. e.
Alternating current can not be measured by D.C. ammeter because
(A) A.C. cannot pass through D.C.
e
rfr
(B) A.C. changes direction
(C) average value of current for complete cycle is zero
fo

(D) D.C. ammeter will get damaged.


es

45. C.
t
no

46. The total current supplied to the circuit by the battery is


2Ω 6Ω
(A) 1 A (B) 2 A
://

(C) 4 A (D) 6 A
6V 1.5 Ω
tp

46. C. 3Ω
ht

The given circuit can be written as


6V
I= = 4A .
1.5 Ω

47. The resistance of the series combination of two resistances is S. When they are joined in
parallel through total resistance is P. If S = nP, then the minimum possible value of n is
(A) 4 (B) 3
(C) 2 (D) 1
47. A.
Let resistances be R1 and R2
So, S = R1 + R2;
R1R2
P=
R1 + R 2
S = nP
nR1R 2
R1 + R 2 =
R1 + R 2
(R1 + R 2 )2 = nR1R 2
If R1 = R2, so minimum value of n = 4.

http://notesforfree.com/
http://notesforfree.com/
AIEEE PAPER-04-PH-11

48. An electric current is passed through a circuit containing two wires of the same material,
connected in parallel. If the length and radii of the wires are in the ratio of 4/3 and 2/3, then
the ratio of the currents passing through the wire will be
(A) 3 (B) 1/3
(C) 8/9 (D) 2.

48. B.
I1 R2
=
I2 R1
[current divider rule since voltage is same in parallel]
I1 L 2 r12
= ×
I2 L1 r22
2
I 3 2 1
∴ 1 = ×  = .
I2 4  3  3

49. In a metre bridge experiment null point is obtained at 20 cm from one end of the wire when
resistance X is balanced against another resistance Y. If X < Y, then where will be the new

/
position of the null point from the same end, if one decides to balance a resistance of 4X

m
against Y?

co
(A) 50 cm (B) 80 cm
(C) 40 cm (D) 70 cm

49. A.
e e.
rfr
We have from meter bridge experiment,
R1 l 1
= , where l2 = (100 − l1) cm
fo

R2 l 2
es

In the first case, X/Y = 20/80


4X l
=
t

In the second case


no

Y 100 − l
l = 50 cm.
://

50. The thermistors are usually made of


tp

(A) metals with low temperature coefficient of resistivity


ht

(B) metals with high temperature coefficient of resistivity


(C) metal oxides with high temperature coefficient of resistivity ‘
(D) semiconducting materials having low temperature coefficient of resistivity.

50. C.
These are devices whose resistance varies quite markedly with temperature mean having
high temperature coefficient of resistivity. [Their name are derived from thermal resistors].
Depending on their composition they can have either negative temperature coefficient or
positive temperature coefficient or positive temperature coefficient or positive temperature
coefficient characteristics.
The negative temperature coefficient types consists of a mixture of oxides of iorn, nickel and
cobalt with small amounts of other substance. The positive temperature coefficient types are
based on barium titanate.

51. Time taken by a 836 W heater to heat one litre of water from 10°C to 40°C is
(A) 50 s (B) 100 s
(C) 150 s (D) 200 s

51. C.

http://notesforfree.com/
http://notesforfree.com/
AIEEE PAPER-04-PH-12

Let t be the time taken, then


836 × t
= 1000 × 1× (40 − 10) [using Q = mst]
4.2
⇒ t = 150 sec.

52. The thermo emf of a thermocouple varies with the temperature θ of the hot junction as
E = a θ + bθ2 in volts where the ratio a/b is 700°C. If the cold junction is kept at 0°C, then the
neutral temperature is
(A) 700°C
(B) 350°C
(C) 1400°C
(D) no neutral temperature is possible for this thermocouple.

52. D.
E = aθ + bθ2
At neutral temperature dE/dθ = 0
dE a
∴ = a + 2bθn = 0 ; θn = −
dθ 2b
a

/
Now = 700°C (given)

m
b
θn =−700/2 = −350°C

co
Now θc = 0°C.
So, θn > 0°C
But mathematically θn < 0°C .
e e.
rfr

53. The electrochemical equivalent of a metal is 3.3 × 10−7 kg per coulomb. The mass of the
fo

metal liberated at the cathode when a 3 A current is passed for 2 seconds will be
es

(A) 19.8 × 10−7 kg (B) 9.9 × 10−7 kg


(C) 6.6 × 10−7 kg (D) 1.1 × 10−7 kg
t
no

53. A.
m = Zit,
://

m = 3.3 × 10−7 × 3 × 2 = 19.8 × 10−7 kg.


tp

54. A current I ampere flows along an infinitely long straight thin walled tube, then the magnetic
ht

induction at any point inside the tube is


(A) infinite (B) zero
µ0 2i 2i
(C) tesla (D) tesla
4π r r

54. B.
Considering Ampere’s loop (shown by dotted line), no current is enclosed by this loop.
Therefore, the magnetic field will be zero inside the tube.

55. A long wire carries a steady current. It is bent into a circle of one turn and the magnetic field
at the centre of the coil is B. It is then bent into a circular loop of n turns. The magnetic field
at the centre of the coil will be
(A) nB (B) n2 B
(C) 2nB (D) 2n2B
55. B.
nµ 0 i µ iπ
B' = = n2 0 = n2B .
2r ' l

http://notesforfree.com/
http://notesforfree.com/
AIEEE PAPER-04-PH-13

56. The magnetic field due to a current carrying circular loop of radius 3 cm at a point on the axis
at a distance of 4 cm from the centre is 54 µT. What will be its value at the centre of the
loop?
(A) 250 µT (B) 150 µT
(C) 125 µT (D) 75 µT
56. A.
µ0iR 2
Using formula B = , we get
2(R + X2 )3 / 2
2

µ0i(3)2
54 = …(i)
2[(3)2 + (4)2 ]3 / 2
µ 0i
At the centre of the coil, X = 0 and B =
2(3)
Using equation (i)
54 × 53
B= ⇒ B = 250 µT.
(3)2 × 3

57. Two long conductors, separated by a distance d carry current I1 and I2 in the same direction.

/
They exert a force F on each other. Now the current in one of them increased to two times

m
and its direction reversed. The distance is also increased to 3d. The new value of the force

co
between them is
(A) −2F (B) F/3

57.
(C) −2F/3
C.
(D) −F/3 e e.
rfr
Force between two long conductor carrying current
µ II
fo

F = 0 12 l
2π d
es

According to question
µ ( −2I1 )(I2 )
t

F' = 0 l
no

2π d
3
From equation (i) and (ii), F ' = − F.
://

2
tp

58. The length of a magnet is large compared to its width and breadth. The time period of its
ht

width and breadth. The time period of its oscillation in a vibration magnetometer is 2 s. The
magnet is cut along its length into three equal parts and three parts are then placed on each
other with their like poles together. The time period of this combination will be
(A) 2 s (B) 2/3 s
(C) 2√3 s (D) 2/√3 s .
58. B.
T
Time period of vibration, T = 2π
MB
Where l = moment of inertia of magnet, M = magnetic moment
ml 2
I= and M = pole strength × l
12
2
1  m  l  I
I' =     ×3 =
12  3   3  9
and M’ = pole strength (will remain the same) × (l/3) × 3 = M.
T 2
T' = = s.
9 9

http://notesforfree.com/
http://notesforfree.com/
AIEEE PAPER-04-PH-14

59. The materials suitable for making electromagnets should have


(A) high retentivity and high coercivity (B) low retentivity and low coercivity
(C) high retentivity and low coercivity (D) low retentivity and high coercivity

59. B.

60. In an LCR series a.c. circuit, the voltage across each of the components, L, C and R is 50 V.
The voltage across the LC combination will be
(A) 50 V (B) 50√2 V
(C) 100 V (D) 0 V(zero)
60. D.
In series LCR circuit, the voltage across the inductor (L) and the capacitor (C) are in
opposite phase.
61. A coil having n turns and resistance 4R Ω. This combination is moved in time t seconds from
a magnetic field W1 weber to W2 weber. The induced current in the circuit is
W − W1 (W2 − W1 )
(A) − 2 (B) −
5Rnt 5Rt
W2 − W1 n(W2 − W1 )
(C) − (D) −

/
m
Rnt Rt

co
61. B.
n dφ
I=−
R ' dt
1  W − W1 
e e.
or, I = − n  2 
rfr
R '  t 2 − t1 
fo

(W1 and W2 are not the magnetic field, but the values of flux associated with one turn of coil)
−1 n(W2 − W1 )
es

I=
(R + 4R) t
t

n(W2 − W1 )
no

or, I = −
5Rt
://

62. In a uniform magnetic field of induction B a wire in the form of semicircle of radius r rotates
tp

about the diameter of the circle with angular frequency ω. The axis of rotation is
perpendicular to the field. If the total resistance of the circuit is R the mean power generated
ht

per period of rotation is


Bπr 2 ω (Bπr 2 ω)2
(A) (B)
2R 2R
(Bπrω)2 (Bπrω2 )2
(C) (D)
2R 8R
62. B.
πr 2
Magnetic flux = BA cos θ = B ⋅ cos ωt
2
dφ 1
∴ εind = − = Bπr 2 ω sin ωt
dt 2
ε2 B2 π2r 4 ω2 sin2 ωt
∴ P = ind =
R 4R
Now, <sin2 ωt> = ½ (mean value)
(Bπr 2 ω)2
∴ <P > = .
8R

http://notesforfree.com/
http://notesforfree.com/
AIEEE PAPER-04-PH-15

63. In a LCR circuit capacitance is changed from C to 2C. For the resonant frequency to remain
unchanged, the inductance should be changed from L to
(A) 4L (B) 2L
(C) L/2 (D) L/4

63. C.
1
ωres =
LC
if ωres is to remain same, the product LC should also not change.
⇒ LC = L′C′
⇒ LC = L′2C
⇒ L′ = L/2

64. A metal conductor of length 1 m rotates vertically about one of its ends at angular velocity 5
radians per second. If the horizontal component of earth’s magnetic field is 0.3 × 10−4 T, then
the e.m.f. developed between the two ends of the conductor is
(A) depends on the nature of the metal used
(B) depends on the intensity of the radiation
(C) depends both on the intensity of the radiation and the metal used

/
(D) is the same for all metals and independent of the intensity of the radiation.

m
co
64. B.
emf. developed is given by
1
εind = BωR 2 = 50 µV.
2
e e.
rfr

65. According to Einstein’s photoelectric equation, the plot of the kinetic energy of the emitted
fo

photo electrons from a metal Vs the frequency, of the incident radiation gives straight line
whose slope
es

(A) depends on the nature of the metal used


t

(B) depends on the intensity of the radiation


no

(C) depends both on the intensity of the radiation and the metal used
(D) is the same for all metals and independent of the intensity of the radiation.
://
tp

65. D. K.E.
KEmax = hν − W {y = mx + C}
ht

Slope of the line in the graph is h, the


Planck’s constant. θ=h
W ν

66. The work function of a substance is 4.0 eV. Then longest wavelength of light that can cause
photoelectron emission from this substance approximately
(A) 540 nm (B) 400 nm
(C) 310 nm (D) 220 nm

66. C.
hc
=W
λ
hc 6.6 × 10 −34 × 3 × 108
λ longest = =
W 4.0 × 1.6 × 10 −19
⇒ λ longest ≈ 310 nm.

http://notesforfree.com/
http://notesforfree.com/
AIEEE PAPER-04-PH-16

67. A charged oil drop is suspended in a uniform field of 3 × 104 V/m so that it neither falls nor
rises. The charge on the drop will be (take the mass of the charge = 9.9 × 10−15 kg and g =
10 m/s2)
(A) 3.3 × 10−18 C (B) 3.2 × 10−18 C
(C) 1.6 × 10−18 C (D) 4.8 × 10−18 C.

67. A.
Since ball is hanging in equilibrium, force by gravity is balanced by electric force.
qE = mg
m×g
⇒ q=
E
9.9 × 10−15 × 10

3 × 104
∴ q = 3.3 × 10−18 C

68. A nucleus disintegrates into two nuclear parts which have their velocities in the ratio 2 : 1.
The ratio of their nuclear sizes will be
(A) 21/3 : 1 (B) 1 : 3 1/2
1/2
(C) 3 : 1 (D) 1 : 21/3

/
m
68. B.

co
1/ 3
R1  m2 
= 
R 2  2m2 
R
⇒ 1 = 1: 21/ 3 .
e e.
rfr
R2
fo

69. The binding energy per nucleon of deuteron (12 H) and helium nucleus ( 24 He) is 1.1 MeV and
es

7 MeV respectively. If two deuteron nuclei react to form a single helium nucleus, then the
energy released is
t
no

(A) 13.9 MeV (B) 26.9 MeV


(C) 23.6 MeV (D) 19.2 MeV
://

69. C.
tp

Energy released = total binding energy of product − total binding energy of reactants
ht

⇒ 28 − (2 × 2.2) = 28 − 4.4 = 236 MeV.

70. An α-particle of energy 5 MeV is scattered through 180° by a fixed uranium nucleus. The
distance of the closest approach is of the order of
(A) 1 Å (B) 10−10 cm
−12
(C) 10 cm (D) 10−15 cm

70. C.
At closest approach, all the kinetic energy of the α-particle will converted into the potential
energy of the system, K.E. = P.E.
1 q1q2
5 MeV =
4πε0 r
Z1 × Z 2 e2
5 × 106 × e = 9 × 109
r
9 × 10 × 92 × 2 × 1.6 × 10−19
9

r=
5 × 106
∴ r = 5.3 × 10−14 m = 5.3 × 10−12 cm.

http://notesforfree.com/
http://notesforfree.com/
AIEEE PAPER-04-PH-17

71. When npn transistor is used as amplifier


(A) electrons move from base to collector (B) holes move from emitter to base
(C) electrons move from collector to base (D) holes move from base to emitter.

71. A.
When npn transistor is used, majority charge carrier electrons of n type emitter move from
emitter to base and then base to collector.

72. For a transistor amplifier in common emitter configuration having load impedance of 1 kΩ (hfe
= 50 and hoe = 25) the current gain is
(A) −5.2 (B) −15.7
(C) −24.8 (D) −48.78

72. D.
−hfe
In CE configuration, A i =
1 + h0eRL
−50
= = −48.78
1 + 25 × 10−6 × 1× 103

/
m
73. A piece of copper and another of germanium are cooled from room temperature to 77 K, the

co
resistance of
(A) each of them increases
(B) each of them decreases
(C) copper decreases and germanium increases
e e.
rfr
(D) copper increases and germanium decreases.
fo

73. D.
Copper is metallic conductor and germanium is semiconductor therefore as temperature
es

decreases resistance of good conductor decreases while for semiconductor it increases.


t
no

74. The manifestation of band structure in solids is due to


(A) Heisenberg’s uncertainty principle (B) Pauli’s exclusion principle
://

(C) Bohr’s correspondence principle (D) Boltzmann’s law


tp

74. B.
ht

75. When p-n junction diode is forward biased


(A) the depletion region is reduced and barrier height is increased
(B) the depletion region is widened and barrier height is reduced.
(C) both the depletion region and barrier height reduced
(D) both the depletion region and barrier height increased.

75. C.

http://notesforfree.com/
http://notesforfree.com/
AIEEE-2004-1

AIEEE
76. Which of the following sets of quantum numbers is correct for an electron in 4f orbital?
1 1
(1) n = 4, I =3, m = +4, s = + (2) n = 3, I = 2, m = -2, S = +
2 2
1 1
(3) n =4, I = 3, m = +1, s = + (4) n =4, I = 4, m -4, s = -
2 2
1
Ans. n =4, I = 3, m = +1, s = +
2

77. Consider the ground state of Cr atom (Z = 24). The number of electrons with the azimuthal
quantum numbers I =1 and 2 are respectively
(1) 12 and 4 (2) 16 and 5
(3) 16 and 4 (4) 12 and 5

Ans. 12 and 5

78. Which one the following ions has the highest value of ionic radius?
(1) Li+ (2) F-

/
2-
(4) B3+

m
(3) O

co
Ans. O2-

79. e.
The wavelength of the radiation emitted, when in hydrogen atom electron falls from infinity to
e
stationary state 1, would be (Rydberg constant = 1.097×107 m-1)
rfr
(1) 91 nm (2) 9.1×10-8 nm
(3) 406 nm (4) 192 nm
fo
es

Ans. 91 nm
t

80. The correct order of bond angles (smallest first) in H2S, NH3, BF3 and SiH4 is
no

(1) H2S < SiH4 < NH3 < BF3 (2) H2S < NH3 < BF3 < SiH4
(3) H2S < NH3 < SiH4 < BF3 (4) NH3 < H2S < SiH4 < BF3
://
tp

Ans. H2S < NH3 < SiH4 < BF3


ht

81. Which one the following sets of ions represents the collection of isoelectronic species?
(1) K+, Ca2+, Sc3+, Cl- (2) Na+, Mg2+, Al3+, Cl-
+ - 2+ 3+
(3) K , Cl , Mg , Sc (4) Na+, Ca2+, Sc3+, F-

Ans. K+, Ca2+, Sc3+, Cl-

82. Among Al2O3, SiO2, P2O3 and SO2 the correct order of acid strength is
(1) SO2 < P2O3 < SiO2 < Al2O3 (2) Al2O3 < SiO2 < P2O3 < SO2
(3) Al2O3 < SiO2 < SO2 < P2O3 (4) SiO2 < SO2 < Al2O3 < P2O3

Ans. Al2O3 < SiO2 < P2O3 < SO2

83. The bond order in NO is 2.5 while that in NO+ is 3. Which of the following statements is true
for these two species?
(1) Bond length in NO+ is greater than in NO
(2) Bond length is unpredictable
(3) Bond length in NO+ in equal to that in NO
(4) Bond length in NO is greater than in NO+

http://notesforfree.com/
http://notesforfree.com/
AIEEE-2004-2
Ans. Bond length in NO is greater than in NO+

84. The formation of the oxide ion O2-(g) requires first an exothermic and then an endothermic
step as shown below
O(g) + e− O− (g)∆Ho = −142kJmol−1
O− (g) + e− O2− (g)∆Ho = 844kJmol−1
(1) Oxygen is more electronegative
(2) O- ion has comparatively larger size than oxygen atom
(3) O- ion will tend to resist the addition of another electron
(4) Oxygen has high electron affinity

Ans. O- ion will tend to resist the addition of another electron

85. The states of hybridization of boron and oxygen atoms in boric acid (H3BO3) are respectively
(1) sp2 and sp2 (2) sp3 and sp3
3 2
(3) sp and sp (4) sp2 and sp3

Ans. sp2 and sp3

86. Which one of the following has the regular tetrahedral structure?

/
(2) [Ni(CN)4]2-

m
(1) XeF4
-
(3) BF4 (4) SF4

co
Ans. BF4-

87.
e e.
Of the following outer electronic configurations of atoms, the highest oxidation state is
rfr
achieved by which one of them?
(1) (n -1)d8ns2 (2) (n-1)d5ns2
fo

3 2
(3) (n-1)d ns (4) (n-1)d5ns-1
es

Ans. (n-1)d5ns2
t
no

88. As the temperature is raised from 20°C to 40°C, the average kinetic energy of neon atoms
changes by a factor of which of the following?
://

(1) ½ (2) 2
tp

313 313
(3) (4)
293 293
ht

313
Ans.
293

89. The maximum number of 90° angles between bond pair of electrons is observed in
(1) dsp3 hybridization (2) sp3d2 hybridization
2
(3) dsp hybridization (4) sp3d hybridization

Ans. sp3d2 hybridization

90. Which one of the following aqueous solutions will exhibit highest boiling point?
(1) 0.01 M Na2SO4 (2) 0.015 M glucose
(3) 0.015 M urea (4) 0.01 M KNO3

Ans. 0.01 M Na2SO4

91. Which among the following factors is the most important in making fluorine the strongest
oxidizing halogen?

http://notesforfree.com/
http://notesforfree.com/
AIEEE-2004-3
(1) Electron affinity (2) Bond dissociation energy
(3) Hydration enthalpy (4) Ionization enthalpy

Ans. Bond dissociation energy

92. In Vander Waals equation of state of the gas law, the constant ‘b’ is a measure of
(1) intermolecular repulsions (2) intermolecular collisions per unit volume
(3) Volume occupied by the molecules (4) intermolecular attraction

Ans. Volume occupied by the molecules

93. The conjugate base of H2PO4- is


(1) PO43- (2) HPO42-
(3) H3PO4 (4) P2O5

Ans. HPO42-

94. 6.02×1020 molecules of urea are present in 100 ml of its solution. The concentration of urea
solution is
(1) 0.001 M (2) 0.1 M
(3) 0.02 M (4) 0.01 M

/
m
Ans. 0.01 M

co
95. To neutralize completely 20 mL of 0.1 M aqueous solution of phosphorous acid (H3PO3), the
volume of 0.1 M aqueous KOH solution required is
(1) 10 mL
e
(2) 60 mL
e.
rfr
(3) 40 mL (4) 20 mL
fo

Ans. 40 mL
es

96. For which of the following parameters the structural isomers C2H5OH and CH3OCH3 would
t

be expected to have the same values?


no

(Assume ideal behaviour)


(1) Heat of vaporization
://

(2) Gaseous densities at the same temperature and pressure


tp

(3) Boiling points


(4) Vapour pressure at the same temperature
ht

Ans. Gaseous densities at the same temperature and pressure

97. Which of the following liquid pairs shows a positive deviation from Raoult’s law?
(1) Water – hydrochloric acid (2) Acetone – chloroform
(3) Water – nitric acid (4) Benzene – methanol

Ans. Benzene – methanol

98. Which one of the following statements is false?


(1) Raoult’s law states that the vapour pressure of a components over a solution is
proportional to its mole fraction
(2) Two sucrose solutions of same molality prepared in different solvents will have the same
freezing point depression
(3) The correct order of osmotic pressure for 0.01 M aqueous solution of each compound is
BaCl2 > KCl > CH3COOH > sucrose
(4) The osmotic pressure (π) = MRT, where M is the molarity of the solution

http://notesforfree.com/
http://notesforfree.com/
AIEEE-2004-4
Ans. Two sucrose solutions of same molality prepared in different solvents will have the same
freezing point depression

99. What type of crystal defect is indicated in the diagram below?


Na+ Cl- Na+Cl- Na+Cl-
Cl- Cl- Na+ Na+
+ -
Na Cl Cl Na+ Cl-
-

Cl Na Cl Na+
- + -
Na+
(1) Frenkel defect (2) Frenkel and Schottky defects
(3) Interstitial defect (4) Schottky defect

Ans. Schottky defect

100. An ideal gas expands in volume from 1×10-3 m3 to 1×10-2 m3 at 300 K against a constant
pressure of 1×105 Nm-2. The work done is
(1) -900 J (2) 900 kJ
(3) 2780 kJ (4) -900 kJ

Ans. -900 J

/
m
101. In hydrogen – oxygen fuel cell, combustion of hydrogen occurs to
(1) generate heat

co
(2) remove adsorbed oxygen from electrode surfaces
(3) produce high purity water
e.
(4) create potential difference between the two electrodes
e
rfr
Ans. create potential difference between the two electrodes
fo

102. In first order reaction, the concentration of the reactant decreases from 0.8 M to 0.4 M in 15
es

minutes. The time taken for the concentration to change from 0.1 M to 0.025 M is
(1) 30 minutes (2) 60 minutes
t

(3) 7.5 minutes (4) 15 minutes


no

Ans. 30 minutes
://
tp

103. What is the equilibrium expression for the reaction P4(s) +5O2(g) P4O10(s)?
(1) Kc = [P4O10] / P4] [O2]5 (2) Kc = 1/[O2]5
ht

(3) Kc = [O2]5 (4) Kc = [P4O10] / 5[P4][O2]

Ans. Kc = 1/[O2]5

Kp
104. For the reaction, CO(g) + Cl2(g) COCl2(g) the is equal to
Kc
1
(1) (2) 1.0
RT
(3) RT (4) RT

1
Ans.
RT

105. The equilibrium constant for the reaction N2(g) + O2(g) 2NO(g) at temperature T is
1 1
4×10-4. The value of Kc for the reaction NO(g) N2 (g) + O2 (g) at the same
2 2
temperature is

http://notesforfree.com/
http://notesforfree.com/
AIEEE-2004-5
(1) 2.5×102 (2) 0.02
(3) 4×10-4 (4) 50

Ans. 50

106. The rate equation for the reaction 2A + B → C is found to be: rate k[A][B]. The correct
statement in relation to this reaction is that the
(1) unit of K must be s-1
(2) values of k is independent of the initial concentration of A and B
(3) rate of formation of C is twice the rate of disappearance of A
(4) t1/2 is a constant

Ans. values of k is independent of the initial concentration of A and B

107. Consider the following E° values


EoFe3 + / Fe2 + = 0.77 V
EoSn2 + / Sn = −0.14V
Under standard conditions the potential for the reaction

/
Sn(s) + 2Fe3+(aq) → 2Fe2+(aq) + Sn2+(aq) is

m
(1) 1.68 V (2) 0.63 V

co
(3) 0.91 V (4) 1.40 V

Ans. 0.91 V e e.
108. The molar solubility product is Ksp. ‘s’ is given in terms of Ksp by the relation
rfr
1/ 4 1/ 5
 K sp   K sp 
(1) s =  (2) s = 
fo

 
 128   256 
es

( )
1/ 5
(3) s = 256K sp (4) s = (128Ksp)1/4
t
no

1/ 5
 K sp 
s=
://

Ans. 
 256 
tp
ht

109. The standard e.m.f of a cell, involving one electron change is found to be 0.591 V at 25°C.
The equilibrium constant of the reaction is (F = 96,500 C mol-1: R = 8.314 JK-1 mol-1)
(1) 1.0×101 (2) 1.0×1030
10
(3) 1.0×10 (4) 1.0×105

Ans. 1.0×1010

110. The enthalpies of combustion of carbon and carbon monoxide are -393.5 and -283 kJ mol-1
respectively. The enthalpy of formation of carbon monoxide per mole is
(1) 110.5 kJ (2) -110.5 kJ
(3) -676.5 kJ (4) 676.5 kJ

Ans. -110.5 kJ

111. The limiting molar conductivities Λ° for NaCl, KBr and KCl are 126, 152 and 150 S cm2 mol-1
respectively. The Λ° for NaBr is
(1) 128 S cm2 mol-1 (2) 302 S cm2 mol-1
2 -1
(3) 278 S cm mol (4) 176 S cm2 mol-1

http://notesforfree.com/
http://notesforfree.com/
AIEEE-2004-6
Ans. 128 S cm2 mol-1

112. In a cell that utilises the reaction Zn(s) + 2H+(aq) → Zn2+(aq) + H2(g) addition of
H2SO4 to cathode compartment, will
(1) lower the E and shift equilibrium to the left
(2) increases the E and shift equilibrium to the left
(3) increase the E and shift equilibrium to the right
(4) Lower the E and shift equilibrium to the right

Ans. increase the E and shift equilibrium to the right

113. Which one the following statement regarding helium is incorrect?


(1) It is used to fill gas balloons instead of hydrogen because it is lighter and non –
inflammable
(2) It is used in gas – cooled nuclear reactors
(3) It is used to produce and sustain powerful superconducting reagents
(4) It is used as cryogenic agent for carrying out experiments at low temperatures

Ans. It is used to fill gas balloons instead of hydrogen because it is lighter and non – inflammable

/
m
114. Identify the correct statements regarding enzymes
(1) Enzymes are specific biological catalysts that can normally function at very high

co
temperature (T ~ 1000 K)
(2) Enzymes are specific biological catalysts that the posses well – defined active sites
e e.
(3) Enzymes are specific biological catalysts that can not be poisoned
(4) Enzymes are normally heterogeneous catalysts that are very specific in their action
rfr

Ans. Enzymes are specific biological catalysts that the posses well – defined active sites
fo
es

115. One mole of magnesium nitride on the reaction with an excess of water gives
(1) one mole of ammonia (2) two moles of nitric acid
t

(3) two moles of ammonia (4) one mole of nitric acid


no

Ans. two moles of ammonia


://
tp

116. Which one of the following ores is best concentrated by froth – floatation method?
(1) Magnetite (2) Malachite
ht

(3) Galena (4) Cassiterite

Ans. Galena

117. Beryllium and aluminium exhibit many properties which are similar. But the two elements
differ in
(1) exhibiting maximum covalency in compound
(2) exhibiting amphoteric nature in their oxides
(3) forming covalent halides
(4) forming polymeric hydrides

Ans. exhibiting maximum covalency in compound

118. Aluminium chloride exists as dimer, Al2Cl6 in solid state as well as in solution of non-polar
solvents such as benzene. When dissolved in water, it gives
(1) Al3+ + 3Cl- (2) Al2O3 + 6HCl
(3) [Al(OH)6]3- (D) [Al(H2O)6]3+ + 3Cl-

Ans. [Al(H2O)6]3+ + 3Cl-

http://notesforfree.com/
http://notesforfree.com/
AIEEE-2004-7

119. The soldiers of Napolean army while at Alps during freezing winter suffered a serious
problem as regards to the tin buttons of their uniforms. White metallic tin buttons got
converted to grey powder. This transformation is related to
(1) an interaction with nitrogen of the air at very low temperatures
(2) an interaction with water vapour contained in the humid air
(3) a change in the partial pressure of oxygen in the air
(4) a change in the crystalline structure of tin

Ans. a change in the crystalline structure of tin

120. The EoM+3 / M2 + values for Cr, Mn, Fe and Co are – 0.41, +1.57, + 0.77 and +1.97 V
respectively. For which one of these metals the change in oxidation state form +2 to +3 is
easiest?
(1) Cr (2) Co
(3) Fe (4) Mn

Ans. Cr

/
121. Excess of KI reacts with CuSO4 solution and then Na2S2O3 solution is added to it. Which of

m
the statements is incorrect for this reaction?

co
(1) Cu2I2 is reduced (2) Evolved I2 is reduced
(3) Na2S2O3 is oxidized (4) CuI2 is formed

Ans. CuI2 is formed


e e.
rfr

122. Among the properties (a) reducing (b) oxidising (c) complexing, the set of properties shown
fo

by CN- ion towards metal species is


(1) a, b (2) a, b, c
es

(3) c, a (4) b, c
t
no

Ans. c, a
://

123. The coordination number of central metal atom in a complex is determined by


tp

(1) the number of ligands around a metal ion bonded by sigma bonds
ht

(2) the number of only anionic ligands bonded to the metal ion
(3) the number of ligands around a metal ion bonded by sigma and pi- bonds both
(4) the number of ligands around a metal ion bonded by pi-bonds

Ans. the number of ligands around a metal ion bonded by sigma

124. Which one of the following complexes in an outer orbital complex?


(1) [Fe(CN)6]4- (2) [Ni(NH3)6]2+
3+
(3) [Co(NH3)6] (4) [Mn(CN)6]4-

Ans. [Ni(NH3)6]2+

125. Coordination compound have great importance in biological systems. In this context which of
the following statements is incorrect?
(1) Chlorophylls are green pigments in plants and contains calcium
(2) Carboxypeptidase – A is an enzyme and contains zinc
(3) Cyanocobalamin is B12 and contains cobalt
(4) Haemoglobin is the red pigment of blood and contains iron

http://notesforfree.com/
http://notesforfree.com/
AIEEE-2004-8

Ans. Chlorophylls are green pigments in plants and contains calcium

126. Cerium (Z = 58) is an important member of the lanthanoids. Which of the following
statements about cerium is incorrect?
(1) The common oxidation states of cerium are +3 and +4
(2) Cerium (IV) acts as an oxidizing agent
(3) The +4 oxidation state of cerium is not known in solutions
(4) The +3 oxidation state of cerium is more stable than the +4 oxidation state

Ans. The +4 oxidation state of cerium is not known in solutions

127. Which one the following has largest number of isomers?


(1) [Ru(NH3)4Cl2+] (2) [Co(en)2Cl2]+
2+
(3) [Ir(PR3)2 H(CO)] (4) [Co(NH3)5Cl]2+
(R -= alkyl group, en = ethylenediamine)

Ans. [Co(en)2Cl2]+

128. The correct order of magnetic moments (spin only values in B.M.) among is
(1) [MnCl4]2- > [CoCl4]-2 > [Fe(CN)6]-4 (2) [Fe(CN)6]-4 > [CoCl4]2- > [MnCl4]2-

/
m
4- 2- 2-
(3) [Fe(CN)6] > [MnCl4] > [CoCl4] (4) [MnCl4]2- > [Fe(CN)6]4- > [CoCl4]2-
(Atomic numbers: Mn = 25; Fe = 26, Co =27)

co
Ans. [MnCl4]2- > [CoCl4]-2 > [Fe(CN)6]-4

129. Consider the following nuclear reactions


e e.
rfr
238 x 4
92 M → y N + 2 He
fo

x
yN →BA L + 2β+
es

The number of neutrons in the element L is


(1) 142 (2) 146
t

(3) 140 (4) 144


no

Ans. 144
://
tp

130. The half – life of a radioisotope is four hours. If the initial mass of the isotope was 200 g, the
mass remaining after 24 hours undecayed is
ht

(1) 1.042 g (2) 4.167 g


(3) 3.125 g (4) 2.084 g

Ans. 3.125 g

131. The compound formed in the positive test for nitrogen with the Lassaigne solution of an
organic compound is
(1) Fe4[Fe(CN)6]3 (2) Na4[Fe(CN)5NOS]
(3) Fe(CN)3 (4) Na3[Fe(CN)6]

Ans. Fe4[Fe(CN)6]3

132. The ammonia evolved from the treatment of 0.30 g of an organic compound for the
estimation of nitrogen was passed in 100 mL of 0.1 M sulphuric acid. The excess of acid
required 20 mL of 0.5 M sodium hydroxide solution hydroxide solutio for complete
neutralization. The organic compound is
(1) acetamide (2) thiourea
(3) urea (4) benzamide

http://notesforfree.com/
http://notesforfree.com/
AIEEE-2004-9
Ans. urea

133. Which one of the following has the minimum boiling point?
(1) n-butane (2) isobutane
(3) 1- butene (4) 1- butyne

Ans. isobutane

134. The IUPAC name of the compound

HO
(1) 3, 3- dimethyl -1- hydroxy cyclohexane (2) 1,1 – dimethyl -3- cyclohexanol
(3) 3,3- dimethyl -1- cyclohexanol (4) 1,1 – dimethyl -3- hydroxy cyclohexane

Ans. 3,3- dimethyl -1- cyclohexanol

135. Which one the following does not have sp2 hybridized carbon?
(1) Acetone (2) Acetamide

/
m
(3) Acetonitrile (4) Acetic acid

co
Ans. Acetonitrile

136. Which of the following will have meso-isomer also?


(1) 2- chlorobutane
e e.
(2) 2- hydroxyopanoic acid
rfr
(3) 2,3 – dichloropentane (4) 2-3- dichlorobutane
fo

Ans. 2-3- dichlorobutane


es

137. Rate of the reaction


t

O O
no

R + Nu− → R +Z−


://

Z Nu
tp

is fastest when Z is
ht

(1) Cl (2) OCOCH3


(3) OC2H5 (4) NH2

Ans. Cl

138. Amongst the following compound, the optically active alkane having lowest molecular mass
is
(1) H3C (2) H3C
CH3
CH
CH3 CH3

(3) H3C (4 )
C 2H 5 H3C CH3

CH3

Ans. H3C

C 2H 5

http://notesforfree.com/
http://notesforfree.com/
AIEEE-2004-10

139. Consider the acidity of the carboxylic acids:


(1) PhCOOH (2) o – NO2C6H4COOH
(3) p – NO2C6H4COOH (4) m – NO2C6H4COOH

Ans. o – NO2C6H4COOH

140. Which of the following is the strongest base?

(1) NH2 (2)


NH2

NH2
(3) (4 ) NH

CH3
CH3

Ans.

/
m
NH2

co
141. Which base is present in RNA but not in DNA?
(1) Uracil
(3) Guanine
(2) Thymine
e e.
(4) Cytosine
rfr

Ans. Uracil
fo

142. The compound formed on heating chlorobenzene with chloral in the presence concentrated
es

sulphuric acid is
t

(1) gammexene (2) hexachloroethane


no

(3) Freon (4) DDT


://

Ans. DDT
tp

143. On mixing ethyl acetate with aqueous sodium chloride, the composition of the resultant
ht

solution is
(1) CH3COOC2H5 + NaCl (2) CH3Cl + C2H5COONa
(3) CH3COCl + C2H5OH + NaOH (4) CH3COONa + C2H5OH

Ans. CH3COOC2H5 + NaCl

144. Acetyl bromide reacts with excess of CH3MgI followed by treatment with a saturated solution
of NH4Cl given
(1) acetone (2) acetyl iodide
(3) 2- methyl -2- propanol (4) acetamide

Ans. 2- methyl -2- propanol

145. Which one of the following reduced with zinc and hydrochloric acid to give the corresponding
hydrocarbon?
(1) Ethyl acetate (2) Butan -2-one
(3) Acetamide (4) Acetic acid

Ans. Butan -2-one

http://notesforfree.com/
http://notesforfree.com/
AIEEE-2004-11

146. Which of the following undergoes reaction with 50% sodium hydroxide solution to give the
corresponding alcohol and acid?
(1) Phenol (2) Benzoic acid
(3) Butanal (4) Benzaldehyde

Ans. Benzaldehyde

147. Among the following compound which can be dehydrated very easily is
H3C H3C

(1) (2)
OH H3C OH
CH3
OH

(3) (D )
H3C CH3 H3C CH3
OH

CH3

Ans.

/
m
H3C OH CH3

co
148. Which of the following compound is not chiral?
(1) 1- chloropentane (2) 3-chloro-2- methyl pentane
(3) 1-chloro -2- methyl pentane e.
(4) 2- chloropentane
e
rfr
Ans. 1- chloropentane
fo

149. Insulin production and its action in human body are responsible for the level of diabetes. This
es

compound belongs to which of the following categories?


(1) A co- enzyme (2) An antibiotic
t

(3) An enzyme (4) A hormone


no

Ans. A hormone
://
tp

150. The smog is essentially caused by the presence of


(1) O2 and O3 (2) O3 and N2
ht

(3) Oxides of sulphur and nitrogen (4) O2 and N2

Ans. Oxides of sulphur and nitrogen

http://notesforfree.com/
http://notesforfree.com/
AIEEE-2004-12

SOLUTIONS (AIEEE)
76. (3) 77. (4) 78. (3) 79. (1)

80. (3) 81. (1) 82. (2) 83. (4)

84. (3) 85. (4) 86. (3) 87. (2)

88. (3) 89. (2) 90. (1) 91. (2)

92. (3) 93. (2) 94. (4) 95. (3)

96. (2) 97. (4) 98. (2) 99. (4)

100. (1) 101. (4) 102. (1) 103. (2)

104. (1) 105. (4) 106. (2) 107. (3)

108. (2) 109. (3) 110. (2) 111. (1)

/
m
112. (3) 113. (1) 114. (2) 115. (3)

co
116. (3) 117. (1) 118. (4) 119. (4)

120. (1) 121. (4) 122.


e e. (3) 123. (1)
rfr

124. (2) 125. (1) 126. (3) 127. (2)


fo

128. (1) 129. (4) 130. (3) 131. (1)


es

132. (3) 133. (2) 134. (2) 135. (3)


t
no

136. (4) 137. (1) 138. (3) 139. (2)


://

140. (2) 141. (1) 142. (4) 143. (1)


tp
ht

144. (3) 145. (2) 146. (4) 147. (3)

148. (1) 149. (4) 150. (3)

SOLUTION
76. 4f → n =4
l=3
m = -l to + l
- 3 to +3

77. 24 → 1s22s22p63s23p64s13d5


l = 1 → p → 12
l = 2 → d → 5

78. Li+ F- O-2 B+3

http://notesforfree.com/
http://notesforfree.com/
AIEEE-2004-13
e 2 10 10 2
p 3 9 8 5

1 1 1
79. =R 2 − 2 
λ  n1 n2 
 1
= 1.097 ×107  
 1
1
λ= × 10 −7 m
1.097

80. H2S → sp3


NH3 → sp3
BF3 → sp2
SiH4 → sp3

82. Al, Si, P, S acidity of oxides increases

83. Bond order of NO = 2.5


Bond order of NO+ = 3

/
m
Higher the bond order shorter is the bond length

co
84. O-1(g) + e → O-2(g)
Due to the electronic repulsion, amount of the energy is needed to add electron

86. Total no of valence electrons


e e.
rfr
= 3+7×4+1 = 32
Total No of hybrid orbital = 4
fo

Q Hybridisation = sp3
es

E1 T1
88. =
t
no

E2 T2
E1 293
=
://

E2 313
tp

313
Q factor =
ht

293

89. sp3d2 hybridisation confirms to octahedral or square bipyramidal configuration


∴ all the bond angles are 90° in the structure

90. Von’t Hoffs factor (i) for Na2SO4 is maximum i.e. 3( maximum no of particles)
Na2SO4 → 2Na+ + SO4—

92. In Vander Waals equation ‘b’ is the excluded volume i.e. the volume occupied by the
molecules
0.0001× 1000
93. Q 6.02×10+20 molecules of urea is present in = = 0.01M
100

95. No. of gm equivalents of phosphorous acid


= No. of gm equivalents of KOH
20×0.1×2 (n = factor) = 0.1 ×V
= 0.1 ×V

http://notesforfree.com/
http://notesforfree.com/
AIEEE-2004-14
4
V= = 40ml
0.1

96. Q the molecular weight of C2H5OH & CH3OCH3 are same so in its vapour phase at same
temperature & pressure the densities will be same

97. Benzene in methanol breaks the H – bonding of the alcohol making its boiling point decrease
& there by its vapour pressure increases leading two +ve deviation.

100. Work done = -P(∆V)


= - 1×105 [10-2 – 10-3] = - 900 J

102. t1/2 = 15 minutes


∴ No. of half lives s =2
(∴ for change of 0.1 to 0.025)
is 30 minutes

103. Applying law of mass action

104. Kp = Kc (RT)∆n

/
m
105. As per property of equilibria reverse the equation & divide it by 2

co
107. o
Ecell = ERHS − ELHS
o

= (0.77) – (-0.14)
= 0.91 V
e e.
rfr

108. Ksp = 108s5


fo

1×44×s1+4 = 256 s5 = Ksp


es

nEo 1× 0.591
t

109. ∴ log Keq = =


no

0.0591 0.0591
⇒ Keq = 1010
://

110. C + O2 → CO2 ∆H = -393.5 kJ


tp

2CO + ½ O2 → 2CO2 ∆H = -283 kJ


2C + O2 → 2CO ∆H = -110 kJ
ht

111. Λ oNaCl = λNa


o
+ λ Cl
o
= 126 … (1)
Λ oKBr = λKo + + λBr
o
− = 152 … (2)
Λ oKCl = λKo + + λ oCl− = 150 … (3)
Λ NaBr =
o
λNa
o
+ λBr
o

Λ oNaBr = 126 + 152 − 150 = 128

115. Mg3N2 +6H2O → 3Mg(OH)2 + 2NH3

117. Q Be & Al have diagonal relationship & so possess similar properties but Be cannot form
polymeric hydrides

120. Q oxidation of potential of Cr is least & so it changes easily from +2 to +3 state

121. 2 CuSO4 + 4KI (excess) → 2K2SO4 + Cu2 I2 + I2↑

http://notesforfree.com/
http://notesforfree.com/
AIEEE-2004-15
Na2S2O3 + I2 → Na2S4O6 + 2NaI

124. sp3d2 ∴ outer orbital octahedral complex

125. Chlorophyll contains magnesium instead of calcium

126. Oxidation potential of Ce(IV) in aqueous solution is supposed to be –ve i.e. -0.784 V at
25°C

200
130. 26 =
a−x
(a – x) = 3.125 gm

135. It is having only sp3 & sp hybridized carbon atom

136. CH3

H Cl
plane symmetry
H Cl

/
CH3

m
co
137. Rate of reaction will be fastest when Z is Cl because it is a weakest base

138.
H3C
H e e.
rfr

C 2H5
fo

146. Benzaldehyde does not contain α - hydrogen. Hence goes for cannizarro’s reaction forming
es

alcohol and acid


t
no

CH3
147.
://

H3C OH CH3
tp

Tertiory alcohols will undergo more easily dehydration than secondary & primary
ht

H H H
148.
H Cl No. chiral centre
Hence not chiral compound
H H H

149. Insulin

http://notesforfree.com/
http://notesforfree.com/

AIEEE − 2004 (MATHEMATICS)

Important Instructions:

1
i) The test is of 1 hours duration.
2

ii) The test consists of 75 questions.

iii) The maximum marks are 225.

iv) For each correct answer you will get 3 marks and for a wrong answer you will get -1 mark.

1. Let R = {(1, 3), (4, 2), (2, 4), (2, 3), (3, 1)} be a relation on the set A = {1, 2, 3, 4}. The
relation R is

/
m
(1) a function (2) reflexive
(3) not symmetric (4) transitive

co
7− x
2. The range of the function f(x) =
(1) {1, 2, 3}
Px −3 is e e.
(2) {1, 2, 3, 4, 5}
rfr
(3) {1, 2, 3, 4} (4) {1, 2, 3, 4, 5, 6}
fo

3. Let z, w be complex numbers such that z + iw = 0 and arg zw = π. Then arg z equals
es

π 5π
(1) (2)
4 4
t
no

3π π
(3) (4)
4 2
://
tp

x y
 + 
ht

1
4. If z = x – i y and z 3 = p + iq , then p q is equal to
(p 2
+ q2 )
(1) 1 (2) -2
(3) 2 (4) -1

2
5. If z2 − 1 = z + 1, then z lies on
(1) the real axis (2) an ellipse
(3) a circle (4) the imaginary axis.

 0 0 −1
 
6. Let A =  0 −1 0  . The only correct statement about the matrix A is
 −1 0 0 
 
(1) A is a zero matrix (2) A 2 = I
(3) A −1 does not exist (4) A = ( −1) I , where I is a unit matrix

http://notesforfree.com/
http://notesforfree.com/
AIEEE-PAPERS--2

 1 −1 1   4 2 2
   
7. Let A =  2 1 −3  (10 ) B =  −5 0 α  . If B is the inverse of matrix A, then α is
1 1 1   1 −2 3 
   
(1) -2 (2) 5
(3) 2 (4) -1

8. If a1, a2 , a3 , ....,an , .... are in G.P., then the value of the determinant
logan logan+1 logan+ 2
logan+ 3 logan+ 4 logan+5 , is
logan+ 6 logan+7 logan+8
(1) 0 (2) -2
(3) 2 (4) 1

9. Let two numbers have arithmetic mean 9 and geometric mean 4. Then these numbers are
the roots of the quadratic equation
(1) x 2 + 18x + 16 = 0 (2) x 2 − 18x − 16 = 0

/
m
(3) x 2 + 18x − 16 = 0 (4) x 2 − 18x + 16 = 0

co
10. If (1 – p) is a root of quadratic equation x 2 + px + (1 − p ) = 0 , then its roots are
(1) 0, 1
(3) 0, -1
e e.
(2) -1, 2
(4) -1, 1
rfr

Let S(K) = 1 + 3 + 5 + ... + ( 2K − 1) = 3 + K 2 . Then which of the following is true?


fo

11.
(1) S(1) is correct
es

(2) Principle of mathematical induction can be used to prove the formula


t

(3) S(K) ⇒ S(K + 1)


no

(4) S(K) ⇒ S(K + 1)


://

12. How many ways are there to arrange the letters in the word GARDEN with the vowels in
tp

alphabetical order?
ht

(1) 120 (2) 480


(3) 360 (4) 240

13. The number of ways of distributing 8 identical balls in 3 distinct boxes so that none of the
boxes is empty is
(1) 5 (2) 8 C3
(3) 38 (4) 21

14. If one root of the equation x 2 + px + 12 = 0 is 4, while the equation x 2 + px + q = 0 has equal
roots, then the value of ‘q’ is
49
(1) (2) 4
4
(3) 3 (4) 12

2
http://notesforfree.com/
http://notesforfree.com/
AIEEE-PAPERS--3

The coefficient of the middle term in the binomial expansion in powers of x of (1 + αx ) and
4
15.
of (1 − αx ) is the same if α equals
6

5 3
(1) − (2)
3 5
−3 10
(3) (4)
10 3

The coefficient of xn in expansion of (1 + x )(1 − x ) is


n
16.
(2) ( −1) (1 − n )
n
(1) (n – 1)
(3) ( −1) (n − 1) (4) ( −1)
n −1 2 n −1
n

n n
1 r t
17. If Sn = ∑
r =0
n
Cr
and tn = ∑
r =0
n
Cr
, then n is equal to
Sn
1 1

/
(1) n (2) n −1

m
2 2

co
2n − 1
(3) n – 1 (4)
2

18.
e e.
Let Tr be the rth term of an A.P. whose first term is a and common difference is d. If for
rfr
1 1
some positive integers m, n, m ≠ n, Tm = and Tn = , then a – d equals
fo

n m
(1) 0 (2) 1
es

1 1 1
(3) (4) +
t

mn m n
no

n ( n + 1)
2
://

2 2 2 2 2 2
19. The sum of the first n terms of the series 1 + 2 ⋅ 2 + 3 + 2 ⋅ 4 + 5 + 2 ⋅ 6 + ... is
tp

2
when n is even. When n is odd the sum is
ht

3n ( n + 1) n2 ( n + 1)
(1) (2)
2 2
n ( n + 1)  n ( n + 1) 
2 2

(3) (4)  
4  2 

1 1 1
20. The sum of series + + + ... is
2! 4! 6!

(1)
(e 2
−1 ) (2)
( e − 1)
2

2 2e

(3)
( 2
e −1 ) (4)
( e −22
)
2e e

3
http://notesforfree.com/
http://notesforfree.com/
AIEEE-PAPERS--4

21 27
21. Let α, β be such that π < α - β < 3π. If sinα + sinβ = − and cosα + cosβ = − , then the
65 65
α −β
value of cos is
2
3 3
(1) − (2)
130 130
6 6
(3) (4) −
65 65

22. If u = a2 cos2 θ + b2 sin2 θ + a2 sin2 θ + b2 cos2 θ , then the difference between the
maximum and minimum values of u2 is given by
(
(1) 2 a2 + b2 ) (2) 2 a2 + b2

(3) ( a + b ) (4) ( a − b )
2 2

/
23. The sides of a triangle are sinα, cosα and 1 + sin α cos α for some 0 < α < . Then the

m
2

co
greatest angle of the triangle is
(1) 60o (2) 90o
(3) 120o
e e.
(4) 150o
rfr
24. A person standing on the bank of a river observes that the angle of elevation of the top of a
tree on the opposite bank of the river is 60o and when he retires 40 meter away from the
fo

tree the angle of elevation becomes 30o . The breadth of the river is
es

(1) 20 m (2) 30 m
t

(3) 40 m (4) 60 m
no

25. If f : R → S, defined by f(x) = sin x − 3 cos x + 1 , is onto, then the interval of S is


://

(1) [0, 3] (2) [-1, 1]


tp

(3) [0, 1] (4) [-1, 3]


ht

26. The graph of the function y = f(x) is symmetrical about the line x = 2, then
(1) f(x + 2)= f(x – 2) (2) f(2 + x) = f(2 – x)
(3) f(x) = f(-x) (4) f(x) = - f(-x)

sin−1 ( x − 3 )
27. The domain of the function f(x) = is
9 − x2
(1) [2, 3] (2) [2, 3)
(3) [1, 2] (4) [1, 2)

2x
 a b 
28. If lim  1 + + 2  = e2 , then the values of a and b, are
x →∞  x x 
(1) a ∈ R, b ∈ R (2) a = 1, b ∈ R
(3) a ∈ R, b = 2 (4) a = 1 and b = 2

4
http://notesforfree.com/
http://notesforfree.com/
AIEEE-PAPERS--5

1 − tan x π  π  π π
29. Let f(x) = , x ≠ , x ∈ 0,  . If f(x) is continuous in 0, 2  , then f  4  is
4x − π 4  2    
1
(1) 1 (2)
2
1
(3) − (4) -1
2

y + ...to ∞ dy
30. If x = e y + e , x > 0, then is
dx
x 1
(1) (2)
1+ x x
1− x 1+ x
(3) (4)
x x

31. A point on the parabola y 2 = 18x at which the ordinate increases at twice the rate of the
abscissa is

/
(1) (2, 4) (2) (2, -4)

m
 −9 9  9 9

co
(3)  ,  (4)  , 
 8 2 8 2

32.
e e.
A function y = f(x) has a second order derivative f″(x) = 6(x – 1). If its graph passes through
the point (2, 1) and at that point the tangent to the graph is y = 3x – 5, then the function is
rfr

(1) ( x − 1) (2) ( x − 1)
2 3
fo

(3) ( x + 1) (4) ( x + 1)
3 2
t es

The normal to the curve x = a(1 + cosθ), y = asinθ at ‘θ’ always passes through the fixed
no

33.
point
://

(1) (a, 0) (2) (0, a)


(3) (0, 0) (4) (a, a)
tp
ht

34. If 2a + 3b + 6c =0, then at least one root of the equation ax 2 + bx + c = 0 lies in the interval
(1) (0, 1) (2) (1, 2)
(3) (2, 3) (4) (1, 3)

n r
1 n
35. lim
n→∞

r =1 n
e is

(1) e (2) e – 1
(3) 1 – e (4) e + 1

sin x
36. If ∫ sin(x − α) dx = Ax + B logsin(x − α) + C , then value of (A, B) is
(1) (sinα, cosα) (2) (cosα, sinα)
(3) (- sinα, cosα) (4) (- cosα, sinα)

dx
37. ∫ cos x − sin x is equal to
5
http://notesforfree.com/
http://notesforfree.com/
AIEEE-PAPERS--6

1  x π 1 x
(1) log tan  −  + C (2) log cot   + C
2 2 8 2 2
1  x 3π  1  x 3π 
(3) log tan  −  +C (4) log tan  +  +C
2 2 8  2 2 8 

∫ | 1− x
2
38. The value of |dx is
−2
28 14
(1) (2)
3 3
7 1
(3) (4)
3 3

π/2
(sin x + cos x)2
39. The value of I = ∫
0 1 + sin 2x
dx is

(1) 0 (2) 1

/
m
(3) 2 (4) 3

co
π π/2
40. If ∫
0
xf(sin x) dx = A ∫
0
f(sin x) dx, then A is e e.
(1) 0 (2) π
rfr

π
(3) (4) 2π
fo

4
es

f (a) f (a)
ex I2
t

41. If f(x) = , I1 = ∫ xg{x(1 − x)}dx and I2 = ∫ g{x(1 − x)}dx then the value of is
no

1 + ex f ( − a) f ( − a)
I1
://

(1) 2 (2) –3
(3) –1 (4) 1
tp
ht

42. The area of the region bounded by the curves y = |x – 2|, x = 1, x = 3 and the x-axis is
(1) 1 (2) 2
(3) 3 (4) 4

43. The differential equation for the family of curves x 2 + y 2 − 2ay = 0 , where a is an arbitrary
constant is
(1) 2(x 2 − y 2 )y ′ = xy (2) 2(x 2 + y 2 )y ′ = xy
(3) (x 2 − y 2 )y ′ = 2xy (4) (x 2 + y 2 )y ′ = 2xy

44. The solution of the differential equation y dx + (x + x2y) dy = 0 is


1 1
(1) − =C (2) − + log y = C
xy xy
1
(3) + log y = C (4) log y = Cx
xy

6
http://notesforfree.com/
http://notesforfree.com/
AIEEE-PAPERS--7

45. Let A (2, –3) and B(–2, 1) be vertices of a triangle ABC. If the centroid of this triangle moves
on the line 2x + 3y = 1, then the locus of the vertex C is the line
(1) 2x + 3y = 9 (2) 2x – 3y = 7
(3) 3x + 2y = 5 (4) 3x – 2y = 3

46. The equation of the straight line passing through the point (4, 3) and making intercepts on
the co-ordinate axes whose sum is –1 is
x y x y x y x y
(1) + = −1and + = −1 (2) − = −1 and + = −1
2 3 −2 1 2 3 −2 1
x y x y x y x y
(3) + = 1and + = 1 (4) − = 1 and + =1
2 3 2 1 2 3 −2 1

47. If the sum of the slopes of the lines given by x 2 − 2cxy − 7y 2 = 0 is four times their product,
then c has the value
(1) 1 (2) –1
(3) 2 (4) –2

/
48. If one of the lines given by 6x 2 − xy + 4cy 2 = 0 is 3x + 4y = 0, then c equals

m
(1) 1 (2) –1

co
(3) 3 (4) –3

49.
e e.
If a circle passes through the point (a, b) and cuts the circle x 2 + y 2 = 4 orthogonally, then
the locus of its centre is
rfr

(1) 2ax + 2by + (a2 + b2 + 4) = 0 (2) 2ax + 2by − (a2 + b2 + 4) = 0


fo

(3) 2ax − 2by + (a2 + b2 + 4) = 0 (4) 2ax − 2by − (a2 + b2 + 4) = 0


es

50. A variable circle passes through the fixed point A (p, q) and touches x-axis. The locus of the
t
no

other end of the diameter through A is


(1) (x − p)2 = 4qy (2) (x − q)2 = 4py
://

(3) (y − p)2 = 4qx (4) (y − q)2 = 4px


tp
ht

51. If the lines 2x + 3y + 1 = 0 and 3x – y – 4 = 0 lie along diameters of a circle of circumference


10π, then the equation of the circle is
(1) x 2 + y 2 − 2x + 2y − 23 = 0 (2) x 2 + y 2 − 2x − 2y − 23 = 0
(3) x 2 + y 2 + 2x + 2y − 23 = 0 (4) x 2 + y 2 + 2x − 2y − 23 = 0

52. The intercept on the line y = x by the circle x 2 + y 2 − 2x = 0 is AB. Equation of the circle on
AB as a diameter is
(1) x 2 + y 2 − x − y = 0 (2) x 2 + y 2 − x + y = 0
(3) x 2 + y 2 + x + y = 0 (4) x 2 + y 2 + x − y = 0

53. If a ≠ 0 and the line 2bx + 3cy + 4d = 0 passes through the points of intersection of the
parabolas y 2 = 4ax and x 2 = 4ay , then
(1) d2 + (2b + 3c)2 = 0 (2) d2 + (3b + 2c)2 = 0
(3) d2 + (2b − 3c)2 = 0 (4) d2 + (3b − 2c)2 = 0

7
http://notesforfree.com/
http://notesforfree.com/
AIEEE-PAPERS--8

1
54. The eccentricity of an ellipse, with its centre at the origin, is . If one of the directrices is x =
2
4, then the equation of the ellipse is
(1) 3x 2 + 4y 2 = 1 (2) 3x 2 + 4y 2 = 12
(3) 4x 2 + 3y 2 = 12 (4) 4x 2 + 3y 2 = 1

55. A line makes the same angle θ, with each of the x and z axis. If the angle β, which it makes
with y-axis, is such that sin2 β = 3 sin2 θ , then cos2 θ equals
2 1
(1) (2)
3 5
3 2
(3) (4)
5 5

56. Distance between two parallel planes 2x + y + 2z = 8 and 4x + 2y + 4z + 5 = 0 is


3 5
(1) (2)
2 2

/
m
7 9
(3) (4)
2 2

co
57.
e.
A line with direction cosines proportional to 2, 1, 2 meets each of the lines x = y + a = z and
x + a = 2y = 2z. The co-ordinates of each of the point of intersection are given by
e
(1) (3a, 3a, 3a), (a, a, a) (2) (3a, 2a, 3a), (a, a, a)
rfr

(3) (3a, 2a, 3a), (a, a, 2a) (4) (2a, 3a, 3a), (2a, a, a)
fo

t
es

58. If the straight lines x = 1 + s, y = –3 – λs, z = 1 + λs and x = , y = 1 + t, z = 2 – t with


2
t

parameters s and t respectively, are co-planar then λ equals


no

(1) –2 (2) –1
1
://

(3) – (4) 0
2
tp
ht

59. The intersection of the spheres x 2 + y 2 + z2 + 7x − 2y − z = 13 and


2 2 2
x + y + z − 3x + 3y + 4z = 8 is the same as the intersection of one of the sphere and the
plane
(1) x – y – z = 1 (2) x – 2y – z = 1
(3) x – y – 2z = 1 (4) 2x – y – z = 1
r r r
60. Let a, b and c be three non-zero vectors such that no two of these are collinear. If the
r r r r r r
vector a + 2b is collinear with c and b + 3c is collinear with a (λ being some non-zero
r r r
scalar) then a + 2b + 6c equals
r r
(1) λa (2) λb
r
(3) λc (4) 0

61. A particle is acted upon by constant forces 4iˆ + ˆj − 3kˆ and 3iˆ + ˆj − kˆ which displace it from a
point ˆi + 2ˆj + 3kˆ to the point 5iˆ + 4ˆj + kˆ . The work done in standard units by the forces is
given by

8
http://notesforfree.com/
http://notesforfree.com/
AIEEE-PAPERS--9

(1) 40 (2) 30
(3) 25 (4) 15

62. If a, b, c are non-coplanar vectors and λ is a real number, then the vectors
a + 2b + 3c, λ b + 4c and (2λ − 1)c are non-coplanar for
(1) all values of λ (2) all except one value of λ
(3) all except two values of λ (4) no value of λ

63. Let u, v, w be such that u = 1, v = 2, w = 3 . If the projection v along u is equal to that of


w along u and v, w are perpendicular to each other then u − v + w equals
(1) 2 (2) 7
(3) 14 (4) 14

1
64. Let a, b and c be non-zero vectors such that (a × b) × c = b c a . If θ is the acute angle
3
between the vectors b and c , then sin θ equals

/
m
1 2
(1) (2)

co
3 3
2 2 2
(3)
3
(4)
e
3 e.
rfr

65. Consider the following statements:


fo

(a) Mode can be computed from histogram


(b) Median is not independent of change of scale
es

(c) Variance is independent of change of origin and scale.


t

Which of these is/are correct?


no

(1) only (a) (2) only (b)


(3) only (a) and (b) (4) (a), (b) and (c)
://
tp

66. In a series of 2n observations, half of them equal a and remaining half equal –a. If the
ht

standard deviation of the observations is 2, then |a| equals


1
(1) (2) 2
n
2
(3) 2 (4)
n

4 3
67. The probability that A speaks truth is, while this probability for B is . The probability that
5 4
they contradict each other when asked to speak on a fact is
3 1
(1) (2)
20 5
7 4
(3) (4)
20 5

68. A random variable X has the probability distribution:


X: 1 2 3 4 5 6 7 8
p(X): 0.15 0.23 0.12 0.10 0.20 0.08 0.07 0.05
9
http://notesforfree.com/
http://notesforfree.com/
AIEEE-PAPERS--10

For the events E = {X is a prime number} and F = {X < 4}, the probability P (E ∪ F) is
(1) 0.87 (2) 0.77
(3) 0.35 (4) 0.50

69. The mean and the variance of a binomial distribution are 4 and 2 respectively. Then the
probability of 2 successes is
37 219
(1) (2)
256 256
128 28
(3) (4)
256 256

70. With two forces acting at a point, the maximum effect is obtained when their resultant is 4N.
If they act at right angles, then their resultant is 3N. Then the forces are
(1) (2 + 2)N and (2 − 2)N (2) (2 + 3)N and (2 − 3 )N
 1   1   1   1 
(3)  2 + 2  N and  2 − 2 N (4)  2 + 3  N and  2 − 3 N
 2   2   2   2 

/
m
71. In a right angle ∆ABC, ∠A = 90° and sides a, b, c are respectively, 5 cm, 4 cm and 3 cm. If a

co
r
force F has moments 0, 9 and 16 in N cm. units respectively about vertices A, B and C,
r
then magnitude of F is
(1) 3 (2) 4
e e.
rfr
(3) 5 (4) 9
fo

r r r
72. Three forces P, Q and R acting along IA, IB and IC, where I is the incentre of a ∆ABC, are
es

r r r
in equilibrium. Then P : Q : R is
t

A B C A B C
no

(1) cos : cos : cos (2) sin : sin : sin


2 2 2 2 2 2
A B C A B C
://

(3) sec : sec : sec (4) co sec : co sec : co sec


2 2 2 2 2 2
tp
ht

73. A particle moves towards east from a point A to a point B at the rate of 4 km/h and then
towards north from B to C at the rate of 5 km/h. If AB = 12 km and BC = 5 km, then its
average speed for its journey from A to C and resultant average velocity direct from A to C
are respectively
17 13 13 17
(1) km/h and km/h (2) km/h and km/h
4 4 4 4
17 13 13 17
(3) km/h and km/h (4) km/h and km/h
9 9 9 9
1
74. A velocity m/s is resolved into two components along OA and OB making angles 30° and
4
45° respectively with the given velocity. Then the component along OB is
1 1
(1) m/s (2) ( 3 − 1) m/s
8 4
1 1
(3) m/s (4) ( 6 − 2) m/s
4 8

10
http://notesforfree.com/
http://notesforfree.com/
AIEEE-PAPERS--11

75. If t1 and t2 are the times of flight of two particles having the same initial velocity u and range
R on the horizontal, then t12 + t 22 is equal to
u2 4u2
(1) (2)
g g2
u2
(3) (4) 1
2g

/
m
co
e e.
rfr
fo
t es
no
://
tp
ht

11
http://notesforfree.com/
http://notesforfree.com/
AIEEE-PAPERS--12

FIITJEE AIEEE − 2004 (MATHEMATICS)


ANSWERS

1. 3 16. 2 31. 4 46. 4 61. 1


2. 1 17. 1 32. 2 47. 3 62. 3
3. 3 18. 1 33. 1 48. 4 63. 3
4. 2 19. 2 34. 1 49. 2 64. 4
5. 4 20. 2 35. 2 50. 1 65. 3
6. 2 21. 1 36. 2 51. 1 66. 3
7. 2 22. 4 37. 4 52. 1 67. 3
8. 1 23. 3 38. 1 53. 1 68. 2
9. 4 24. 1 39. 3 54. 2 69. 4
10. 3 25. 4 40. 2 55. 3 70. 3
11. 4 26. 2 41. 1 56. 3 71. 3

/
12. 3 27. 2 42. 1 57. 2 72. 1

m
13. 4 28. 2 43. 3 58. 1 73. 1

co
14. 1 29. 3 44. 2 59. 4 74. 4
15. 3 30. 3 45. 1 60. 4 75. 2
e e.
rfr
fo
est
no
://
tp
ht

12
http://notesforfree.com/
http://notesforfree.com/
AIEEE-PAPERS--13

FIITJEE AIEEE − 2004 (MATHEMATICS)


SOLUTIONS
1. (2, 3) ∈ R but (3, 2) ∉ R.
Hence R is not symmetric.

2. f(x) = 7 − x Px −3
7−x ≥0 ⇒ x ≤7
x−3 ≥ 0 ⇒ x ≥ 3,
and 7 − x ≥ x − 3 ⇒ x≤5
⇒ 3 ≤ x ≤ 5 ⇒ x = 3, 4, 5 ⇒ Range is {1, 2, 3}.

z  z 3π
3. Here ω = ⇒ arg  z.  = π ⇒ 2 arg(z) – arg(i) = π ⇒ arg(z) = .
i  i 4
4. ( ) (
z = ( p + iq ) = p p2 − 3q2 − iq q2 − 3p2
3
)

/
m
x y
+

co
x y p q
⇒ = p2 − 3q2 & = q2 − 3p2 ⇒ = −2 .
p q (p 2
+ q2
e )
e.
( ) ( )( )
2 2 4 2
2
z2 − 1 = z + 1 ⇒ z2 − 1 z 2 − 1 = z + 2 z + 1
rfr
5.
fo

⇒ z2 + z 2 + 2zz = 0 ⇒ z + z = 0
⇒ R (z) = 0 ⇒ z lies on the imaginary axis.
t es

1 0 0
no

6. A.A = 0 1 0  = I .
://

0 0 1
tp

7. AB = I ⇒ A(10 B) = 10 I
ht

 1 −1 1   4 2 2  10 0 5 − α   1 0 0
⇒  2 1 −3   −5 0 α  =  0 10 α − 5  = 10 0 1 0  if α = 5 .
     

 1 1 1   1 −2 3   0 0 5 + α  0 0 1
logan logan+1 logan+ 2
8. logan+ 3 logan+ 4 logan+5
logan+ 6 logan+7 logan+8
C3 → C3 – C2, C2 → C3 – C1
logan logr logr
= logan+ 3 logr logr = 0 (where r is a common ratio).
logan+ 6 logr logr

9. Let numbers be a, b ⇒ a + b = 18, ab = 4 ⇒ ab = 16 , a and b are roots of the


equation

13
http://notesforfree.com/
http://notesforfree.com/
AIEEE-PAPERS--14

⇒ x 2 − 18x + 16 = 0 .

10. (3)
(1 − p ) + p (1 − p ) + (1 − p ) = 0 (since (1 – p) is a root of the equation x2 + px + (1 – p) = 0)
2

⇒ (1 − p )(1 − p + p + 1) = 0
⇒ 2 (1 − p ) = 0 ⇒ (1 – p) = 0 ⇒ p = 1
sum of root is α + β = −p and product αβ = 1 − p = 0 (where β = 1 – p = 0)
⇒ α + 0 = −1 ⇒ α = −1 ⇒ Roots are 0, –1

11. S ( k ) = 1 + 3 + 5 + ........ + ( 2k − 1) = 3 + k 2
S(k + 1)=1 + 3 + 5 +............. + (2k – 1) + (2k + 1)
( )
= 3 + k 2 + 2k + 1 = k 2 + 2k + 4 [from S(k) = 3 + k 2 ]
= 3 + (k + 2k + 1) = 3 + (k + 1)2 = S (k + 1).
2

Although S (k) in itself is not true but it considered true will always imply towards S (k + 1).

/
m
12. Since in half the arrangement A will be before E and other half E will be before A.
6!

co
Hence total number of ways = = 360.
2

13. Number of balls = 8


e e.
number of boxes = 3
rfr

Hence number of ways = 7C2 = 21.


fo

14. Since 4 is one of the root of x2 + px + 12 = 0 ⇒ 16 + 4p + 12 = 0 ⇒ p = –7


es

and equation x2 + px + q = 0 has equal roots


t

49
no

⇒ D = 49 – 4q = 0 ⇒ q = .
4
://

Coefficient of Middle term in (1 + αx ) = t 3 = 4 C2 ⋅ α 2


4
15.
tp

Coefficient of Middle term in (1 − αx ) = t 4 = 6 C3 ( −α )


6 3
ht

4 −3
C 2 α 2 = −6 C3 .α3 ⇒ − 6 = 20α ⇒ α=
10
16. Coefficient of xn in (1 + x)(1 – x)n = (1 + x)(nC0 – nC1x + …….. + (–1)n –1 nCn – 1 xn – 1 + (–1)n
n
Cn xn)
= (–1)n nCn + (–1)n –1 nCn – 1 = ( −1)n (1 − n ) .

n n n
n−r n−r
17. t= ∑ n
r
Cr
= ∑ n
Cn−r
= ∑ n
Cr
(Q n
Cr =n Cn−r )
r =0 r =0 r =0
n n
r +n−r n n n 1 n tn n
2tn = ∑ r =0
n
Cr
= ∑
r =0
n
Cr
⇒ t n = ∑
n
= Sn ⇒
2 r =0 Cr 2
=
Sn 2

1
18. Tm = = a + ( m − 1) d .....(1)
n
1
and Tn = = a + ( n − 1) d .....(2)
m
14
http://notesforfree.com/
http://notesforfree.com/
AIEEE-PAPERS--15

1 1
from (1) and (2) we get a = , d=
mn mn
Hence a – d = 0

(n − 1) n2 n2 ( n + 1)
19. If n is odd then (n – 1) is even ⇒ sum of odd terms = + n2 = .
2 2

eα + e −α α2 α 4 α6
20. = 1+ + + + ……..
2 2! 4! 6!
eα + e −α α2 α 4 α6
−1= + + + .......
2 2! 4! 6!
put α = 1, we get
( e − 1)
2
1 1 1
= + + + ………..
2e 2! 4! 6!

21 27
sin α + sin β = − and cos α + cos β = −

/
21. .

m
65 65
Squaring and adding, we get

co
1170
2 + 2 cos (α – β) =

α −β 9
(65)2
α −β −3
e e.  π α − β 3π 
⇒ cos2  = ⇒ cos  = Q 2 < 2 < 2  .
rfr
 
 2  130  2  130  
fo
es

22. u = a2 cos2 θ + b2 sin2 θ + a2 sin2 θ + b2 cos2 θ


a2 + b2 a2 − b2 a2 + b2 b2 − a2
t

= + cos 2θ + + cos 2θ
no

2 2 2 2
://

2 2
2  a2 + b2   a 2 − b 2 
2 2 2
⇒ u = a +b +2   −  cos 2θ
tp

 2   2 
ht

min value of u2 = a2 + b2 + 2ab


(
max value of u2 = 2 a2 + b2 )
= (a − b) .
2 2 2
⇒ umax − umin

23. Greatest side is 1 + sin α cos α , by applying cos rule we get greatest angle = 120ο.

h
24. tan30° = h
40 + b
⇒ 3 h = 40 + b …..(1) 30° 60°
tan60° = h/b ⇒ h = 3 b ….(2) 40 b
⇒ b = 20 m

25. −2 ≤ sin x − 3 cos x ≤ 2 ⇒ −1 ≤ sin x − 3 cos x + 1 ≤ 3


⇒ range of f(x) is [–1, 3].
Hence S is [–1, 3].
15
http://notesforfree.com/
http://notesforfree.com/
AIEEE-PAPERS--16

26. If y = f (x) is symmetric about the line x = 2 then f(2 + x) = f(2 – x).

27. 9 − x 2 > 0 and −1 ≤ x − 3 ≤ 1 ⇒ x ∈ [2, 3)

 
 1 
×2x× + 
a b
2x 
 a b   a b  a + b   x x2 
28. lim  1 + + 2  = lim  1 + + 2  x x
2 
 = e2a ⇒ a = 1, b ∈ R
x →∞  x x  x →∞  x x 

1 − tan x 1 − tan x 1
29. f(x) = ⇒ lim =−
4x − π x→
π 4x − π 2
4

y + e y + ..... ∞
30. x = ey+e ⇒ x = ey + x
dy 1 1− x
⇒ lnx – x = y ⇒ = −1= .
dx x x

/
m
9 
31. Any point be  t 2 , 9t  ; differentiating y2 = 18x

co
2 
dy 9 1
⇒ = = = 2 (given) ⇒ t = .
dx y t
1
2
e e.
rfr
9 9
⇒ Point is  , 
8 2
fo
es

32. f″ (x) = 6(x – 1) ⇒ f′ (x) = 3(x – 1)2 + c


and f′ (2) = 3 ⇒ c = 0
t
no

⇒ f (x) = (x – 1)3 + k and f (2) = 1 ⇒ k = 0


⇒ f (x) = (x – 1)3.
://
tp

33. Eliminating θ, we get (x – a)2 + y2 = a2.


Hence normal always pass through (a, 0).
ht

ax 3 bx 2
34. Let f′(x) = ax 2 + bx + c ⇒ f(x) = + + cx + d
3 2
⇒ f(x) =
1
6
( )
2ax 3 + 3bx 2 + 6cx + 6d , Now f(1) = f(0) = d, then according to Rolle’s theorem

⇒ f′(x) = ax 2 + bx + c = 0 has at least one root in (0, 1)

n r 1
1 n
35. lim
n→∞

r =1 n
e = ∫ e dx = (e − 1)
x

36. Put x – α = t
sin(α + t)
⇒ ∫ sin t
dt = sin α cot tdt + cos α dt ∫ ∫
= cos α ( x − α ) + sin α ln sin t + c
A = cos α, B = sin α
16
http://notesforfree.com/
http://notesforfree.com/
AIEEE-PAPERS--17

dx 1 1 1  π 1  x 3π 
37. ∫ cos x − sin x = 2
∫  π
dx =
2
∫ sec  x + 4  dx =
2
log tan  +
2 8 
 +C
cos  x + 
 4

−1 1 3 −1 1 3
x3 x3 x3
∫ (x ) ∫ (1 − x ) dx + ∫ ( )
2 2 2 28
38. − 1 dx + x − 1 dx = −x +x− + −x = .
−2 −1 1
3 −2
3 −1
3 1
3

π π

( sin x + cos x )
2 2 2 π
39. ∫ dx = ∫ ( sin x + cos x ) dx = − cos x + sin x 02 = 2.
( sin x + cos x )
2
0 0

π π π
40. Let I = ∫ xf(sin x)dx = ∫ (π − x)f(sin x)dx = π∫ f(sin x)dx − I
0 0 0
(since f (2a – x) = f (x))

/
π/2

m
⇒I=π ∫ f(sin x)dx ⇒ A = π.

co
0

41. f(-a) + f(a) = 1


f (a) f (a)
e e.  b b

I1 = ∫ xg{x(1 − x)}dx = ∫ (1 − x ) g{x(1 − x)}dx Q f ( x ) dx = f ( a + b − x ) dx 
∫ ∫
rfr
 
f ( − a) f ( − a)  a a 
fo

f (a)

∫ g{x(1 − x)}dx = I2 ⇒ I2 / I1 = 2.
es

2I1 =
f ( − a)
t
no

2 3 y=x–2
42. Area = ∫ (2 − x)dx + ∫ (x − 2)dx = 1.
://

1 2 y=2 – x
tp
ht

1 2 3

43. 2x + 2yy′ - 2ay′ = 0


x + yy ′
a= (eliminating a)
y′
⇒ (x2 – y2)y′ = 2xy.

45. y dx + x dy + x2y dy = 0.
d(xy) 1 1
2 2
+ dy = 0 ⇒ − + log y = C .
x y y xy

45. If C be (h, k) then centroid is (h/3, (k – 2)/3) it lies on 2x + 3y = 1.


⇒ locus is 2x + 3y = 9.

17
http://notesforfree.com/
http://notesforfree.com/
AIEEE-PAPERS--18

x y 4 3
46. + = 1where a + b = -1 and + = 1
a b a b
⇒ a = 2, b = -3 or a = -2, b = 1.
x y x y
Hence − = 1 and + = 1.
2 3 −2 1

2c 1
47. m1 + m2 = − and m1 m2 = −
7 7
m1 + m2 = 4m1m2 (given)
⇒ c = 2.

1 6 3
48. m1 + m2 = , m1m2 = and m1 = − .
4c 4c 4
Hence c = -3.

49. Let the circle be x2 + y2 + 2gx + 2fy + c = 0 ⇒ c = 4 and it passes through (a, b)
⇒ a2 + b2 + 2ga + 2fb + 4 = 0.

/
Hence locus of the centre is 2ax + 2by – (a2 + b2 + 4) = 0.

m
co
50. Let the other end of diameter is (h, k) then equation of circle is
(x – h)(x – p) + (y – k)(y – q) = 0
Put y = 0, since x-axis touches the circle
e
⇒ x2 – (h + p)x + (hp + kq) = 0 ⇒ (h + p)2 = 4(hp + kq)
e. (D = 0)
rfr
⇒ (x – p)2 = 4qy.
fo

51. Intersection of given lines is the centre of the circle i.e. (1, − 1)
es

Circumference = 10π ⇒ radius r = 5


⇒ equation of circle is x2 + y2 − 2x + 2y − 23 = 0.
t
no

52. Points of intersection of line y = x with x2 + y2 − 2x = 0 are (0, 0) and (1, 1)


://

hence equation of circle having end points of diameter (0, 0) and (1, 1) is
tp

x2 + y2 − x − y = 0.
ht

53. Points of intersection of given parabolas are (0, 0) and (4a, 4a)
⇒ equation of line passing through these points is y = x
On comparing this line with the given line 2bx + 3cy + 4d = 0, we get
d = 0 and 2b + 3c = 0 ⇒ (2b + 3c)2 + d2 = 0.

54. Equation of directrix is x = a/e = 4 ⇒ a = 2


b2 = a2 (1 − e2) ⇒ b2 = 3
Hence equation of ellipse is 3x2 + 4y2 = 12.

55. l = cos θ, m = cos θ, n = cos β


cos2 θ + cos2 θ + cos2 β = 1 ⇒ 2 cos2 θ = sin2 β = 3 sin2 θ (given)
cos2 θ = 3/5.

56. Given planes are


2x + y + 2z − 8 = 0, 4x + 2y + 4z + 5 = 0 ⇒ 2x + y + 2z + 5/2 = 0
| d1 − d2 | | −8 − 5 / 2 | 7
Distance between planes = = = .
a2 + b2 + c 2 22 + 12 + 22 2
18
http://notesforfree.com/
http://notesforfree.com/
AIEEE-PAPERS--19

x y+a z
57. Any point on the line = = = t1 (say) is (t1, t1 – a, t1) and any point on the line
1 1 1
x+a y z
= = = t 2 ( say ) is (2t2 – a, t2, t2).
2 1 1
Now direction cosine of the lines intersecting the above lines is proportional to
(2t2 – a – t1, t2 – t1 + a, t2 – t1).
Hence 2t2 – a – t1 = 2k , t2 – t1 + a = k and t2 – t1 = 2k
On solving these, we get t1 = 3a , t2 = a.
Hence points are (3a, 2a, 3a) and (a, a, a).

x −1 y + 3 z −1 x y −1 z − 2
58. Given lines = = = s and = = = t are coplanar then plan
1 −λ λ 1/ 2 1 −1
passing through these lines has normal perpendicular to these lines
a
⇒ a - bλ + cλ = 0 and + b − c = 0 (where a, b, c are direction ratios of the normal to
2
the plan)

/
On solving, we get λ = -2.

m
co
59. Required plane is S1 – S2 = 0
where S1 = x2 + y2 + z2 + 7x – 2y – z – 13 = 0 and
S2 = x2 + y2 + z2 – 3x + 3y + 4z – 8 = 0
⇒ 2x – y – z = 1.
e e.
rfr

r
( a + 2br ) = t c
r r
60. ….(1)
fo

1
r r
es

and b + 3c = t 2 a ….(2)
r r
(1) – 2×(2) ⇒ a (1 + 2t 2 ) + c ( − t1 − 6 ) = 0 ⇒ 1+ 2t2 = 0 ⇒ t2 = -1/2 & t1 = -6.
t
no

r r
Since a and c are non-collinear.
r r r r
://

Putting the value of t1 and t2 in (1) and (2), we get a + 2b + 6c = 0 .


tp

r r r r r r
61. Work done by the forces F1 and F2 is (F1 + F2 ) ⋅ d , where d is displacement
ht

r r
According to question F1 + F2 = (4iˆ + ˆj − 3k) ˆ + (3iˆ + ˆj − k)
ˆ = 7iˆ + 2ˆj − 4kˆ
r r r r
and d = (5iˆ + 4ˆj + k)
ˆ − (iˆ + 2ˆj + 3k)
ˆ = 4iˆ + 2ˆj − 2kˆ . Hence (F + F ) ⋅ d is 40.
1 2

1 2 3
63. Condition for given three vectors to be coplanar is 0 λ 4 = 0 ⇒ λ = 0, 1/2.
0 0 2λ − 1
Hence given vectors will be non coplanar for all real values of λ except 0, 1/2.

v ⋅u w ⋅u
63. Projection of v along u and w along u is and respectively
|u| |u|
v ⋅u w ⋅u
According to question = ⇒ v ⋅ u = w ⋅ u . and v ⋅ w = 0
|u| |u|
| u − v + w |2 =| u |2 + | v |2 + | w |2 −2u ⋅ v + 2u ⋅ w − 2v ⋅ w = 14 ⇒| u − v + w |= 14 .

19
http://notesforfree.com/
http://notesforfree.com/
AIEEE-PAPERS--20

r r r r r r r r r
64. ( a × b ) × c = 31 b c a ⇒ (a ⋅ c ) b − (b ⋅ c ) a = 31 b c a
r r r 1 r r r
( ) 3 
( ) 1
⇒ a ⋅ c b =  b c + b ⋅ c  a ⇒ a ⋅ c = 0 and b c + b ⋅ c = 0
3
( )
1  2 2
⇒ b c  + cos θ  = 0 ⇒ cosθ = –1/3 ⇒ sinθ = .
3  3

65. Mode can be computed from histogram and median is dependent on the scale.
Hence statement (a) and (b) are correct.

66. xi = a for i = 1, 2, .... ,n and xi = −a for i = n, ...., 2n


1 2n
1 2n
 2n
 1
∑(x − x) ∑x ∑x
2 2
S.D. = i ⇒2= i  Since i = 0 ⇒ 2 = ⋅ 2na2 ⇒ a = 2
2n i=1 2n i=1  i=1  2n

67. E1 : event denoting that A speaks truth


E2 : event denoting that B speaks truth

/
m
Probability that both contradicts each other = P E1 ∩ E2 + P E1 ∩ E2( ) ( ) = 54 ⋅ 41 + 51 ⋅ 34 = 207

co
68. P(E ∪ F) = P(E) + P(F) − P (E ∩ F ) = 0.62 + 0.50 – 0.35 = 0.77
e e.
rfr

2 6
 1  1 28
fo

69. Given that n p = 4, n p q = 2 ⇒ q = 1/2 ⇒ p = 1/2 , n = 8 ⇒ p(x = 2) = 8 C2     =


2 2 256
es

 1   1 
t

P + Q = 4, P2 + Q2 = 9 ⇒ P =  2 + 2  N and Q =  2 −
no

70. 2 N.
 2   2 
://

F . 3 sin θ = 9
C
71.
tp

4cosθ
F . 4 cos θ = 16 θ
ht

⇒ F = 5.

B
A θ

3sinθ
F

A
72. By Lami’s theorem
r r r  A  B  C
P : Q : R = sin  90° +  : sin  90° +  : sin  90° + 
 2  2  2 90+C/2 90+B/2
A B C
⇒ cos : cos : cos . 90+A/2
2 2 2
B C

20
http://notesforfree.com/
http://notesforfree.com/
AIEEE-PAPERS--21

C
12
73. Time T1 from A to B = = 3 hrs.
4
13
5
T2 from B to C = = 1 hrs. 5
5
Total time = 4 hrs.
17
Average speed = km/ hr. A 12 B
4
13
Resultant average velocity = km/hr.
4

1
sin 30°
74. Component along OB = 4 =
sin(45° + 30°) 8
1
( 6 − 2 m/s. )
2u sin α 2u sin β
75. t1 = , t2 = where α + β = 900
g g

/
4u2

m
∴ t12 + t 22 = .
g2

co
e e.
rfr
fo
t es
no
://
tp
ht

21
http://notesforfree.com/
SOLUTION TO AIEEE-2005
MATHEMATICS
1. If A2 – A + I = 0, then the inverse of A is
(1) A + I (2) A
(3) A – I (4) I – A
1. (4)
Given A2 – A + I = 0
A–1A2 – A–1A + A–1 – I = A–1⋅0 (Multiplying A–1 on both sides)
⇒ A - I + A-1 = 0 or A–1 = I – A.

2. If the cube roots of unity are 1, ω, ω2 then the roots of the equation
(x – 1)3 + 8 = 0, are
(1) -1 , - 1 + 2ω, - 1 - 2ω2 (2) -1 , -1, - 1
(3) -1 , 1 - 2ω, 1 - 2ω2 (4) -1 , 1 + 2ω, 1 + 2ω2
2. (3)
(x – 1)3 + 8 = 0 ⇒ (x – 1) = (-2) (1)1/3
⇒ x – 1 = -2 or -2ω or -2ω2
or n = -1 or 1 – 2ω or 1 – 2ω2.

3. Let R = {(3, 3), (6, 6), (9, 9), (12, 12), (6, 12), (3, 9), (3, 12), (3, 6)} be a relation on
the set A = {3, 6, 9, 12} be a relation on the set A = {3, 6, 9, 12}. The relation is
(1) reflexive and transitive only (2) reflexive only
(3) an equivalence relation (4) reflexive and symmetric only
3. (1)
Reflexive and transitive only.
e.g. (3, 3), (6, 6), (9, 9), (12, 12) [Reflexive]
(3, 6), (6, 12), (3, 12) [Transitive].

x2 y2
4. Area of the greatest rectangle that can be inscribed in the ellipse + = 1 is
a 2 b2
(1) 2ab (2) ab
a
(3) ab (4)
b
4. (1)
Y
Area of rectangle ABCD = (2acosθ)
(2bsinθ) = 2absin2θ (-acosθ, bsinθ)
⇒ Area of greatest rectangle is equal to B A(acosθ, bsinθ)
2ab
when sin2θ = 1. X

(-acosθ, -bsinθ)C D(acosθ, -bsinθ)

5. The differential equation representing the family of curves y2 = 2c x + c , where c ( )


> 0, is a parameter, is of order and degree as follows:
(1) order 1, degree 2 (2) order 1, degree 1

FIITJEE Ltd. ICES House, 29-A, Kalu Sarai, Sarvapriya Vihar, New Delhi - 110016, Ph : 26515949, 26569493, Fax : 26513942
–2–

(3) order 1, degree 3 (4) order 2, degree 2


5. (3)
y2 = 2c(x + √c) …(i)
2yy′ = 2c⋅1 or yy′ = c …(ii)
⇒ y2 = 2yy′ (x + yy ′ ) [on putting value of c from (ii) in (i)]
On simplifying, we get
(y – 2xy′)2 = 4yy′3 …(iii)
Hence equation (iii) is of order 1 and degree 3.

1 1 2 4 1 
6. lim  2 sec 2 2 + 2 sec 2 2 + .... + 2 sec 2 1 equals
n →∞
n n n n n 
1 1
(1) sec1 (2) cos ec1
2 2
1
(3) tan1 (4) tan1
2
6. (4)
1 1 2 4 3 9 1 
lim sec 2 2 + 2 sec 2 2 + 2 sec 2 2 + .... + sec 2 1 is equal to
n →∞  n2 n n n n n n
 
2 2
r 2 r 1 r 2 r
lim sec = lim ⋅ sec
n→∞ n2 n2 n→∞ n n n2
1
⇒ Given limit is equal to value of integral ∫ x sec
2
x 2 dx
0
1 1
1 1
or
20∫ 2x sec x 2 dx = ∫ sec 2 tdt
20
[put x2 = t]

1 1
( tan t )0 = tan1 .
1
=
2 2

7. ABC is a triangle. Forces P, Q, R acting along IA, IB and IC respectively are in


equilibrium, where I is the incentre of ∆ABC. Then P : Q : R is
A B C
(1) sinA : sin B : sinC (2) sin : sin : sin
2 2 2
A B C
(3) cos : cos : cos (4) cosA : cosB : cosC
2 2 2
7. (3)
Using Lami’s Theorem A
A B C
∴ P : Q : R = cos : cos : cos . P
2 2 2

I R
Q
B C
8. If in a frequently distribution, the mean and median are 21 and 22 respectively, then
its mode is approximately
(1) 22.0 (2) 20.5
(3) 25.5 (4) 24.0
8. (4)
Mode + 2Mean = 3 Median
⇒ Mode = 3 × 22 – 2 × 21= 66 – 42= 24.

FIITJEE Ltd. ICES House, 29-A, Kalu Sarai, Sarvapriya Vihar, New Delhi - 110016, Ph : 26515949, 26569493, Fax : 26513942
–3–

9. Let P be the point (1, 0) and Q a point on the locus y2 = 8x. The locus of mid point of
PQ is
(1) y2 – 4x + 2 = 0 (2) y2 + 4x + 2 = 0
2
(3) x + 4y + 2 = 0 (4) x2 – 4y + 2 = 0
9. (1)
P = (1, 0)
Q = (h, k) such that k2 = 8h
Let (α, β) be the midpoint of PQ
h +1 k+0
α= , β=
2 2
2α - 1 = h 2β = k.
(2β)2 = 8 (2α - 1) ⇒ β2 = 4α - 2
⇒ y2 – 4x + 2 = 0.

10. If C is the mid point of AB and P is any point outside AB, then
(1) PA + PB = 2PC (2) PA + PB = PC
(3) PA + PB + 2PC = 0 (4) PA + PB + PC = 0
10. (1)
PA + AC + CP = 0 P
PB + BC + CP = 0
Adding, we get
PA + PB + AC + BC + 2CP = 0
Since AC = − BC
& CP = −PC
⇒ PA + PB − 2PC = 0 . C
B
A

11. If the coefficients of rth, (r+ 1)th and (r + 2)th terms in the binomial expansion of (1 +
y)m are in A.P., then m and r satisfy the equation
(1) m2 – m(4r – 1) + 4r2 – 2 = 0 (2) m2 – m(4r+1) + 4r2 + 2 = 0
2 2
(3) m – m(4r + 1) + 4r – 2 = 0 (4) m2 – m(4r – 1) + 4r2 + 2 = 0
11. (3)
Given m Cr −1, mCr , mCr +1 are in A.P.
2 mCr = mCr −1 + mCr +1
m
Cr −1 m Cr +1
⇒ 2= m
+ m
Cr Cr
r m−r
= +
m −r +1 r +1
⇒ m2 – m (4r + 1) + 4r2 – 2 = 0.
π P Q
12. In a triangle PQR, ∠R = . If tan   and tan   are the roots of
2 2 2
ax2 + bx + c = 0, a ≠ 0 then
(1) a = b + c (2) c = a + b
(3) b = c (4) b = a + c
12. (2)
P Q
tan   , tan   are the roots of ax2 + bx + c = 0
2
  2
P Q b
tan   + tan   = −
2 2 a

FIITJEE Ltd. ICES House, 29-A, Kalu Sarai, Sarvapriya Vihar, New Delhi - 110016, Ph : 26515949, 26569493, Fax : 26513942
–4–

P Q c
tan   tan   =
2
  2 a
P Q
tan   + tan  
2
   2  = tan  P + Q  = 1
2 2
P Q  
1 − tan   tan  
2 2
b

⇒ a = 1 ⇒ − b = a − c ⇒ −b = a − c
c a a a
1−
a
c = a + b.

13. The system of equations


αx + y + z = α - 1,
x + αy + z = α - 1,
x + y + αz = α - 1
has no solution, if α is
(1) -2 (2) either – 2 or 1
(3) not -2 (4) 1
13. (1)
αx + y + z = α - 1
x + αy + z = α - 1
x + y + zα = α - 1
α 1 1
∆= 1 α 1
1 1 α
= α(α2 – 1) – 1(α - 1) + 1(1 - α)
= α (α - 1) (α + 1) – 1(α - 1) – 1(α - 1)
⇒ (α - 1)[α2 + α - 1 – 1] = 0
⇒ (α - 1)[α2 + α - 2] = 0
[α2 + 2α - α - 2] = 0
(α - 1) [α(α + 2) – 1(α + 2)] = 0
(α - 1) = 0, α + 2 = 0 ⇒ α = –2, 1; but α ≠ 1.

14. The value of α for which the sum of the squares of the roots of the equation
x2 – (a – 2)x – a – 1 = 0 assume the least value is
(1) 1 (2) 0
(3) 3 (4) 2
14. (1)
x2 – (a – 2)x – a – 1 = 0
⇒α+β=a–2
α β = –(a + 1)
α2 + β2 = (α + β)2 - 2αβ

= a2 – 2a + 6 = (a – 1)2 + 5
⇒ a = 1.

15. If roots of the equation x2 – bx + c = 0 be two consectutive integers, then b2 – 4c


equals
(1) – 2 (2) 3

FIITJEE Ltd. ICES House, 29-A, Kalu Sarai, Sarvapriya Vihar, New Delhi - 110016, Ph : 26515949, 26569493, Fax : 26513942
–5–

(3) 2 (4) 1
15. (4)
Let α, α + 1 be roots
α+α+1=b
α(α + 1) = c
∴ b2 – 4c = (2α + 1)2 - 4α(α + 1) = 1.

16. If the letters of word SACHIN are arranged in all possible ways and these words are
written out as in dictionary, then the word SACHIN appears at serial number
(1) 601 (2) 600
(3) 603 (4) 602
16. (1)
Alphabetical order is
A, C, H, I, N, S
No. of words starting with A – 5!
No. of words starting with C – 5!
No. of words starting with H – 5!
No. of words starting with I – 5!
No. of words starting with N – 5!
SACHIN – 1
601.

6
17. The value of 50
C4 + ∑ r =1
56 −r
C3 is

(1) 55C4 (2) 55C3


(3) 56C3 (4) 56C4
17. (4)
6
50
C4 + ∑r =1
56 −r
C3

⇒ 50
C4 +  55 C3 + 54 C3 + 53 C3 + 52C3 + 51C3 + 50 C3 
= ( 50
)
C4 + 50 C3 + 51C3 + 52C3 + 53 C3 + 54 C3 + 55 C3
⇒ ( 51
C4 + 51 C )+3
52
C3 + 53 C3 + 54 C3 + 55 C3
⇒ 55C4 + 55C3 = 56C4.

1 0   1 0
18. If A =   and I =   , then which one of the following holds for all n ≥ 1, by
1 1  0 1
the principle of mathematical indunction
(1) An = nA – (n – 1)I (2) An = 2n-1A – (n – 1)I
n
(3) A = nA + (n – 1)I (4) An = 2n-1A + (n – 1)I
18. (1)
By the principle of mathematical induction (1) is true.

11 11
  1  7 -7   1 
19. If the coefficient of x in ax 2 +    equals the coefficient of x in ax 2 −    ,
  bx     bx  
then a and b satisfy the relation
(1) a – b = 1 (2) a + b = 1
a
(3) = 1 (4) ab = 1
b
19. (4)

FIITJEE Ltd. ICES House, 29-A, Kalu Sarai, Sarvapriya Vihar, New Delhi - 110016, Ph : 26515949, 26569493, Fax : 26513942
–6–

11 r
 1 11−r  1 
Tr + 1 in the expansion ax 2 +  = 11Cr ax 2
 bx 
(
 
 bx 
)
11 11 – r -r 22 – 2r – r
= Cr (a) (b) (x)
⇒ 22 – 3r = 7 ⇒ r = 5
∴ coefficient of x7 = 11C5(a)6 (b)-5 ……(1)
11 r
 1  11−r  1 
Again Tr + 1 in the expansion ax − 2  = 11Cr ( ax )  − 2 
 bx   bx 
11 11 – r r -r -2r 11 - r
= Cr a (-1) × (b) (x) (x)
Now 11 – 3r = -7 ⇒ 3r = 18 ⇒ r = 6
∴ coefficient of x-7 = 11C6 a5 × 1 × (b)-6
C5 ( a ) ( b ) = 11C6a5 × ( b )
6 −5 −6
⇒ 11

⇒ ab = 1.

2x
20. Let f : (-1, 1) → B, be a function defined by f(x) = tan−1 , then f is both one-one
1 − x2
and onto when B is the interval
 π  π
(1)  0,  (2) 0, 
 2  2
 π π  π π
(3)  − ,  (4)  − , 
 2 2  2 2
20. (4)
 2x 
Given f(x) = tan−1  2 
for x∈(-1, 1)
 1− x 
 π π
clearly range of f(x) =  − , 
 2 2
 π π
∴ co-domain of function = B =  − ,  .
 2 2

21. If z1 and z2 are two non-zero complex numbers such that |z1 + z2| = |z1| + |z2| then
argz1 – argz2 is equal to
π
(1) (2) - π
2
π
(3) 0 (4) -
2
21. (3)
|z1 + z2| = |z1| + |z2| ⇒ z1 and z2 are collinear and are to the same side of origin;
hence arg z1 – arg z2 = 0.

z
22. If ω = and |ω| = 1, then z lies on
1
z− i
3
(1) an ellipse (2) a circle
(3) a straight line (4) a parabola.
22. (3)

FIITJEE Ltd. ICES House, 29-A, Kalu Sarai, Sarvapriya Vihar, New Delhi - 110016, Ph : 26515949, 26569493, Fax : 26513942
–7–

z |z|
As given w = ⇒ |w| = = 1 ⇒ distance of z from origin and point
1 1
z− i | z − i|
3 3
 1
 0, 3  is same hence z lies on bisector of the line joining points (0, 0) and (0, 1/3).
 
Hence z lies on a straight line.

1 + a2 x (1 + b ) x (1 + c ) x
2 2

23. If a2 + b2 + c2 = -2 and f(x) = 1 + a2 ( ) x 1 + b x (1 + c ) x


2 2
then f(x) is a

(1 + a 2
) x (1 + b ) x 1 + c x
2 2

polynomial of degree
(1) 1 (2) 0
(3) 3 (4) 2
23. (4)
( ) (1 + b ) x (1 + c ) x
1 + a2 + b 2 + c 2 + 2 x 2 2

f ( x ) = 1 + ( a + b + c + 2 ) x 1 + b x (1 + c ) x , Applying C
2 2 2 2 2
1 → C1 + C2 + C3
1 + ( a + b + c + 2 ) x (1 + b ) x 1 + c x
2 2 2 2 2

1 (1 + b ) x (1 + c ) x
2 2

= 1 1 + b x (1 + c ) x ∵ a + b + c + 2 = 0
2 2 2 2 2

1 (1 + b ) x 1 + c x
2 2

0 x −1 0
f(x) = 0 1− x x − 1 ; Applying R1 → R1 – R2 , R2 → R2 – R3
1 (1 + b ) x2
1 + c2 x
f(x) = (x – 1)2
Hence degree = 2.

24. The normal to the curve x = a(cosθ + θ sinθ), y = a( sinθ - θ cosθ) at any point ‘θ’ is
such that
(1) it passes through the origin
π
(2) it makes angle + θ with the x-axis
2
 π 
(3) it passes through  a , − a 
 2 
(4) it is at a constant distance from the origin
24. (4)
dy
Clearly = tan θ ⇒ slope of normal = - cot θ
dx
Equation of normal at ‘θ’ is
y – a(sin θ - θ cos θ) = - cot θ(x – a(cos θ + θ sin θ)
⇒ y sin θ - a sin2 θ + a θ cos θ sin θ = -x cos θ + a cos2 θ + a θ sin θ cos θ
⇒ x cos θ + y sin θ = a
Clearly this is an equation of straight line which is at a constant distance ‘a’ from
origin.

FIITJEE Ltd. ICES House, 29-A, Kalu Sarai, Sarvapriya Vihar, New Delhi - 110016, Ph : 26515949, 26569493, Fax : 26513942
–8–

25. A function is matched below against an interval where it is supposed to be


increasing. Which of the following pairs is incorrectly matched?
Interval Function
(1) (-∞, ∞) x3 – 3x2 + 3x + 3
(2) [2, ∞) 2x3 – 3x2 – 12x + 6
 1
(3)  −∞,  3x2 – 2x + 1
 3
(4) (- ∞, -4] x3 + 6x2 + 6
25. (3)
Clearly function f(x) = 3x2 – 2x + 1 is increasing when
f′(x) = 6x – 2 ≥ 0 ⇒ x∈[1/3, ∞)
Hence (3) is incorrect.

26. Let α and β be the distinct roots of ax + bx + c = 0, then lim 2 (


1 − cos ax 2 + bx + c ) is
( x − α)
x →α 2

equal to
a2
(α − β)
2
(1) (2) 0
2
a2 1
(3) − ( α − β ) (α − β)
2 2
(4)
2 2
26. (1)
 ( x − α )( x − β ) 
2 sin2  a 
1 − cos a ( x − α )( x − β )  2 
Given limit = lim = lim
( ) ( )
2 2
x →α
x − α x →α
x − α
 ( x − α )( x − β ) 
sin2  a 
a2 ( x − α ) ( x − β )
2 2
2  2 
= lim × ×
(x − α) a2 ( x − α ) ( x − β )
2 2 2
x →α 4
4
a2 ( α − β )
2

= .
2

1
27. Suppose f(x) is differentiable x = 1 and lim f (1 + h ) = 5 , then f′(1) equals
h →0 h

(1) 3 (2) 4
(3) 5 (4) 6
27. (3)
f (1 + h ) − f (1)
f ′ (1) = lim ; As function is differentiable so it is continuous as it is given
h→0 h
f (1 + h )
that lim = 5 and hence f(1) = 0
h →0 h
f (1 + h )
Hence f′(1) = lim =5
h →0 h
Hence (3) is the correct answer.

28. Let f be differentiable for all x. If f(1) = - 2 and f′(x) ≥ 2 for x ∈ [1, 6] , then
(1) f(6) ≥ 8 (2) f(6) < 8
(3) f(6) < 5 (4) f(6) = 5

FIITJEE Ltd. ICES House, 29-A, Kalu Sarai, Sarvapriya Vihar, New Delhi - 110016, Ph : 26515949, 26569493, Fax : 26513942
–9–

28. (1)
As f(1) = - 2 & f′(x) ≥ 2 ∀ x ∈ [1, 6]
Applying Lagrange’s mean value theorem
f ( 6 ) − f (1)
= f ′ (c ) ≥ 2
5
⇒ f(6) ≥ 10 + f(1)
⇒ f(6) ≥ 10 – 2
⇒ f(6) ≥ 8.

29. If f is a real-valued differentiable function satisfying |f(x) – f(y)| ≤ (x – y)2, x, y ∈ R and


f(0) = 0, then f(1) equals
(1) -1 (2) 0
(3) 2 (4) 1
29. (2)
f ( x + h) − f ( x )
f′(x) = lim
h→0 h
f ( x + h) − f ( x ) (h)
2

| f ′ ( x ) |= lim ≤ lim
h→0 h h→0 h
⇒ |f′(x)| ≤ 0 ⇒ f′(x) = 0 ⇒ f(x) = constant
As f(0) = 0 ⇒ f(1) = 0.

30. If x is so small that x3 and higher powers of x may be neglected, then


3
 1 
(1 + x )
3/2
− 1 + x 
 2 
may be approximated as
(1 − x )
1/ 2

3 2 3 2
(1) 1 − x (2) 3x +x
8 8
3 2 x 3
(3) − x (4) − x 2
8 2 8
30. (3)
 3 33  2 1  1  
2

(1 – x) 1 + x +  − 1 x − 1 − 3  x  − 3 ( 2 )  x  
1/2

 2 22  2   2  
 3  3
= (1 – x)1/2  − x 2  = - x 2 .
 8  8

∞ ∞ ∞
31. If x = ∑a ,
n =0
n
y = ∑ bn , z = ∑ c n where a, b, c are in A.P. and |a| < 1, |b|<1, |c|< 1,
n=0 n =0

then x, y, z are in
(1) G.P. (2) A.P.
(3) Arithmetic − Geometric Progression (4) H.P.
31. (4)

1 1
x = ∑ an = a = 1−
n =0 1− a x

1 1
y = ∑ bn = b = 1−
n =0 1− b y

FIITJEE Ltd. ICES House, 29-A, Kalu Sarai, Sarvapriya Vihar, New Delhi - 110016, Ph : 26515949, 26569493, Fax : 26513942
–10–


1 1
z = ∑ cn = c = 1−
n =0 1 − c z
a, b, c are in A.P.
2b = a + c
 1 1 1
2 1 −  = 1 − + 1 −
 y x y
2 1 1
= +
y x z
⇒ x, y, z are in H.P.

π
32. In a triangle ABC, let ∠C = . If r is the inradius and R is the circumradius of the the
2
triangle ABC, then 2 (r + R) equals
(1) b + c (2) a + b
(3) a + b + c (4) c + a
32. (2)
(a + b) + c (a + b) = a + b
2
2ab
2r + 2R = c + = ( since c2 = a2 + b2).
( a + b + c ) ( a + b + c )
y
33. If cos−1 x − cos−1 = α, then 4x2 − 4xy cos α + y2 is equal to
2
(1) 2 sin 2α (2) 4
(3) 4 sin2 α (4) − 4 sin2 α
33. (3)
y
cos-1x – cos-1 2 = α

 xy  y2  
cos−1 
 2
( )
+ 1 − x2 1 −   = α
4 
  
 xy + 4 − y 2 − 4x 2 + x 2 y 2 
cos−1  =α
 2 
 
⇒ 4 – y – 4x + x y = 4 cos α + x2y2 – 4xy cosα
2 2 2 2 2

⇒ 4x2 + y2 – 4xy cosα = 4 sin2α.

34. If in a triangle ABC, the altitudes from the vertices A, B, C on opposite sides are in
H.P., then sin A, sin B, sin C are in
(1) G.P. (2) A.P.
(3) Arithmetic − Geometric Progression (4) H.P.
34. (2)
1 1 1
∆ = p1a = p2b = p3b
2 2 2
p1, p2, p3 are in H.P.
2∆ 2∆ 2∆
⇒ , , are in H.P.
a b c
1 1 1
⇒ , , are in H.P
a b c
⇒ a, b, c are in A.P.
⇒ sinA, sinB, sinC are in A.P.

FIITJEE Ltd. ICES House, 29-A, Kalu Sarai, Sarvapriya Vihar, New Delhi - 110016, Ph : 26515949, 26569493, Fax : 26513942
–11–

1 1 2 2
If I1 = ∫ 2x dx, I2 = ∫ 2x dx , I3 = ∫ 2 dx and I4 = ∫ 2 dx then
2 3 2 3
x x
35.
0 0 1 1
(1) I2 > I1 (2) I1 > I2
(3) I3 = I4 (4) I3 > I4
35. (2)
1 1 1 1
I1 = ∫ 2 dx , I2 = ∫ 2 dx , I3 = ∫ 2 dx , I4 =
x2 x3 x2
∫2
x3
dx
0 0 0 0

∀ 0 < x < 1, x2 > x3


1 1

∫ 2 dx > ∫2
2
x3
⇒ x
dx
0 0

⇒ I1 > I2.

36. The area enclosed between the curve y = loge (x + e) and the coordinate axes is
(1) 1 (2) 2
(3) 3 (4) 4
36. (1)
0
Required area (OAB) = ∫ ln ( x + e )dx
1− e
1
 1 
=  x ln ( x + e ) − ∫ x dx  = 1.
 x+e 0

37. The parabolas y2 = 4x and x2 = 4y divide the square region bounded by the lines x =
4, y = 4 and the coordinate axes. If S1, S2, S3 are respectively the areas of these
parts numbered from top to bottom; then S1 : S2 : S3 is
(1) 1 : 2 : 1 (2) 1 : 2 : 3
(3) 2 : 1 : 2 (4) 1 : 1 : 1
37. (4)
y2 = 4x and x2 = 4y are symmetric about line y = x
4
⇒ area bounded between y = 4x and y = x is ∫ 2 x − x dx =
2

0
8
3
( )
16 16
⇒ A s2 = and A s1 = A s3 =
3 3
⇒ A s1 : A s2 : A s3 :: 1 : 1 : 1.

dy
38. If x = y (log y − log x + 1), then the solution of the equation is
dx
x y
(1) y log   = cx (2) x log   = cy
y
  x
y x
(3) log   = cx (4) log   = cy
x y
38. (3)
x dy
= y (log y – log x + 1)
dx
dy y  y 
=  log   + 1
dx x  x 
Put y = vx

FIITJEE Ltd. ICES House, 29-A, Kalu Sarai, Sarvapriya Vihar, New Delhi - 110016, Ph : 26515949, 26569493, Fax : 26513942
–12–

dy x dv
=v+
dx dx
xdv
⇒ v+ = v ( log v + 1)
dx
xdv
= v log v
dx
dv dx
⇒ =
v log v x
put log v = z
1
dv = dz
v
dz dx
⇒ =
z x
ln z = ln x + ln c
z = cx
log v = cx
y
log   = cx .
x

39. The line parallel to the x−axis and passing through the intersection of the lines ax +
2by + 3b = 0 and bx − 2ay − 3a = 0, where (a, b) ≠ (0, 0) is
3
(1) below the x−axis at a distance of from it
2
2
(2) below the x−axis at a distance of from it
3
3
(3) above the x−axis at a distance of from it
2
2
(4) above the x−axis at a distance of from it
3
39. (1)
ax + 2by + 3b + λ(bx – 2ay – 3a) = 0
⇒ (a + bλ)x + (2b – 2aλ)y + 3b - 3λa = 0
a + bλ = 0 ⇒ λ = -a/b
a
⇒ ax + 2by + 3b - (bx – 2ay – 3a) = 0
b
2a2 3a2
⇒ ax + 2by + 3b – ax + y+ =0
b b
 2a2  3a2
y  2b +  + 3b + =0
 b  b
 2b2 + 2a2   3b2 + 3a2 
y  = − 
 b   b 

y=
(
−3 a 2 + b 2 ) = −3
2 (b 2
+a 2
) 2
3
y=− so it is 3/2 units below x-axis.
2

FIITJEE Ltd. ICES House, 29-A, Kalu Sarai, Sarvapriya Vihar, New Delhi - 110016, Ph : 26515949, 26569493, Fax : 26513942
–13–

40. A spherical iron ball 10 cm in radius is coated with a layer of ice of uniform thickness
than melts at a rate of 50 cm3/min. When the thickness of ice is 5 cm, then the rate at
which the thickness of ice decreases, is
1 1
(1) cm/min (2) cm/min
36π 18π
1 5
(3) cm/min (4) cm/min
54π 6π
40. (2)
dv
= 50
dt
dr
4πr2 = 50
dt
dr 50
⇒ = where r = 15
dt 4π (15 )2
1
= .
16π

2
 (log x − 1) 
41. ∫  (1 + (log x)2  dx is equal to
log x x
(1) +C (2) +C
(log x)2 + 1 x +1
2

xe x x
(3) +C (4) +C
1 + x2 (log x)2 + 1
41. (4)
(log x − 1) dx
2


(1 + ( log x ) )
2 2

 
 1 2log x 
= ∫ − dx

(
 1 + ( log x )
2
) (
1 + ( log x ) 
2
2

 )
 t 
e 2t et 
= ∫ − dt put logx = t ⇒ dx = et dt
1 + t 2
( 
)
2

 1+ t 2

 
t  1 2t 
∫ 1 + t 2 1 + t 2 2 dt
e −
 (  )
et x
= +c = +c
1+ t 1 + ( log x )
2 2

 1 
42. Let f : R → R be a differentiable function having f (2) = 6, f′ (2) =   . Then
 48 
f ( x)
4t 3
lim ∫ dt equals
x →2
6
x−2
(1) 24 (2) 36
(3) 12 (4) 18

FIITJEE Ltd. ICES House, 29-A, Kalu Sarai, Sarvapriya Vihar, New Delhi - 110016, Ph : 26515949, 26569493, Fax : 26513942
–14–

42. (4)
f ( x)
4t 3
x→2 ∫ x − 2
lim dt
0
Applying L Hospital rule
lim  4f ( x ) f ′ ( x )  = 4f(2)3 f′(2)
2

x →2  
1
= 4 × 63 × = 18.
48

43. Let f (x) be a non−negative continuous function such that the area bounded by the
π π
curve y = f (x), x−axis and the ordinates x = and x = β >
4 4
 π   π
is  β sin β + cos β + 2β  . Then f   is
 4  2
π  π 
(1)  + 2 − 1 (2)  − 2 + 1
4  4 
 π   π 
(3)  1 − − 2  (4)  1 − + 2 
 4   4 
43. (4)
β
π
Given that ∫ f ( x ) dx = β sin β + cos β + 2 β
π/4
4
Differentiating w. r. t β
π
f(β) = β cosβ + sinβ - sinβ + 2
4
 π  π π π
f   =  1 −  sin + 2 = 1 − + 2 .
2  4 2 4

44. The locus of a point P (α, β) moving under the condition that the line y = αx + β is a
x2 y2
tangent to the hyperbola 2 − 2 = 1 is
a b
(1) an ellipse (2) a circle
(3) a parabola (4) a hyperbola
44. (4)
x2 y2
Tangent to the hyperbola 2 − 2 = 1 is
a b
y = mx ± a2m2 − b 2
Given that y = αx + β is the tangent of hyperbola
⇒ m = α and a2m2 – b2 = β2
∴ a2α2 – b2 = β2
Locus is a2x2 – y2= b2 which is hyperbola.

x +1 y −1 z − 2
45. If the angle θ between the line = = and the plane 2x − y + λz+4=
1 2 2
1
0 is such that sin θ = the value of λ is
3
5 −3
(1) (2)
3 5

FIITJEE Ltd. ICES House, 29-A, Kalu Sarai, Sarvapriya Vihar, New Delhi - 110016, Ph : 26515949, 26569493, Fax : 26513942
–15–

3 −4
(3) (4)
4 3
45. (1)
Angle between line and normal to plane is
π  2−2+2 λ
cos  − θ  = where θ is angle between line & plane
2  3× 5 + λ
2 λ 1
⇒ sinθ = =
3 5+λ 3
5
⇒λ= .
3

46. The angle between the lines 2x = 3y = − z and 6x = − y = − 4z is


(1) 00 (2) 900
0
(3) 45 (4) 300

46. (2)
Angle between the lines 2x = 3y = - z & 6x = -y = -4z is 90°
Since a1a2 + b1b2 + c1c2 = 0.

47. If the plane 2ax − 3ay + 4az + 6 = 0 passes through the midpoint of the line joining
the centres of the spheres
x2 + y2 + z2 + 6x − 8y − 2z = 13 and
x2 + y2 + z2 − 10x + 4y − 2z = 8, then a equals
(1) − 1 (2) 1
(3) − 2 (4) 2
47. (3)
Plane
2ax – 3ay + 4az + 6 = 0 passes through the mid point of the centre of spheres
x2 + y2 + z2 + 6x – 8y – 2z = 13 and x2 + y2 + z2 – 10x + 4y – 2z = 8 respectively
centre of spheres are (-3, 4, 1) & (5, - 2, 1)
Mid point of centre is (1, 1, 1)
Satisfying this in the equation of plane, we get
2a – 3a + 4a + 6 = 0 ⇒ a = -2.

48. The distance between the line r = 2iˆ − 2ˆj + 3kˆ + λ(iˆ − ˆj + 4k)
ˆ and the plane
r ⋅ (iˆ + 5ˆj + k)
ˆ = 5 is
10 10
(1) (2)
9 3 3
3 10
(3) (4)
10 3
48. (2)
Distance between the line
( ) (
r = 2iˆ − 2jˆ + 3kˆ + λ ˆi − ˆj + 4kˆ and the plane r ⋅ ˆi + 5ˆj + kˆ = 5 is )
equation of plane is x + 5y + z = 5
∴ Distance of line from this plane
= perpendicular distance of point (2, -2, 3) from the plane
2 − 10 + 3 − 5 10
i.e. = .
1+ 5 + 1
2
3 3

FIITJEE Ltd. ICES House, 29-A, Kalu Sarai, Sarvapriya Vihar, New Delhi - 110016, Ph : 26515949, 26569493, Fax : 26513942
–16–

49. For any vector a , the value of (a × ˆi)2 + (a × ˆj)2 + (a × k)


ˆ 2 is equal to
(1) 3a2 (2) a2
(3) 2a2 (4) 4a2
49. (3)
Let a = xiˆ + yjˆ + zkˆ
a × ˆi = zjˆ − ykˆ

( )
2
⇒ a × ˆi = y 2 + z2

( )
2
similarly a × ˆj = x 2 + z2

(a × kˆ ) = x + y ⇒ (a × ˆi ) = y + z
2 2
2 2 2 2
and

similarly ( a × ˆj ) = x + z
2
2 2

and ( a × kˆ ) = x + y
2
2 2

⇒ ( a × ˆi ) + ( a × ˆj ) + ( a × kˆ ) = 2 ( x + y + z ) = 2 a
2 2 2
2 2 2 2
.
x y 1
50. If non-zero numbers a, b, c are in H.P., then the straight line + + = 0 always
a b c
passes through a fixed point. That point is
(1) (-1, 2) (2) (-1, -2)
 1
(3) (1, -2) (4)  1, − 
 2
50. (3)
a, b, c are in H.P.
2 1 1
⇒ − − =0
b a c
x y 1
+ + =0
a b c
x y 1
⇒ = = ∴ x = 1, y = -2
−1 2 −1

51. If a vertex of a triangle is (1, 1) and the mid-points of two sides through this vertex
are (-1, 2) and (3, 2), then the centroid of the triangle is
 7  −1 7 
(1)  −1,  (2)  , 
 3  3 3
 7  1 7
(3)  1,  (4)  , 
 3 3 3
51. (3)
Vertex of triangle is (1, 1) and midpoint of sides A(1, 1)
through this vertex is (-1, 2) and (3, 2)
⇒ vertex B and C come out to be
(-3, 3) and (5, 3)
1− 3 + 5 1+ 3 + 3 (-1, 2)
∴ centroid is , (3, 2)
3 3
⇒ (1, 7/3)

B C

FIITJEE Ltd. ICES House, 29-A, Kalu Sarai, Sarvapriya Vihar, New Delhi - 110016, Ph : 26515949, 26569493, Fax : 26513942
–17–

52. If the circles x2 + y2 + 2ax + cy + a = 0 and x2 + y2 – 3ax + dy – 1 = 0 intersect in two


distinct points P and Q then the line 5x + by – a = 0 passes through P and Q for
(1) exactly one value of a (2) no value of a
(3) infinitely many values of a (4) exactly two values of a
52. (2)
S1 = x2 + y2 + 2ax + cy + a = 0
S2 = x2 + y2 – 3ax + dy – 1 = 0
Equation of radical axis of S1 and S2
S1 – S2 = 0
⇒ 5ax + (c – d)y + a + 1 = 0
Given that 5x + by – a = 0 passes through P and Q
a c − d a +1
⇒ = =
1 b −a
2
⇒ a + 1 = -a
a2 + a + 1 = 0
No real value of a.

53. A circle touches the x-axis and also touches the circle with centre at (0, 3) and radius
2. The locus of the centre of the circle is
(1) an ellipse (2) a circle
(3) a hyperbola (4) a parabola
53. (4)
Equation of circle with centre (0, 3) and radius 2 is
x2 + (y – 3)2 = 4.
Let locus of the variable circle is (α, β)
∵ It touches x-axis.
∴ It equation (x - α)2 + (y - β)2 = β2
Circles touch externally
α2 + (β − 3 ) = 2 + β
2

(α, β)
α2 + (β - 3)2 = β2 + 4 + 4β
α2 = 10(β - 1/2)
∴ Locus is x2 = 10(y – 1/2) which is parabola.

54. If a circle passes through the point (a, b) and cuts the circle x2 + y2 = p2 orthogonally,
then the equation of the locus of its centre is
(1) x2 + y2 – 3ax – 4by + (a2 + b2 – p2) = 0 (2) 2ax + 2by – (a2 – b2 + p2) = 0
(3) x2 + y2 – 2ax – 3by + (a2 – b2 – p2) = 0 (4) 2ax + 2by – (a2 + b2 + p2) = 0
54. (4)
Let the centre be (α, β)
∵ It cut the circle x2 + y2 = p2 orthogonally
2(-α) × 0 + 2(-β) × 0 = c1 – p2
c1 = p2
Let equation of circle is x2 + y2 - 2αx - 2βy + p2 = 0
It pass through (a, b) ⇒ a2 + b2 - 2αa - 2βb + p2 = 0
Locus ∴ 2ax + 2by – (a2 + b2 + p2) = 0.

55. An ellipse has OB as semi minor axis, F and F′ its focii and the angle FBF′ is a right
angle. Then the eccentricity of the ellipse is
1 1
(1) (2)
2 2

FIITJEE Ltd. ICES House, 29-A, Kalu Sarai, Sarvapriya Vihar, New Delhi - 110016, Ph : 26515949, 26569493, Fax : 26513942
–18–

1 1
(3) (4)
4 3
55. (1)
∵ ∠FBF′ = 90o B(0, b)

( ) +( )
2 2
∴ a2 e2 + b2 a2 e2 + b2 = (2ae)2
⇒ 2(a2 e2 + b2) = 4a2e2
F′(-ae, 0) O F(ae, 0)
⇒ e2 = b2/a2
Also e2 = 1- b2/a2 = 1 – e2
1
⇒ 2e2 = 1, e = .
2

56. Let a, b and c be distinct non-negative numbers. If the vectors aiˆ + ajˆ + ck,
ˆ ˆi + kˆ and
ciˆ + cjˆ + bkˆ lie in a plane, then c is
(1) the Geometric Mean of a and b (2) the Arithmetic Mean of a and b
(3) equal to zero (4) the Harmonic Mean of a and b
56. (1)
Vector a ˆi + ajˆ + ckˆ , ˆi + kˆ and ciˆ + cjˆ + bkˆ are coplanar
a a c
1 0 1 = 0 ⇒ c2 = ab
c c b
∴ a, b, c are in G.P.

57. If a, b, c are non-coplanar vectors and λ is a real number then

 ( )
 λ a + b λ 2b λc  = a b + c b  for
  
(1) exactly one value of λ (2) no value of λ
(3) exactly three values of λ (4) exactly two values of λ
57. (2)

 ( )
 λ a + b λ 2b λ c  =  a b + c b 
  
λ λ 0 1 0 0
0 λ2 0 =0 1 1
0 0 λ 0 1 0
⇒ λ4 = -1
Hence no real value of λ.

58. Let a = ˆi − k,ˆ b = xiˆ + ˆj + (1 − x ) kˆ and c = yiˆ + xjˆ + (1 + x − y ) kˆ . Then a, b, c 


 
depends on
(1) only y (2) only x
(3) both x and y (4) neither x nor y
58. (4)
a = ˆi − kˆ , b = xiˆ + ˆj + (1 − x ) kˆ and c = yiˆ + xjˆ + (1 + x − y ) kˆ
a b c  = a ⋅ b × c
  ( )

FIITJEE Ltd. ICES House, 29-A, Kalu Sarai, Sarvapriya Vihar, New Delhi - 110016, Ph : 26515949, 26569493, Fax : 26513942
–19–

ˆi ˆj kˆ
b×c = x 1 1 − x = î (1 + x – x –x2) - ĵ (x + x2- xy – y + xy) + k̂ (x2 – y)
y x 1+ x − y

(
a. b × c = 1 )
which does not depend on x and y.

59. Three houses are available in a locality. Three persons apply for the houses. Each
applies for one house without consulting others. The probability that all the three
apply for the same house is
2 1
(1) (2)
9 9
8 7
(3) (4)
9 9
59. (2)
For a particular house being selected
1
Probability =
3
 1 1 1 1
Prob(all the persons apply for the same house) =  × ×  3 = .
3 3 3 9

60. A random variable X has Poisson distribution with mean 2. Then P(X > 1.5) equals
2
(1) 2 (2) 0
e
3 3
(3) 1 − 2 (4) 2
e e
60. (3)
λk
P(x = k) = e−λ
k!
P(x ≥ 2) = 1 – P(x = 0) – P(x = 1)
λ
= 1 – e-λ – e-λ  1! 
 
3
=1- .
e2

61.
1
Let A and B be two events such that P A ∪ B =
6
( 1
) 1
, P ( A ∩ B ) = and P A = ,
4 4
( )
where A stands for complement of event A. Then events A and B are
(1) equally likely and mutually exclusive
(2) equally likely but not independent
(3) independent but not equally likely
(4) mutually exclusive and independent
61. (3)

( 1
)
P A ∪ B = , P(A ∩ B) =
6
1
4
and P A =
1
4
( )
⇒ P(A ∪ B) = 5/6 P(A) = 3/4
Also P(A ∪ B) = P(A) + P(B) – P(A ∩ B)
⇒ P(B) = 5/6 – 3/4 + 1/4 = 1/3
P(A) P(B) = 3/4 – 1/3 = 1/4 = P(A ∩ B)

FIITJEE Ltd. ICES House, 29-A, Kalu Sarai, Sarvapriya Vihar, New Delhi - 110016, Ph : 26515949, 26569493, Fax : 26513942
–20–

Hence A and B are independent but not equally likely.

62. A lizard, at an initial distance of 21 cm behind an insect, moves from rest with an
acceleration of 2 cm/s2 and pursues the insect which is crawling uniformly along a
straight line at a speed of 20 cm/s. Then the lizard will catch the insect after
(1) 20 s (2) 1 s
(3) 21 s (4) 24 s
62. (3)
1 2
2t = 21 + 20t
2
⇒ t = 21.

63. Two points A and B move from rest along a straight line with constant acceleration f
and f′ respectively. If A takes m sec. more than B and describes ‘n’ units more than B
in acquiring the same speed then
(1) (f - f′)m2 = ff′n (2) (f + f′)m2 = ff′n
1 1
(3) ( f + f ′ ) m = ff ′n2 (4) ( f ′ − f ) n = ff ′m2
2 2
63. (4)
v2 = 2f(d + n) = 2f′d
v = f′(t) = (m + t)f
eliminate d and m we get
1
(f′ - f)n = ff ′m2 .
2

64. A and B are two like parallel forces. A couple of moment H lies in the plane of A and
B and is contained with them. The resultant of A and B after combining is displaced
through a distance
2H H
(1) (2)
A −B A +B
H H
(3) (4)
2 ( A + B) A −B
64. (2)
(A + B) = d = H
 H 
d=  .
 A +B

65. The resultant R of two forces acting on a particle is at right angles to one of them and
its magnitude is one third of the other force. The ratio of larger force to smaller one is
(1) 2 : 1 (2) 3 : 2
(3) 3 : 2 (4) 3 : 2 2
65. (4)
F′ = 3F cos θ F
F = 3F sin θ 3F

⇒ F′ = 2 2 F
F : F′ : : 3 : 2 2 .
F′

FIITJEE Ltd. ICES House, 29-A, Kalu Sarai, Sarvapriya Vihar, New Delhi - 110016, Ph : 26515949, 26569493, Fax : 26513942
–21–

1 1 1
66. The sum of the series 1 + + + + ......... ad inf. is
4.2! 16.4! 64.6!
e −1 e +1
(1) (2)
e e
e −1 e +1
(3) (4)
2 e 2 e
66. (4)
e x + e− x x2 x4 x6
= 1+ + + + .......
2 2! 4! 6!
putting x = 1/2 we get
e +1
.
2 e

π
cos2 x
67. The value of ∫
−π 1 + a
x
dx, a > 0, is

π
(1) a π (2)
2
π
(3) (4) 2 π
a
67. (2)
π π
cos2 x π
∫−π 1 + ax = ∫0 cos x dx = 2 .
2
dx

68. The plane x + 2y – z = 4 cuts the sphere x2 + y2 + z2 – x + z – 2 = 0 in a circle of


radius
(1) 3 (2) 1
(3) 2 (4) 2
68. (2)
1 1
Perpendicular distance of centre  , 0, −  from x + 2y – 2 = 4
2 2
1 1
+ −4
2 2 3
=
6 2
5 3
radius = − = 1.
2 2

69. If the pair of lines ax2 + 2(a + b)xy + by2 = 0 lie along diameters of a circle and divide
the circle into four sectors such that the area of one of the sectors is thrice the area
of another sector then
(1) 3a2 – 10ab + 3b2 = 0 (2) 3a2 – 2ab + 3b2 = 0
2 2
(3) 3a + 10ab + 3b = 0 (4) 3a2 + 2ab + 3b2 = 0
69. (4)
(a + b)
2
2 − ab
=1
a+b

⇒ (a + b)2 = 4(a2 + b2 + ab)


⇒ 3a2 + 3b2 + 2ab = 0.

FIITJEE Ltd. ICES House, 29-A, Kalu Sarai, Sarvapriya Vihar, New Delhi - 110016, Ph : 26515949, 26569493, Fax : 26513942
–22–

70. Let x1, x2, …,xn be n observations such that ∑x 2


i = 400 and ∑ xi = 80 . Then a
possible value of n among the following is
(1) 15 (2) 18
(3) 9 (4) 12
70. (2)
∑x  ∑ xi 
2 2
i
≥  
n  n 
⇒ n ≥ 16.

71. A particle is projected from a point O with velocity u at an angle of 60o with the
horizontal. When it is moving in a direction at right angles to its direction at O, its
velocity then is given by
u u
(1) (2)
3 2
2u u
(3) (4)
3 3
71. (4)
u cos 60o = v cos 30o
4
v= .
3 30o
o
60 30o

72. If both the roots of the quadratic equation x2 – 2kx + k2 + k – 5 = 0 are less than 5,
then k lies in the interval
(1) (5, 6] (2) (6, ∞)
(3) (-∞, 4) (4) [4, 5]
72. (3)
−b
<5
2a
f(5) > 0
⇒ k∈(-∞, 4).

73. If a1, a2, a3,…, an,… are in G.P., then the determinant
logan logan+1 logan+ 2
∆ = logan+ 3 logan+ 4 logan+ 5 is equal to
logan + 6 logan + 7 logan+ 8
(1) 1 (2) 0
(3) 4 (4) 2
73. (2)
C1 – C2, C2 – C3
two rows becomes identical
Answer: 0.

74. A real valued function f(x) satisfies the functional equation f(x – y) = f(x) f(y) – f(a – x)
f(a + y) where a is a given constant and f(0) = 1, f(2a – x) is equal to
(1) –f(x) (2) f(x)
(3) f(a) + f(a – x) (4) f(-x)

FIITJEE Ltd. ICES House, 29-A, Kalu Sarai, Sarvapriya Vihar, New Delhi - 110016, Ph : 26515949, 26569493, Fax : 26513942
–23–

74. (1)
f(a – (x – a)) = f(a) f(x – a) – f(0) f(x)
= -f(x) ∵ x = 0, y = 0, f ( 0 ) = f 2 ( 0 ) − f 2 ( a ) ⇒ f 2 ( a ) = 0 ⇒ f ( a ) = 0  .

75. If the equation


an xn + an−1x n−1 + ...... + a1x = 0 , a1 ≠ 0, n ≥ 2, has a positive root x = α, then the
equation nan x n−1 + (n − 1) an−1x n− 2 + ..... + a1 = 0 has a positive root, which is
(1) greater than α (2) smaller than α
(3) greater than or equal to α (4) equal to α
75. (2)
f(0) = 0, f(α) = 0
⇒ f′(k) = 0 for some k∈(0, α).

FIITJEE Ltd. ICES House, 29-A, Kalu Sarai, Sarvapriya Vihar, New Delhi - 110016, Ph : 26515949, 26569493, Fax : 26513942
–1–

FIITJEE
SOLUTION TO AIEEE-2005
CHEMISTRY
76. Which of the following oxides is amphoteric in character?
(1) CaO (2) CO2
(3) SiO2 (4) SnO2
76. (4)
CaO → basic
SiO2 & CO2 → acidic
SnO2 → amphoteric

77. Which one of the following species is diamagnetic in nature?


(1) He2+ (2) H2
(3) H2+ (4) H2−
77. (2)
H2 σ1s2 σ*1s0, no unpaired so diamagnetic

78. If α is the degree of dissociation of Na2SO4, the vant Hoff’s factor (i) used for
calculating the molecular mass is
(1) 1 + α (2) 1 - α
(3) 1 + 2 α (4) 1 – 2 α
78. (3)
Na2SO4 2Na+ + SO4-2
1-α 2α α
Total moles = 1+2α

79. The oxidation state of Cr in [Cr(NH3)4Cl2]+ is


(1) +3 (2) +2
(3) +1 (4) 0
79. (1)
(Cr(NH3)4Cl2)+
X + 4×0 + 2×-1 = 1
X = +3

80. Hydrogen bomb is based on the principle of


(1) Nuclear fission (2) Natural radioactivity
(3) Nuclear fusion (4) Artificial radioactivity
80. (3)

81. An ionic compound has a unit cell consisting of A ions at the corners of a cube and B
ions on the centres of the faces of the cube. The empirical formula for this compound
would be
(1) AB (2) A2B
(3) AB3 (4) A3B
81. (3)
1
A = ×8 = 1
8
(Corner)

FIITJEE Ltd. ICES House, 29-A, Kalu Sarai, Sarvapriya Vihar, New Delhi - 110016, Ph : 26515949, 26569493, Fax : 26513942
–2–

1
B= ×6 =3
2
(Face centre)
∴ AB3

82. For a spontaneous reaction the ∆G , equilibrium constant (K) and Eocell will be
respectively
(1) -ve, >1, +ve (2) +ve, >1, -ve
(3) -ve, <1, -ve (4) -ve, >1, -ve
82. (1)

83. Which of the following is a polyamide?


(1) Teflon (3) Nylon – 66
(3) Terylene (4) Bakelite
83 (2)
O
N
H
Amide → Nylon 66

84. Which one of the following types of drugs reduces fever?


(1) Analgesic (2) Antipyretic
(3) Antibiotic (4) Tranquiliser
84. (2)

85. Due to the presence of an unpaired electron, free radicals are:


(1) Chemically reactive (2) Chemically inactive
(3) Anions (4) Cations
85. (1)

86. Lattice energy of an ionic compounds depends upon


(1) Charge on the ion only (2) Size of the ion only
(3) Packing of ions only (4) Charge on the ion and size of the ion
86. (4)

87. The highest electrical conductivity of the following aqueous solutions is of


(1) 0.1 M acetic acid (2) 0.1 M chloroacetic acid
(3) 0.1 M fluoroacetic acid (4) 0.1 M difluoroacetic acid
87. (4)

88. Aluminium oxide may be electrolysed at 1000o C to furnish aluminium metal (Atomic
mass = 27 amu; 1 Faraday = 96,500 Coulombs). The cathode reaction is
Al3 + + 3e− → Alo
To prepare 5.12 kg of aluminium metal by this method would require
(1) 5.49 × 107 C of electricity (2) 1.83 × 107 C of electricity
(3) 5.49 × 104 C of electricity (4) 5.49 × 101 C of electricity
88. (1)
mFZ 5.12 × 105 × 96500 × 3
Q= =
M 27
= 5.49×107 C

FIITJEE Ltd. ICES House, 29-A, Kalu Sarai, Sarvapriya Vihar, New Delhi - 110016, Ph : 26515949, 26569493, Fax : 26513942
–3–

89. Consider an endothermic reaction, X → Y with the activation energies Eb and Ef
for the backward and forward reactions, respectively. In general
(1) Eb < Ef
(2) Eb > Ef
(3) Eb = Ef
(4) There is no definite relation between Eb and Ef
89. (1)
∆H = Ef – Eb
For ∆H = Positive, Eb < Ef

90. Consider the reaction: N2 + 3H2 → 2NH3 carried out at constant temperature and
pressure. If ∆H and ∆U are the enthalpy and internal energy changes for the
reaction, which of the following expressions is true?
(1) ∆H = 0 (2) ∆H = ∆U
(3) ∆H < ∆U (4) ∆H > ∆U
90. (3)
∆H = ∆U + ∆nRT
∆n = -2
∆H = ∆U – 2RT
∆H < ∆U

91. Which one of the following statements is NOT true about the effect of an increase in
temperature on the distribution of molecular speeds in a gas?
(1) The most probable speed increases
(2) The fraction of the molecules with the most probable speed increases
(3) The distribution becomes broader
(4) The area under the distribution curve remains the same as under the lower
temperature
91. (2)
Most probable velocity increase and fraction of molecule possessing most probable
velocity decreases.

92. The volume of a colloidal particle, VC as compared to the volume of a solute particle
in a true solution VS, could be
V V
(1) C  1 (2) C  1023
VS VS
V V
(3) C  10 −3 (4) C  103
VS VS
92. (4)

93. The solubility product of a salt having general formula MX2, in water is: 4 × 10−12. The
concentration of M2+ ions in the aqueous solution of the salt is
(1) 2.0 × 10−6 M (2) 1.0 × 10 −4 M
(3) 1.6 × 10 −4 M (4) 4.0 × 10 −10 M
93. (2)
MX2 M+2 + 2X-
S 2S
K sp
Ksp = 4s3 ,S = 3 = 1× 10 −4
4

FIITJEE Ltd. ICES House, 29-A, Kalu Sarai, Sarvapriya Vihar, New Delhi - 110016, Ph : 26515949, 26569493, Fax : 26513942
–4–

94. Benzene and toluene form nearly ideal solutions. At 20o C, the vapour pressure of
benzene is 75 torr and that of toluene is 22 torr. The partial vapour pressure of
benzene at 20o C for a solution containing 78 g of benzene and 46 g of toluene in
torr is
(1) 50 (2) 25
(3) 37.5 (4) 53.5
94. (1)
1
PB = PB° × B = 75 × = 50 torr
1.5

95. The exothermic formation of ClF3 is represented by the equation:


ZZZ
X
Cl2(g) + 3F2(g) YZZ
Z 2ClF3(g) ; ∆rH = −329 kJ
Which of the following will increase the quantity of ClF3 in an equilibrium mixture of
Cl2, F2 and ClF3?
(1) Increasing the temperature (2) Removing Cl2
(3) Increasing the volume of the container (4) Adding F2
95. (4)
M3V3 = M1V2 + M2V2
480(1.5) + 520(1.2)
M= = 1.344M
1000

96. Two solutions of a substance (non electrolyte) are mixed in the following manner.
480 ml of 1.5 M first solution + 520 mL of 1.2 M second solution. What is the molarity
of the final mixture?
(1) 1.20 M (2) 1.50 M
(3) 1.344 M (4) 2.70 M
96. (3)

97. For the reaction


ZZZ
2NO2(g) YZZXZ 2NO(g) + O2(g) ,
(K c = 1.8 × 10−6 at 184o C)
(R = 0.0831 kJ / (mol.K)
When Kp and Kc are compared at 184o C , it is found that
(1) Kp is greater than Kc
(2) Kp is less than Kc
(3) Kp = Kc
(4) Whether Kp is greater than, less than or equal to Kc depends upon the total gas
pressure
97. (1)
Kp = Kc RT∆n, ∆n =1
Kp > Kc

98. Hydrogen ion concentration in mol / L in a solution of pH = 5.4 will be


(1) 3.98 × 108 (2) 3.88 × 106
(3) 3.68 × 10−6 (4) 3.98 × 10−6
98. (4)
pH = - log (H+)

99. A reaction involving two different reactants can never be


(1) Unimolecular reaction (2) First order reaction
(3) second order reaction (4) Bimolecular reaction

FIITJEE Ltd. ICES House, 29-A, Kalu Sarai, Sarvapriya Vihar, New Delhi - 110016, Ph : 26515949, 26569493, Fax : 26513942
–5–

99. (1)

1 1
100. If we consider that , in place of ; mass of carbon atom is taken to be the relative
6 12
atomic mass unit, the mass of one mole of a substance will
(1) Decrease twice
(2) Increase two fold
(3) Remain unchanged
(4) Be a function of the molecular mass of the substance
100. (3)

101. In a multi – electron atom, which of the following orbitals described by the three
quantum numbers will have the same energy in the absence of magnetic acid and
electric fields?
(a) n = 1, l = 0, m = 0
(b) n = 2, l = 0, m = 0
(c) n = 2, l = 1, m = 1
(d) n = 3, l = 2, m = 1
(e) n = 3, l = 2, m = 0
(1) (a) and (b) (2) (b) and (c)
(3) (c) and (d) (4) (d) and (e)
101. (4)
n = same

102. During the process of electrolytic refining of copper, some metals present as impurity
settle as ‘anode mud’ These are
(1) Sn and Ag (2) Pb and Zn
(3) Ag and Au (4) Fe and Ni
102. (3)

103. Electrolyte KCl KNO3 HCl NaOAc NaCl


∧∞(S cm2mol- 149.9 145.0 426.2 91.0 126.5
1
)

Calculate ∧HOAc Using appropriate molar conductances of the electrolytes listed
above at infinite dilution in H2O at 25°C
(1) 517.2 (2) 552.7
(3) 390.7 (4) 217.5
103. (3)
∧ ∞AcOH = ∧HCl

+ ∧ ∞AcONa − ∧NaCl

= 390.7

104. A schematic plot of In Keq versus inverse of temperature for a reaction is shown
below

6.0
In Keq

2.0
1 −1
1.5 × 10 −3 (K ) 2.0 × 10 −3
T
The reaction must be

FIITJEE Ltd. ICES House, 29-A, Kalu Sarai, Sarvapriya Vihar, New Delhi - 110016, Ph : 26515949, 26569493, Fax : 26513942
–6–

(1) exothermic (2) endothermic


(3) one with negligible enthalpy change (4) highly spontaneous at ordinary
temperature
104. (1)
∆H
Keq = A e−
RT

105. The disperse phase in colloidal iron (III) hydroxide and colloidal gold is positively and
negatively charged, respectively, which of the following statements is NOT correct?

(1) magnesium chloride solution coagulates, the gold sol more readily than the iron
(III) hydroxide sol.
(2) sodium sulphate solution causes coagulation in both sols
(3) mixing the sols has no effect
(4) coagulation in both sols can be brought about by electrophoresis
105. (3)

106. Based on lattice energy and other considerations which one of the following alkali
metal chlorides is expected to have the highest melting point.
(1) LiCl (2) NaCl
(3) KCl (4) RbCl
106. (2)
Although lattice energy of LiCl higher than NaCl but LiCl is covalent in nature and
NaCl ionic there after , the melting point decreases as we move NaCl because the
lattice energy decreases as a size of alkali metal atom increases (lattice energy ∝ to
melting point of alkali metal halide)

107. Heating mixture of Cu2O and Cu2S will give


(1) Cu + SO2 (2) Cu + SO3
(3) CuO + CuS (4) Cu2SO3
107. (1)
2Cu2O + Cu2S → 6Cu + SO2

108. The molecular shapes of SF4, CF4 and XeF4 are


(1) the same with 2,0 and 1 lone pairs of electrons on the central atom, respectively
(2) the same with 1, 1 and 1 lone pair of electrons on the central atoms, respectively
(3) different with 0, 1 and 2 lone pair of electrons on the central atoms, respectively
(4) different with 1, 0 and 2 lone pairs of electron on the central atoms respectively
108 (4)

109. The number and type of bonds between two carbon atoms in calcium carbide are
(1) One sigma, one pi (2) One sigma, two pi
(3) Two sigma, one pi (4) Two sigma, two pi
109. (2)
C
CaC2 Ca +2
C
One σ
Two π

110. The oxidation state of chromium in the final product formed by the reaction between
KI and acidified potassium dichromate solution is
(1) +4 (2) +6
(3) +2 (4) +3
110. (4)

FIITJEE Ltd. ICES House, 29-A, Kalu Sarai, Sarvapriya Vihar, New Delhi - 110016, Ph : 26515949, 26569493, Fax : 26513942
–7–

Cr2O7−2 + 14H+ + 6I− → 2Cr +3 + 7H2O + 3I2

+6 +3

111. The number of hydrogen atom(s) attached to phosphorus atom in hypophosphorous


acid is
(1) zero (2) two
(3) one (4) three
111. (2)
O

H P H

O
H

112. What is the conjugate base of OH-?


(1) O2 (2) H2O
(3) O- (4) O-2
112. (4)
OH- → O-2 + H+

113. The correct order of the thermal stability of hydrogen halides (H – X) is


(1) HI > HBr > HCl > HF (2) HF > HCl > HBr > HI
(3) HCl < HF > HBr < HI (4) HI > HCl < HF < HBr
113. (2)

114. Heating an aqueous solution of aluminium chloride to dryness will give


(1) AlCl3 (2) Al2Cl6
(3) Al2O3 (4) Al(OH)Cl2
114. (3)
Al2Cl6 6H2O → Al2O3 + + 6HCl + 3H2O↑

115. Calomel (Hg2Cl2) on reaction with ammonium hydroxide gives


(1) HgNH2Cl (2) NH2 – Hg – Hg – Cl
(3) Hg2O (4) HgO
115. (1)
Hg2Cl2 + 2NH4OH → Hg + Hg(NH2 )Cl + NH4Cl + 2H2O

116. In which of the following arrangements the order is NOT according to the property
indicated against it?
(1) Al3+ < Mg2+ < Na+ < F-
Increasing ionic size
(2) B < C < N < O
Increasing first ionization enthalpy
(3) I < Br < F < Cl
Increasing electron gain enthalpy (with negative sign)
(4) Li < Na < K < Rb
Increasing metallic radius
116. (2)
B<C<O<N

FIITJEE Ltd. ICES House, 29-A, Kalu Sarai, Sarvapriya Vihar, New Delhi - 110016, Ph : 26515949, 26569493, Fax : 26513942
–8–

117. In silicon dioxide


(1) Each silicon atom is surrounded by four oxygen atoms and each oxygen atom is
bonded to two silicon atoms
(2) Each silicon atom is surrounded by two oxygen atoms and each oxygen atom is
bonded to two silicon atoms
(3) Silicon atoms is bonded to two oxygen atoms
(4) there are double bonds between silicon and oxygen atoms
117. (1)

O O

O Si O Si O

O O

118. Of the following sets which one does NOT contain isoelectronic species?
(1) PO−43 ,SO−42 ,ClO−4 (2) CN− ,N2 ,C2−2
(3) SO3−2 ,CO3−2 ,NO3− (4) BO3−3 ,CO3−2 ,NO3−
118. (3)

119. The lanthanide contraction is responsible for the fact that


(1) Zr and Y have about the same radius (2) Zr and Nb have similar oxidation
state
(3) Zr and Hf have about the same radius (4) Zr and Zn have the same oxidation
119. (3)
Due to Lanthanide contraction.

120. The IUPAC name of the coordination compound K3[Fe(CN)6] is


(1) Potassium hexacyanoferrate (II) (2) Potassium hexacyanoferrate (III)
(3) Potassium hexacyanoiron (II) (4) tripotassium hexcyanoiron (II)
120. (2)

121. Which of the following compounds shows optical isomerism?


(1) [Cu(NH3)4]+2 (2) [ZnCl4]-2
-3
(3) [Cr(C2O4)3] (4) [Co(CN)6]-3
121. (3)
−3 −3
OX OX

OX Cr Cr OX

OX
OX

122. Which one of the following cyano complexes would exhibit the lowest value of
paramagnetic behaviour?
(1) [Cr(CN)6]-3 (2) [Mn(CN)6]-3
-3
(3) [Fe(CN)6] (4) [Co(CN)6]-3
(At. No. Cr = 24, Mn = 25, Fe = 26, Co = 27)
122. (4)

FIITJEE Ltd. ICES House, 29-A, Kalu Sarai, Sarvapriya Vihar, New Delhi - 110016, Ph : 26515949, 26569493, Fax : 26513942
–9–

123. 2 methylbutane on reacting with bromine in the presence of sunlight gives mainly
(1) 1 – bromo -2 - methylbutane (2) 2 – bromo -2 - methylbutane
(3) 2 – bromo -3 - methylbutane (4) 1 – bromo -3 – methylbutane
123. (2)
Br

H3C CH CH2 CH3 + Br2 → H3C C CH2 CH3

CH3 CH3
Major

24
124. The photon of hard gamma radiation knocks a proton out of 12 Mg nucleus to form
(1) the isotope of parent nucleus (2) the isobar of parent nucleus
23 23
(3) the nuclide 11 Na (4) the isobar of 11 Na
124. (3)

125. The best reagent to convert pent -3- en-2-ol into pent -3-en-2-one is
(1) Acidic permanganate (2) Acidic dichromate
(3) Chromic anhydride in glacial acetic acid (4) Pyridinium chloro – chromate
125. (3)

126. Tertiary alkyl halides are practically inert to substitution by SN2 mechanism because
of
(1) insolubility (2) instability
(3) inductive effect (4) steric hindrance
126. (4)

127. In both DNA and RNA, heterocyclic base and phosphate ester linkages are at-
(1) Cl5 and Cl2 respectively of the sugar molecule
(2) Cl2 and Cl5 respectively of the sugar molecule
(3) C1l and Cl5 respectively of the sugar molecule
(4) Cl5 and C1l respectively of the sugar molecule
127. (3)
NH2
O
- N
O P O
5I
- CH2
O O N O

1I
H H

H H

HO OH

128. Reaction of one molecule of HBr with one molecule of 1,3-butadiene at 400C gives
predominantly
(1) 3-bromobutene under kinetically controlled conditions
(2) 1-bromo-2-butene under thermodymically controlled conditions
(3) 3-bromobutene under thermodynamically controlled conditions
(4) 1-bromo-2-butene under kinetically controlled conditions

FIITJEE Ltd. ICES House, 29-A, Kalu Sarai, Sarvapriya Vihar, New Delhi - 110016, Ph : 26515949, 26569493, Fax : 26513942
–10–

128. (2)

129. Among the following acids which has the lowest pKa value?
(1) CH3COOH
(2) HCOOH
(3) (CH3)2COOH
(4) CH3CH2COOH
129. (2)

130. The decreasing order of nucleophilicity among the nucleophiles


(a) CH3 − C− O−
||
O
(b) CH3O-
(c) CN-
(d) O
-
H3C S O

O
(1) (a), (b), (c), (d) (2) (d), (c), (b), (a)
(3) (b), (c), (a), (d) (4) (c), (b), (a), (d)
130. (4)

131. Which one of the following methods is neither meant for the synthesis nor for
separation of amines?
(1) Hinsberg method
(2) Hofmann method
(3) Wurtz reaction
(4) Curtius reaction
131. (3)

132. Which of the following is fully fluorinated polymer?


(1) Neoprene (2) Teflon
(3) Thiokol (4) PVC
132. (2)

133. Of the five isomeric hexanes, the isomer which can give two monochlorinated
compounds is
(1) n-hexane
(2) 2, 3-dimethylbutane
(3) 2,2-dimethylbutane
(4) 2-methylpentane
133. (2)
CH3 CH3 CH2Cl CH3 CH3 CH3

H3C C C CH3 


Cl2
→ H3C C C CH3 H3C C C CH3

H H H H Cl H

134. Alkyl halides react with dialkyl copper reagents to give


(1) alkenes (2) alkyl copper halides
(3) alkanes (3) alkenyl halides
134. (3)
R2CuLi + R′X → R − R′ + R − Cu + LiX

FIITJEE Ltd. ICES House, 29-A, Kalu Sarai, Sarvapriya Vihar, New Delhi - 110016, Ph : 26515949, 26569493, Fax : 26513942
–11–

135. Acid catalyzed hydration of alkenes except ethene leads to the formation of
(1) primary alcohol
(2) secondary or tertiary alcohol
(3) mixture of primary and secondary alcohols
(4) mixture of secondary and tertiary alcohols
135. (4)

136. Amongst the following the most basic compound is


(1) benzylamine (2) aniline
(3) acetanilide (4) p-nitroaniline
136. (1)
-NH2 group is not linked with benzene ring.

137. Which types of isomerism is shown by 2,3-dichlorobutane?


(1) Diastereo (2) Optical
(3) Geometric (4) Structural
137. (2)
CH3 CH3 CH3

H Cl H Cl Cl H
H Cl
, Cl H
, H Cl

CH3 CH3 CH3

138. The reaction


O O

R C Nu R C X

X Nu
is fastest when X is
(1) Cl (2) NH2
(3) OC2H5 (4) OCOR
138. (1)
Conjugated acid of Cl- is a stronger acid i.e. HCl.

139. Elimination of bromine from 2-bromobutane results in the formation of-


(1) equimolar mixture of 1 and 2-butene (2) predominantly 2-butene
(3) predominantly 1-butene (4) predominantly 2-butyne
139. (2)
Saytzeffs product.

140. Equimolar solutions in the same solvent have


(1) Same boiling point but different freezing point
(2) Same freezing point but different boiling point
(3) Same boiling and same freezing points
(4) Different boiling and different freezing points
140. (3)

141. Which of the following statements in relation to the hydrogen atom is correct?
(1) 3s orbital is lower in energy than 3p orbital
(2) 3p orbital is lower in energy than 3d orbital
(3) 3s and 3p orbitals are of lower energy than 3d orbital
(4) 3s, 3p and 3d orbitals all have the same energy
141. (4)

FIITJEE Ltd. ICES House, 29-A, Kalu Sarai, Sarvapriya Vihar, New Delhi - 110016, Ph : 26515949, 26569493, Fax : 26513942
–12–

142. The structure of diborane (B2H6) contains


(1) four 2c-2e bonds and two 3c-2e bonds
(2) two 2c-2e bonds and four 3c-2e bonds
(3) two 2c-2e bonds and two 3c-3e bonds
(4) four 2c-2e bonds and four 3c-2e bonds
142. (1)
H H H
B B
H H H

143. The value of the ‘spin only’ magnetic moment for one of the following configurations
is 2.84 BM. The correct one is
(1) d4 (in strong ligand filed)
(2) d4 (in weak ligand field)
(3) d3 (in weak as well as in strong fields)
(4) d5 (in strong ligand field)
143. (1)

d4 in strong field, so unpaired electrons = 2.

144. Which of the following factors may be regarded as the main cause of lanthanide
contraction?
(1) Poor shielding of one of 4f electron by another in the subshell
(2) Effective shielding of one of 4f electrons by another in the subshell
(3) Poorer shielding of 5d electrons by 4f electrons
(4) Greater shielding of 5d electrons by 4f electrons
144. (1)

145. Reaction of cyclohexanone with dimethylamine in the presence of catalytic amount of


an acid forms a compound if water during the reaction is continuously removed. The
compound formed is generally known as
(1) a Schiff’s base (2) an enamine
(3) an imine (4) an amine
145. (2)
CH3
N
CH3

146. p-cresol reacts with chloroform in alkaline medium to give the compound A which
adds hydrogen cyanide to form, the compound B. The latter on acidic hydrolysis
gives chiral carboxylic acid. The structure of the carboxylic acid is
CH3 CH3

CH(OH)COOH

(1) (2)
CH(OH)COOH

OH OH

FIITJEE Ltd. ICES House, 29-A, Kalu Sarai, Sarvapriya Vihar, New Delhi - 110016, Ph : 26515949, 26569493, Fax : 26513942
–13–

CH3 CH3

CH2COOH

(3) (4)
CH2COOH

OH OH
146. (2)
CH3 CH3 CH3

→
CHCl3
+ OH− 
OH



HCN

CHCl2 CHO
OH O O
A
CH3 CH3

+
H3 O
 →
OH OH
CH CH
OH CN OH COOH

147. An organic compound having molecular mass 60 is found to contain C = 20%, H =


6.67% and N = 46.67% while rest is oxygen. On heating it gives NH3 alongwith a
solid residue. The solid residue give violet colour with alkaline copper sulphate
solution. The compound is
(1) CH3NCO (2) CH3CONH2
(3) (NH2)2CO (4) CH3CH2CONH2
147. (3)

148. If the bond dissociation energies of XY, X2 and Y2 (all diatomic molecules) are in the
ratio of 1:1:0.5 and ∆ f H for the formation of XY is -200 kJ mole-1. The bond
dissociation energy of X2 will be
(1) 100 kJ mol-1 (2) 200 kJ mol-1
-1
(3) 300 kJ mol (4) 400 kJ mol-1
148. -
(None of the options is correct.)
XY → X(g) + Y(g) ; ∆H = +a kJ / mole ............(i)
X2 → 2X; ∆H = +a kJ / mole ...............(ii)
Y2 → 2Y; ∆H = +0.5 a kJ / mole ............(iii)
1 1
× (ii) + × (iii) − (i), Gives
2 2
1 1  a 0.5 
X2 + Y2 → XY; ∆H =  + + a − a  kJ / mole
2 2  2 2 
a 0.5 a
+ + − a = −200
2 2
a = 800.

FIITJEE Ltd. ICES House, 29-A, Kalu Sarai, Sarvapriya Vihar, New Delhi - 110016, Ph : 26515949, 26569493, Fax : 26513942
–14–

3
149. t1/4 can be taken as the time taken for the concentration of a reactant to drop to of
4
its initial value. If the rate constant for a first order reaction is K, the t1/4 can be
written as
(1) 0.10 / K (2) 0.29 / K
(3) 0.69 / K (4) 0.75 / K
149. (2)
2.303 1 0.29
t1/ 4 = log = .
K 1 K
1−
4

150. An amount of solid NH4HS is placed in a flask already containing ammonia gas at a
certain temperature and 0.50 atm. Pressure. Ammonium hydrogen sulphide
decomposes to yield NH3 and H2S gases in the flask. When the decomposition
reaction reaches equilibrium, the total pressure in the flask rises to 0.84 atm. The
equilibrium constant for NH4HS decomposition at this temperature is
(1) 0.30 (2) 0.18
(3) 0.17 (4) 0.11
150. (4)
NH4HS YZZZZZ
XZ NH3(g) + H2S(g)
a 0.5 atm
a−x 0.5 + x x
Total pressure = 0.5 + 2x = 0.84
i.e., x = 0.17
K p = pNH3 . pH2S
= (0.67). (0.17)
= 0.1139.

FIITJEE Ltd. ICES House, 29-A, Kalu Sarai, Sarvapriya Vihar, New Delhi - 110016, Ph : 26515949, 26569493, Fax : 26513942
-1-

FIITJEE
SOLUTION TO AIEEE-2005
PHYSICS

1. A projectile can have the same range R for two angles of projection. If t1 and t2 be the
times of flights in the two cases, then the product of the two time of flights is
proportional to
(1) R2 (2) 1/R2
(3) 1/R (4) R

1. (4)
2u2 sin 2θ 2R
t1 t 2 = =
g2 g

2. An annular ring with inner and outer radii R1 and R2 is rolling without slipping with a
uniform angular speed. The ratio of the forces experienced by the two particles
situated on the inner and outer parts of the ring, F1/F2 is
2
R2  R1 
(1) (2)  
R1  R2 
R1
(3) 1 (4)
R2
2. (4)
F1 R1ω2 R1
= =
F2 R 2 ω2 R 2

3. A smooth block is released at rest on a 45° incline and then slides a distance d. The
time taken to slide is n times as much to slide on rough incline than on a smooth
incline. The coefficient of friction is
1 1
(1) µk = 1 − (2) µk = 1 −
n2 n2
1 1
(3) µ s = 1 − (4) µ s = 1 −
n2 n2

3. (1)
1 g 2
d= t1
2 2
1 g
d= (1 − µk ) t 22
2 2
t 22 1
= n2 =
t1 2
1 − µk

4. The upper half of an inclined plane with inclination φ is perfectly smooth while the
lower half is rough. A body starting from rest at the top will again come to rest at the
bottom if the coefficient of friction for the lower half is given by
-2-

(1) 2sinφ (2) 2cosφ


(3) 2tanφ (4) tanφ

4. (3)
s S/2
mg s sin φ = µmgcos φ.
2
S/2
φ
5. A bullet fired into a fixed target loses half of its velocity after penetrating 3 cm. How
much further it will penetrate before coming to rest assuming that it faces constant
resistance to motion?
(1) 3.0 cm (2) 2.0 cm
(3) 1.5 cm (4) 1.0 cm

5. (4)
1 1 v2
F.3 = mv 2 − m
2 2 4
1
F(3+x) = mv 2
2
x = 1 cm

6. Out of the following pair, which one does NOT have identical dimensions is
(1) angular momentum and Planck’s constant
(2) impulse and momentum
(3) moment of inertia and moment of a force
(4) work and torque

6. (3)
Using dimension

7. The relation between time t and distance x is t=ax2+bx where a and b are constants.
The acceleration is
(1) −2abv2 (2) 2bv3
(3) −2av3 (4) 2av2

7. (3)
t = ax2 + bx
by differentiating acceleration = - 2av3

8. A car starting from rest accelerates at the rate f through a distance S, then continues
at constant speed for time t and then decelerates at the rate f/2 to come to rest. If the
total distance traversed is 15 S, then
(1) S=ft (2) S = 1/6 ft2
2
(3) S = 1/2 ft (4) S = 1/4 ft2

8. (none) V
ft 2 V0
S= 1
2
v 0 = 2Sf
t1 t2 t
During retardation
S2 = 2S
-3-

During constant velocity


15S -3S = 12S = vot
ft 2
⇒S=
72

9. A particle is moving eastwards with a velocity of 5 m/s in 10 seconds the velocity


changes to 5 m/s northwards. The average acceleration in this time is
1 1
(1) m / s2 towards north-east (2) m / s2 towards north.
2 2
1
(3) zero (4) m / s2 towards north-west
2

9. (4) r (N)
r ĵ
r Vf − Vi
a= î
t (W) (E)
5ˆj − 5iˆ 1 ˆ ˆ
=
10
=
2
( j−i ) (S)

1
∴a = ms−2 towards north west
2

10. A parachutist after bailing out falls 50 m without friction. When parachute opens, it
decelerates at 2 m/s2. He reaches the ground with a speed of 3 m/s. At what height,
did he bail out?
(1) 91 m (2) 182 m
(3) 293 m (4) 111 m

10. (3)
 32 − ( 2 × 10 × 50 ) 
s = 50 +  

 2 ( −2 ) 
= 293 m.

11. A block is kept on a frictionless inclined surface with angle


of inclination α. The incline is given an acceleration a to
keep the block stationary. Then a is equal to a
(1) g/tanα (2) g cosecα
α
(3) g (4) g tanα

11. (4) ma
N
α
mg sinα = ma cos α
∴ a = g tan α
mg

12. A spherical ball of mass 20 kg is stationary at the top of a hill of height 100 m. It rolls
down a smooth surface to the ground, then climbs up another hill of height 30 m and
finally rolls down to a horizontal base at a height of 20 m above the ground. The
velocity attained by the ball is
(1) 40 m/s (2) 20 m/s
(3) 10 m/s (4) 10 30 m/s
-4-

12. (1)
mgh = ½ mv2
v = 2gh
= 2 × 10 × 80 = 40 m/s

13. A body A of mass M while falling vertically downwards under gravity breaks into two
parts; a body B of mass 1/3 M and a body C of mass 2/3 M. The centre of mass of
bodies B and C taken together shifts compared to that of body A towards
(1) depends on height of breaking (2) does not shift
(3) body C (4) body B

13. (2)
No horizontal external force is acting
∴ acm = 0
since v cm = 0
∴ ∆x cm = 0

14. The moment of inertia of a uniform semicircular disc of mass M and radius r about a
line perpendicular to the plane of the disc through the centre is
1 2 2 2
(1) Mr (2) Mr
4 5
1
(3) Mr2 (4) Mr 2
2
14. (4)
R2
2I = 2M
2
MR 2
∴I =
2

15. A particle of mass 0.3 kg is subjected to a force F=−kx with k=15 N/m. What will be
its initial acceleration if it is released from a point 20 cm away from the origin?
(1) 3 m/s2 (2) 15 m/s2
2
(3) 5 m/s (4) 10 m/s2

15. (4)
kx
a= = 10m / s2
m

16. The block of mass M moving on the frictionless


M
horizontal surface collides with a spring of spring
constant K and compresses it by length L. The
maximum momentum of the block after collision is
KL2
(1) MK L (2)
2M
ML2
(3) zero (4)
K
16. (1)
1 2 P2
KL = ∴ P = MK L
2 2m
-5-

17. A mass ‘m’ moves with a velocity v and collides


v/√3
inelastically with another identical mass. After
m
collision the 1st mass moves with velocity v/√3 in a v
m

direction perpendicular to the initial direction of Before After


motion. Find the speed of the 2nd mass after collision collision

collision
(1) v (2) √3 v
(3) 2v/√3 (4) v/√3

17. (3) v
mv = mv1 cos θ 3
m
mv v
m
0= − mv1 sin θ θ
3 V1
2 Before After
∴ v1 = v collision collision
3

18. A 20 cm long capillary tube is dipped in water. The water rises up to 8 cm. If the
entire arrangement is put in a freely falling elevator the length of water column in the
capillary tube will be
(1) 8 cm (2) 10 cm
(3) 4 cm (4) 20 cm

18. (4)
Water will rise to the full length of capillary tube

19. If S is stress and Y is Young’s modulus of material of a wire, the energy stored in the
wire per unit volume is
(1) 2S2Y (2) S2/2Y
2
(3) 2Y/S (4) S/2Y

19. (2)
1 S2
U= stress × strain =
2 2Y

20. Average density of the earth


(1) does not depend on g (2) is a complex function of g
(3) is directly proportional to g (4) is inversely proportional to g

20. (3)
G4π
g= ρav R
3

21. A body of mass m is accelerated uniformly from rest to a speed v in a time T. The
instantaneous power delivered to the body as a function time is given by
mv 2 mv 2 2
(1) ⋅ t (2) ⋅t
T2 T2
1 mv 2 1 mv 2 2
(3) ⋅ t (4) ⋅t
2 T2 2 T2
21. (1)
P= (ma).v
-6-

= m a2 t
V2
=m 2 t
T

22. Consider a car moving on a straight road with a speed of 100 m/s. The distance at
which car can be stopped is [µk = 0.5]
(1) 800 m (2) 1000 m
(3) 100 m (4) 400 m

22. (2)
1
µk mgs = mu2
2
u2
s= = 1000m
2µk g

23. Which of the following is incorrect regarding the first law of thermodynamics?
(1) It is not applicable to any cyclic process
(2) It is a restatement of the principle of conservation of energy
(3) It introduces the concept of the internal energy
(4) It introduces the concept of the entropy

23. (none)
More than one statements are incorrect

24. A ‘T’ shaped object with dimensions shown in l

the figure, is lying on a smooth floor. A force F is A B


applied at the point P parallel to AB, such that r
P
2l
the object has only the translational motion F
without rotation. Find the location of P with
respect to C C

2 3
(1) l (2) l
3 2
4
(3) l (4) l
3

24. (3)
P will be the centre of mass of system

25. The change in the value of g at a height ‘h’ above the surface of the earth is the
same as at a depth ‘d’ below the surface of earth. When both ‘d’ and ‘h’ are much
smaller than the radius of earth, then which one of the following is correct?
h 3h
(1) d = (2) d =
2 2
(3) d = 2h (4) d = h

25. (3)
GM GM
= (R − d)
(R + h )
2
R3
⇒ d = 2h
-7-

26. A particle of mass 10 g is kept on the surface of a uniform sphere of mass 100 kg
and radius 10 cm. Find the work to be done against the gravitational force between
them to take the particle far away from the sphere
(you may take G = 6 . 67 × 10-11 Nm2 / kg2)
(1) 13.34 × 10 −10 J (2) 3.33 × 10 −10 J
(3) 6.67 × 10 −9 J (4) 6.67 × 10−10 J

26. (4)
w = GMm / R = 6.67 × 10-10 J

CP
27. A gaseous mixture consists of 16 g of helium and 16 g of oxygen. The ratio of the
Cv
mixture is
(1) 1.59 (2) 1.62
(3) 1.4 (4) 1.54

27. (2)
n1c v1 + n2 c v 2 29R
cv = =
n1 + n2 18
47R cP
cP = , = 1.62
18 cv

28. The intensity of gamma radiation from a given source is I. On passing through 36 mm
I
of lead, it is reduced to . The thickness of lead which will reduce the intensity to
8
I
will be
2
(1) 6 mm (2) 9 mm
(3) 18 mm (4) 12 mm

28. (4)
Use I = I0 e-µx

29. The electrical conductivity of a semiconductor increases when electromagnetic


radiation of wavelength shorter than 2480 nm is incident on it. The band gap in (eV)
for the semiconductor is
(1) 1.1 eV (2) 2.5 eV
(3) 0.5 eV (4) 0.7 eV

29. (3)
hc
Eg = = 0.5 eV
λ

30. A photocell is illuminated by a small bright source placed 1 m away. When the same
1
source of light is placed m away, the number of electrons emitted by photo
2
cathode would
(1) decrease by a factor of 4 (2) increase by a factor of 4
(3) decrease by a factor of 2 (4) increase by a factor of 2
-8-

30. (2)
1
I∝
r2
66
31 Starting with a sample of pure Cu, 7/8 of it decays into Zn in 15 minutes. The
corresponding half-life is
(1) 10 minutes (2) 15 minutes
1
(3) 5 minutes (4) 7 minutes
2

31. (3)
N0 t1/ 2 N0 t1/ 2 N0
N0 
t1/ 2
→  → 

2 4 8
3t1/ 2 = 15 ∴ t1/ 2 = 5

32. If radius of 27
13
Al nucleus is estimated to be 3.6 Fermi then the radius 125
52 Te nucleus
be nearly
(1) 6 fermi (2) 8 fermi
(3) 4 fermi (4) 5 fermi

32. (1)
1
R  125  3
= ⇒ R = 6 fermi
3.6  27 

33. The temperature-entropy diagram of a reversible engine T


cycle is given in the figure. Its efficiency is
(1) 1/2 (2) 1/4 2To

(3) 1/3 (4) 2/3


To

S
S 2So

33. (3) T
B
S0 T0 2T0
∆W
η= = 2 = 1/ 3
QBC 3S0 T0
T0 A C
2
S0 2S0 S

34. The figure shows a system of two concentric spheres of radii


r1 and r2 and kept at temperatures T1 and T2 respectively. The
radial rate of flow of heat in a substance between the two r1
T1
concentric sphere is proportional to r2
T2

r2 − r1 r 
(1) (2) ln  2 
r1r2  r1 
r1r2
(3) (4) ln ( r2 − r1 )
r2 − r1
-9-

34. (3)
 dQ  4πr1r2K
 dt  = ( T1 − T2 ) r − r
  ( 2 1)
35. A system goes from A to B via two processes I and II as shown
II
in the figure. If ∆U1 and ∆U2 are the changes in internal energies P
in the processes I and II respectively, the
(1) ∆U1 = ∆U2 A B
I
(2) relation between ∆U1 and ∆U2 can not be determined
(3) ∆U2 > ∆U1 V
(4) ∆U2 < ∆U1

35. (1)
Internal energy is state function

36. The function sin2(ωt) represents


(1) a periodic, but not simple harmonic motion with a period 2π/ω
(2) a periodic, but not simple harmonic motion with a period π/ω
(3) a simple harmonic motion with a period 2π/ω
(4) a simple harmonic motion with a period π/ω.

36. (4)

y=
(1 − cos 2ωt )
2

37. A Young’s double slit experiment uses a monochromatic source. The shape of the
interference fringes formed on a screen is
(1) hyperbola (2) circle
(3) straight line (4) parabola

37. (3)
Straight line
Note: If instead of young’s double slit experiment, young’s double hole experiment
was given shape would have been hyperbola.

38. Two simple harmonic motions are represented by the equation y1 = 0.1
 π
sin  100πt +  and y2 = 0.1 cosπt. The phase difference of the velocity of particle 1
 3 
w.r.t. the velocity of the particle 2 is
(1) −π/6 (2) π/3
(3) −π/3 (4) π/6

38. (1)
Phase difference (φ) = 99πt + π/3 −π/2
at t = 0 φ = −π/6.

39. A fish looking up through the water sees the outside world contained in a circular
horizon. If the refractive index of water is 4/3 and the fish is 12 cm below the surface,
the radius of this circle in cm is
(1) 36 7 (2) 36 / 7
(3) 36 5 (4) 4 5
-10-

39. (2)
h 36
r= =
µ −1 2
7

40. Two point white dots are 1 mm apart on a black paper. They are viewed by eye of
pupil diameter 3 mm. Approximately, what is the maximum distance at which these
dots can be resolved by the eye? [ Take wavelength of light = 500 nm ]
(1) 5 m (2) 1m
(3) 6 m (4) 3m

40. (1)
1.22λ (1mm )
= Re solution limit =
( 3mm ) R
∴R=5m

41. A thin glass (refractive index 1.5) lens has optical power of – 5D in air. Its optical
power in a liquid medium with refractive index 1.6 will be
(1) 1 D (2) -1D
(3) 25 D (4) – 25 D

41. (none)
 µl 
 − 1
Pm µ
=  a 
Pair  µ l 
 − 1
µ
 m 
Pm=5/8 D

42. The diagram shows the energy levels for an electron n=4
in a certain atom. Which transition shown represents n=3
the emission of a photon with the most energy ?
(1) III (2) IV n=2
(3) I (4) II

n=1
I II III IV
42. (1)
 1 1
∆E ∝  2
− 2
 n1 n2 

43. If the kinetic energy of a free electron doubles. Its deBroglie wavelength changes by
the factor
1
(1) (2) 2
2
1
(3) (4) 2
2
43. (3)
h
λ=
2Km
-11-

44. In a common base amplifier, the phase difference between the input signal voltage
and output voltage is
π
(1) (2) π
4
π
(3) 0 (4)
2

44. (3)
No phase difference between input and output signal.

45. In a full wave rectifier circuit operating from 50 Hz mains frequency, the fundamental
frequency in the ripple would be
(1) 50 Hz (2) 25 Hz
(3) 100 Hz (4) 70.7 Hz

45. (3)
frequency = 2 (frequency of input signal).

46. A nuclear transformation is denoted by X(n, α) 73 Li . Which of the following is the


nucleus of element X ?
(1) 12C6 (2) 105 B
(3) 95 B (4) 114Be

46. (2)
X +10 n →24 He + 73 Li

47. A moving coil galvanometer has 150 equal divisions. Its current sensitivity is
10 divisions per milliampere and voltage sensitivity is 2 divisions per millivolt. In
order that each division reads 1 volt, the resistance in ohms needed to be connected
in series with the coil will be
(1) 103 (2) 105
(3) 99995 (4) 9995

47. (4)
Ig=15mA Vg = 75mV
V Vg
R= −
Ig Ig

48. Two voltameters one of copper and another of silver, are joined in parallel. When a
total charge q flows through the voltameters, equal amount of metals are deposited.
If the electrochemical equivalents of copper and silver are z1 and z2 respectively the
charge which flows through the silver voltameter is
q q
(1) (2)
z z
1+ 1 1+ 2
z2 z1
z1 z2
(3) q (4) q
z2 z1

48. (2)
q1Z1=q2Z2
q=q1+q2
-12-

q
∴ q2 =
Z
1+ 2
Z1
49. In the circuit, the galvanometer G shows 500Ω
G
zero deflection. If the batteries A and B
have negligible internal resistance, the R 2V

value of the resistor R will be B 12V A

(1) 200 Ω (2) 100 Ω


(3) 500 Ω (4) 1000 Ω

49. (2)
12R
=2
500 + R

50. Two sources of equal emf are connected to an external resistance R. The internal
resistance of the two sources are R1 and R2 (R2 > R1). If the potential difference
across the source having internal resistance R2 is zero, then
(1) R = R2 × (R1 + R2)/R2 – R1) (2) R = R2 – R1
(3) R = R1R2 / (R1 + R2) (4) R = R1R2 / (R2 – R1)

50. (2)
2E
I=
R1 + R2 + R
E − R2I = 0
⇒ R = R2−R1

51. A fully charged capacitor has a capacitance ‘C’ it is discharged through a small coil of
resistance wire embedded in a thermally insulated block of specific heat capacity ‘s’
and mass ‘m’. If the temperature of the block is raised by ‘∆T’. The potential
difference V across the capacitance is
2mC∆T mC∆T
(1) (2)
s s
ms∆T 2ms∆T
(3) (4)
C C

51. (4)
Dimensionally only 4th option is correct.

52. One conducting U tube can slide inside another as x x x x


A B
shown in figure, maintaining electrical contacts
x x x x
between the tubes. The magnetic field B is V V
perpendicular to the plane of the figure. if each tube x x x C x
D
moves towards the other at a constant speed V, x D x x
then the emf induced in the circuit in terms of B, l
and V where l is the width of each tube will be
(1) BlV (2) – BlV
(3) zero (4) 2 BlV

52. (4)

= 2Blv
dt
-13-

53. A heater coil is cut into two equal parts and only one part is now used in the heater.
The heat generated will now be
(1) doubled (2) four times
(3) one fourth (4) halved

53. (1)
V 2 ∆t
H=
R
V2
H ' = ' ∆t Given R’ = R/2
R

54. Two thin, long parallel wires separated by a distance ‘d’ carry a current of ‘i’ A in the
same direction. They will
(1) attract each other with a force of µ0i2/(2πd)
(2) repel each other with a force of µ0i2 / (2πd)
(3) attract each other with a force of µ0i2(2πd2)
(4) repel each other with a force of µ0i2/(2πd2)

54. (1)
Using the definition of force per unit length due to two long parallel wires carrying
currents.

55. When an unpolarized light of intensity I0 is incident on a polarizing sheet, the intensity
of the light which does not get transmitted is
1 1
(1) I0 (2) I0
2 4
(3) zero (4) I0

55. (1)
When unpolarised light of intensity Io is incident on a polarizing sheet, only Io/2 is
transmitted.

56. A charged ball B hangs from a silk thread S which makes an +


angle θ with a large charged conducting sheet P, as show in +
the figure. The surface charge density σ of the sheet is P +
proportional to + θ
S
(1) cos θ (2) cot θ +
(3) sin θ (4) tan θ +
+ B

56. (4)
qσ θ
tanθ =
( o ) mg


2εo

Mg

57. Two point charges + 8q and – 2q are located at x = 0 and x = L respectively. The
location of a point on the x axis at which the net electric field due to these two point
charges is zero is
(1) 2L (2) L/4
-14-

(3) 8L (4) 4L

57. (1)
k2q k8q
− + =0
( x − L)
2
x2
⇒ x = 2L

58. Two thin wires rings each having a radius R are placed at a distance d apart with
their axes coinciding. The charges on the two rings are +q and –q. The potential
difference between the centres of the two rings is
Q 1 1 
(1) QR/4πε0d2 (2)  − 
2πε0  R R + d2
2

Q 1 1 
(3) zero (4)  − 
4πε0  R R + d2
2

58. (2)
kq kq
v1 = −
R R2 + d2
−kq kq
v2 = +
R R + d2
2

59. A parallel plate capacitor is made by stacking n equally spaced plates connected
alternatively. If the capacitance between any two adjacent plates is C then the
resultant capacitance is
(1) (n − 1)C (2) (n + 1)C
(3) C (4) nC

59. (1)
Ceq=(n−1) C (Q all capacitors are in parallel)

60. When two tuning forks (fork 1 and fork 2) are sounded simultaneously, 4 beats per
second are heard. Now, some tape is attached on the prong of the fork 2. When the
tuning forks are sounded again, 6 beats per seconds are heard. If the frequency of
fork 1 is 200 Hz, then what was the original frequency of fork 2?
(1) 200 Hz (2) 202 Hz
(3) 196 Hz (4) 204 Hz

60. (3)
|f1−f2| =4
Since mass of second tuning fork increases so f2 decrease and beats increase so
f1>f2
⇒ f2=f1−4 = 196

d2 x
61. If a simple harmonic motion is represented by + αx = 0, its time period is
dt 2
2π 2π
(1) (2)
α α
(3) 2πα (4) 2π α
-15-

61. (2)
ω2=α
ω = √α

T=
α

62. The bob of a simple pendulum is a spherical hollow ball filled with water. A plugged
hole near the bottom of the oscillation bob gets suddenly unplugged. During
observation, till water is coming out, the time period of oscillation would
(1) first increase and then decrease to the original value.
(2) first decreased then increase to the original value.
(3) remain unchanged.
(4) increase towards a saturation value.

62. (1)
First CM goes down and then comes to its initial position.

63. An observer moves towards a stationary source of sound, with a velocity one fifth of
the velocity of sound. What is the percentage increase in the apparent frequency?
(1) zero. (2) 0.5%
(3) 5% (4) 20%

63. (4)
v + v /5 6f
f= f=
v 5
% increase in frequency = 20%

64. If I0 is the intensity of the principal maximum in the single slit diffraction pattern, then
what will be its intensity when the slit width is doubled?
(1) 2I0 (2) 4I0
(3) I0 (4) I0/2

64. (3)
Maximum intensity is independent of slit width.

65. Two concentric coils each of radius equal to 2π cm are placed at right angles to each
other. 3 Ampere and 4 ampere are the currents flowing in each coil respectively. The
magnetic induction in Weber/m2 at the centre of the coils will be (µ0 = 4π × 10−7
Wb/A-m)
(1) 12 × 10−5 (2) 10−5
(3) 5 × 10−5 (4) 7 × 10−5

65. (3)
µo 2 2
B= I1 + I2
2r
4π × 10 −7
B= ×5
2 × 2π × 10 −2
B = 5×10−5

66. A coil of inductance 300 mH and resistance 2Ω is connected to a source of voltage


2V. The current reaches half of its steady state value in
(1) 0.05 s (2) 0.1 s
-16-

(3) 0.15 s (4) 0.3 s

66. (2)
 − t 
R

I = Io  1 − e L 
 
R
0.693 = t
L
.3 × 0.693
t= = 0.1sec
2

67. The self inductance of the motor of an electric fan is 10 H. In order to impart
maximum power at 50 Hz, it should be connected to a capacitance of
(1) 4µF (2) 8µF
(3) 1µF (4) 2µF

67. (3)
1
f=
2π LC
1
C=
4 × π2 f 2 × 10
C = 1µF

68. An energy source will supply a constant current into the load of its internal resistance
is
(1) equal to the resistance of the load.
(2) very large as compared to the load resistance.
(3) zero.
(4) non-zero but less than the resistance of the load.

68. (2)
Eo E
I= if R << r
R+r r

69. A circuit has a resistance of 12 Ω and an impedance of 15 Ω. The power factor of the
circuit will be
(1) 0.8 (2) 0.4
(3) 1.25 (4) 0.125

69. (1)
R 12 4
cos φ = = = = 0.8
Z 15 5

70. The phase difference between the alternating current and emf is π/2. Which of the
following cannot be the constituent of the circuit?
(1) C alone (2) R.L
(3) L. C (4) L alone

70. (2)
0<phase difference for R-L circuit < π/2
-17-

71. A uniform electric field and a uniform magnetic field are acting along the same
direction in a certain region. If an electron is projected along the direction of the fields
with a certain velocity then
(1) its velocity will decrease. (2) its velocity will increase.
(3) it will turn towards right of direction of motion. (4) it will turn towards left of
direction of motion.

71. (1)
r r r r r
F = −e E + v × B  = −eE
r
r eE
a=−
m
eE
v ( t ) = vo − t
m

72. A charged particle of mass m and charge q travels on a circular path of radius r that
is perpendicular to a magnetic field B. The time taken by the particle to complete one
revolution is
2πmq 2πq2 B
(1) (2)
B m
2πqB 2πm
(3) (4)
m qB

72. (4)
mω2r=Bqωr
ω = Bq/m
2πm
T=
qB

73. In a potentiometer experiment the balancing with a cell is at length 240 cm. On
shunting the cell with a resistance of 2Ω the balancing length becomes 120 cm. The
internal resistance of the cell is
(1) 1 Ω (2) 0.5 Ω
(3) 4 Ω (4) 2 Ω

73. (4)
l   240 
r = R  1 − 1 = 2  − 1 = 2Ω
l
 2   120 

74. The resistance of hot tungsten filament is about 10 times the cold resistance. What
will be the resistance of 100 W and 200 V lamp when not in use?
(1) 40 Ω (2) 20 Ω
(3) 400 Ω (4) 200 Ω

74. (1)
V 2 200 × 200
Rhot = = = 400Ω
P 100
cold resistance = Rhot/10 = 400/10 = 40 Ω

75. A magnetic needle is kept in a non-uniform magnetic field. It experiences


(1) a torque but not a force (2) neither a force nor a torque
(3) a force and a torque. (4) a force but not a torque.
-18-

75. (3)
In non uniform magnetic field, dipole experiences both force and torque.
https://www.gofacademy.in AIEEE Old Question Paper by GOF Academy
https://www.gofacademy.in AIEEE Old Question Paper by GOF Academy
https://www.gofacademy.in AIEEE Old Question Paper by GOF Academy
https://www.gofacademy.in AIEEE Old Question Paper by GOF Academy
https://www.gofacademy.in AIEEE Old Question Paper by GOF Academy
https://www.gofacademy.in AIEEE Old Question Paper by GOF Academy
https://www.gofacademy.in AIEEE Old Question Paper by GOF Academy
https://www.gofacademy.in AIEEE Old Question Paper by GOF Academy
https://www.gofacademy.in AIEEE Old Question Paper by GOF Academy
https://www.gofacademy.in AIEEE Old Question Paper by GOF Academy
https://www.gofacademy.in AIEEE Old Question Paper by GOF Academy
https://www.gofacademy.in AIEEE Old Question Paper by GOF Academy
https://www.gofacademy.in AIEEE Old Question Paper by GOF Academy
https://www.gofacademy.in AIEEE Old Question Paper by GOF Academy
https://www.gofacademy.in AIEEE Old Question Paper by GOF Academy
https://www.gofacademy.in AIEEE Old Question Paper by GOF Academy
https://www.gofacademy.in AIEEE Old Question Paper by GOF Academy
https://www.gofacademy.in AIEEE Old Question Paper by GOF Academy
https://www.gofacademy.in AIEEE Old Question Paper by GOF Academy
https://www.gofacademy.in AIEEE Old Question Paper by GOF Academy
https://www.gofacademy.in AIEEE Old Question Paper by GOF Academy
https://www.gofacademy.in AIEEE Old Question Paper by GOF Academy
https://www.gofacademy.in AIEEE Old Question Paper by GOF Academy
https://www.gofacademy.in AIEEE Old Question Paper by GOF Academy
https://www.gofacademy.in AIEEE Old Question Paper by GOF Academy
https://www.gofacademy.in AIEEE Old Question Paper by GOF Academy
https://www.gofacademy.in AIEEE Old Question Paper by GOF Academy
https://www.gofacademy.in AIEEE Old Question Paper by GOF Academy
https://www.gofacademy.in AIEEE Old Question Paper by GOF Academy
https://www.gofacademy.in AIEEE Old Question Paper by GOF Academy
https://www.gofacademy.in AIEEE Old Question Paper by GOF Academy
https://www.gofacademy.in AIEEE Old Question Paper by GOF Academy
https://www.gofacademy.in AIEEE Old Question Paper by GOF Academy
https://www.gofacademy.in AIEEE Old Question Paper by GOF Academy
https://www.gofacademy.in AIEEE Old Question Paper by GOF Academy
https://www.gofacademy.in AIEEE Old Question Paper by GOF Academy
https://www.gofacademy.in AIEEE Old Question Paper by GOF Academy
https://www.gofacademy.in AIEEE Old Question Paper by GOF Academy
https://www.gofacademy.in AIEEE Old Question Paper by GOF Academy
https://www.gofacademy.in AIEEE Old Question Paper by GOF Academy
https://www.gofacademy.in AIEEE Old Question Paper by GOF Academy
https://www.gofacademy.in AIEEE Old Question Paper by GOF Academy
https://www.gofacademy.in AIEEE Old Question Paper by GOF Academy
https://www.gofacademy.in AIEEE Old Question Paper by GOF Academy
https://www.gofacademy.in AIEEE Old Question Paper by GOF Academy
https://www.gofacademy.in AIEEE Old Question Paper by GOF Academy
https://www.gofacademy.in AIEEE Old Question Paper by GOF Academy
https://www.gofacademy.in AIEEE Old Question Paper by GOF Academy
https://www.gofacademy.in AIEEE Old Question Paper by GOF Academy
https://www.gofacademy.in AIEEE Old Question Paper by GOF Academy
https://www.gofacademy.in AIEEE Old Question Paper by GOF Academy
https://www.gofacademy.in AIEEE Old Question Paper by GOF Academy
https://www.gofacademy.in AIEEE Old Question Paper by GOF Academy
https://www.gofacademy.in AIEEE Old Question Paper by GOF Academy
https://www.gofacademy.in AIEEE 2008 Old Question Paper by GOF Academy
https://www.gofacademy.in AIEEE 2008 Old Question Paper by GOF Academy
https://www.gofacademy.in AIEEE 2008 Old Question Paper by GOF Academy
https://www.gofacademy.in AIEEE 2008 Old Question Paper by GOF Academy
https://www.gofacademy.in AIEEE 2008 Old Question Paper by GOF Academy
https://www.gofacademy.in AIEEE 2008 Old Question Paper by GOF Academy
https://www.gofacademy.in AIEEE 2008 Old Question Paper by GOF Academy
https://www.gofacademy.in AIEEE 2008 Old Question Paper by GOF Academy
https://www.gofacademy.in AIEEE 2008 Old Question Paper by GOF Academy
https://www.gofacademy.in AIEEE 2008 Old Question Paper by GOF Academy
https://www.gofacademy.in AIEEE 2008 Old Question Paper by GOF Academy
https://www.gofacademy.in AIEEE 2008 Old Question Paper by GOF Academy
https://www.gofacademy.in AIEEE 2008 Old Question Paper by GOF Academy
https://www.gofacademy.in AIEEE 2008 Old Question Paper by GOF Academy
https://www.gofacademy.in AIEEE 2008 Old Question Paper by GOF Academy
https://www.gofacademy.in AIEEE 2008 Old Question Paper by GOF Academy
https://www.gofacademy.in AIEEE 2008 Old Question Paper by GOF Academy
https://www.gofacademy.in AIEEE 2008 Old Question Paper by GOF Academy
https://www.gofacademy.in AIEEE 2008 Old Question Paper by GOF Academy
https://www.gofacademy.in AIEEE 2008 Old Question Paper by GOF Academy
https://www.gofacademy.in AIEEE 2008 Old Question Paper by GOF Academy
https://www.gofacademy.in AIEEE 2008 Old Question Paper by GOF Academy
https://www.gofacademy.in AIEEE 2008 Old Question Paper by GOF Academy
https://www.gofacademy.in AIEEE 2008 Old Question Paper by GOF Academy
https://www.gofacademy.in AIEEE 2008 Old Question Paper by GOF Academy
https://www.gofacademy.in AIEEE 2008 Old Question Paper by GOF Academy
https://www.gofacademy.in AIEEE Old Question Paper by GOF Academy
https://www.gofacademy.in AIEEE Old Question Paper by GOF Academy
https://www.gofacademy.in AIEEE Old Question Paper by GOF Academy
https://www.gofacademy.in AIEEE Old Question Paper by GOF Academy
https://www.gofacademy.in AIEEE Old Question Paper by GOF Academy
https://www.gofacademy.in AIEEE Old Question Paper by GOF Academy
https://www.gofacademy.in AIEEE Old Question Paper by GOF Academy
https://www.gofacademy.in AIEEE Old Question Paper by GOF Academy
https://www.gofacademy.in AIEEE Old Question Paper by GOF Academy
https://www.gofacademy.in AIEEE Old Question Paper by GOF Academy
https://www.gofacademy.in AIEEE Old Question Paper by GOF Academy
https://www.gofacademy.in AIEEE Old Question Paper by GOF Academy
https://www.gofacademy.in AIEEE Old Question Paper by GOF Academy
https://www.gofacademy.in AIEEE Old Question Paper by GOF Academy
https://www.gofacademy.in AIEEE Old Question Paper by GOF Academy
https://www.gofacademy.in AIEEE Old Question Paper by GOF Academy
https://www.gofacademy.in AIEEE Old Question Paper by GOF Academy
https://www.gofacademy.in AIEEE Old Question Paper by GOF Academy
https://www.gofacademy.in AIEEE Old Question Paper by GOF Academy
https://www.gofacademy.in AIEEE Old Question Paper by GOF Academy
https://www.gofacademy.in AIEEE Old Question Paper by GOF Academy
https://www.gofacademy.in AIEEE Old Question Paper by GOF Academy
https://www.gofacademy.in AIEEE Old Question Paper by GOF Academy
https://www.gofacademy.in AIEEE Old Question Paper by GOF Academy
https://www.gofacademy.in AIEEE Old Question Paper by GOF Academy
https://www.gofacademy.in AIEEE Old Question Paper by GOF Academy
https://www.gofacademy.in AIEEE Old Question Paper by GOF Academy
https://www.gofacademy.in AIEEE Old Question Paper by GOF Academy
AIEEE–2010
IMPORTANT INSTRUCTIONS

! " #
!

$ $

% &' ( ) %$

* ( +

,- . %% /,0 1 % & $ $' 23 4 .% /


0 1 $ ' 5 6 7 .8 /

, . %% /,0 1 $$ %& *% 9' 23 4 .% /


0 1 $ $ *' *$ 5 6 7 .8 /

- ,: . %% /,0 1 9 9&# ;$ 8 8* &' 23 4


.% / 0 1 ;' ; 8 8% 5 6 7 .8 /

9 - < 1 *
( = . / ( 1

; ! > ! >
!

8 1 ) # # # #
# < # # ) - )

& 4 <
< % . ' , $/

' 3 # < ! <


4 7 7 < #

- 3 ?5 ? : - 3 ?5 ! > !
#
< !

? !

(Hyderabad Classes) Limited. 5-9-14/B, Saifabad, (Opp. Secretariat) Hyderabad. 500 063. Phone: 040-66777000 – 03 Fax: 040-66777004
−2010−
AIEEE− −2

–1
1. The standard enthalpy of formation of NH3 is –46.0 kJ mol . If the enthalpy of formation of H2 from
–1 –1
its atoms is –436 kJ mol and that of N2 is –712 kJ mol , the average bond enthalpy of N–H bond in
NH3 is
(1) –964 kJ mol–1 (2) +352 kJ mol–1 (3) + 1056 kJ mol–1 (4) –1102 kJ mol–1
1. (2)
Sol : Enthalpy of formation of NH3 = –46 kJ/mole
∴ N2 + 3H2 → 2NH3 ∆Hf = – 2 x 46 kJ mol
Bond breaking is endothermic and Bond formation is exothermic
–1
Assuming ‘x’ is the bond energy of N–H bond (kJ mol )
∴ 712 + (3 x 436)– 6x = –46 x 2
∴ x = 352 kJ/mol

2. The time for half life period of a certain reaction A → products is 1 hour. When the initial
–1
concentration of the reactant ‘A’, is 2.0 mol L , how much time does it take for its concentration to
–1
come from 0.50 to 0.25 mol L if it is a zero order reaction ?
(1) 4 h (2) 0.5 h (3) 0.25 h (4) 1 h
2. (3)
x
Sol : For a zero order reaction k = → (1)
t
Where x = amount decomposed
k = zero order rate constant
for a zero order reaction
[ A ]0
k= → (2)
2t1
2

Since [A0] = 2M , t1/2 = 1 hr; k = 1


∴ from equation (1)
0.25
t= = 0.25hr
1

3. A solution containing 2.675 g of CoCl3. 6 NH3 (molar mass = 267.5 g mol–1) is passed through a
cation exchanger. The chloride ions obtained in solution were treated with excess of AgNO3 to give
–1
4.78 g of AgCl (molar mass = 143.5 g mol ). The formula of the complex is (At. Mass of Ag = 108 u)
(1) [Co(NH3)6]Cl3 (2) [CoCl2(NH3)4]Cl (3) [CoCl3(NH3)3] (4) [CoCl(NH3)5]Cl2
3. (1)

Sol : CoCl3. 6NH3 → xCl  AgNO3
→ x AgCl ↓
n(AgCl) = x n(CoCl3. 6NH3)
4.78 2.675
=x ∴x=3
143.5 267.5
∴ The complex is Co (NH3 )6 Cl3

4. Consider the reaction :


+ –
Cl2(aq) + H2S(aq) → S(s) + 2H (aq) + 2Cl (aq)
The rate equation for this reaction is rate = k [Cl2] [H2S]
Which of these mechanisms is/are consistent with this rate equation ?
+ – + –
(A) Cl2 + H2 → H + Cl + Cl + HS (slow)
+ – + –
Cl + HS → H + Cl + S (fast)
+ –
(B) H2S ⇔ H + HS (fast equilibrium)
– – +
Cl2 + HS → 2Cl + H + S (slow)
(1) B only (2) Both A and B (3) Neither A nor B (4) A only
4. (4)

(Hyderabad Classes) Limited. 5-9-14/B, Saifabad, (Opp. Secretariat) Hyderabad. 500 063. Phone: 040-66777000 – 03 Fax: 040-66777004
−2010−
AIEEE− −3

Sol: Rate equation is to be derived wrt slow


Step ∴ from mechanism (A)
Rate = k[Cl2] [H2S]
–4 3 3
5. If 10 dm of water is introduced into a 1.0 dm flask to 300 K, how many moles of water are in the
vapour phase when equilibrium is established ?
(Given : Vapour pressure of H2O at 300 K is 3170 Pa ; R = 8.314 J K–1 mol–1)
–3 –2 –2 –3
(1) 5.56 x 10 mol (2) 1.53 x 10 mol (3) 4.46 x 10 mol (4) 1.27 x 10 mol
5. (4)
PV
Sol : n= =
RT
–5
= 128 x 10 moles
3170 × 10 −5 atm × 1 L –3
= ≈ 1.27 x 10 mol
0.0821 L atm k −1mol−1 × 300K

6. One mole of a symmetrical alkene on ozonolysis gives two moles of an aldehyde having a molecular
mass of 44 u. The alkene is
(1) propene (2) 1–butene (3) 2–butene (4) ethene
6. (3)
Sol : 2–butene is symmetrical alkene
CH3–CH=CH–CH3  O3
Zn / H2 O
→ 2.CH3 CHO
Molar mass of CH3CHO is 44 u.

7. If sodium sulphate is considered to be completely dissociated into cations and anions in aqueous
solution, the change in freezing point of water (∆Tf), when 0.01 mol of sodium sulphate is dissolved
in 1 kg of water, is (Kf = 1.86 K kg mol–1)
(1) 0.0372 K (2) 0.0558 K (3) 0.0744 K (4) 0.0186 K
7. (2)
Sol : Vant Hoff’s factor (i) for Na2SO4 = 3
∴ ∆Tf = (i) kf m
0.01
= 3 x 1.80 x = 0.0558 K
1

8. From amongst the following alcohols the one that would react fastest with conc. HCl and anhydrous
ZnCl2, is
(1) 2–Butanol (2) 2–Methylpropan–2–ol (3) 2–Methylpropanol (4) 1–Butanol
8. (2)
Sol : 3° alcohols react fastest with ZnCl2/conc.HCl due to formation of 3° carbocation and
∴ 2–methyl propan–2–ol is the only 3° alcohol

9. In the chemical reactions,


NH2

NaNO2 HBF4
A B
HCl, 278 K

the compounds ‘A’ and ‘B’ respectively are


(1) nitrobenzene and fluorobenzene (2) phenol and benzene
(3) benzene diazonium chloride and fluorobenzene (4) nitrobenzene and chlorobenzene
9. (3)
Sol :

(Hyderabad Classes) Limited. 5-9-14/B, Saifabad, (Opp. Secretariat) Hyderabad. 500 063. Phone: 040-66777000 – 03 Fax: 040-66777004
−2010−
AIEEE− −4

NH2 N2 Cl F

NaNO2 HBF4
N2 BF3 HCl
HCl, 278 K
(A) (B)
benzene diazonium fluorobenzene
chloride

10. 29.5 mg of an organic compound containing nitrogen was digested according to Kjeldahl’s method
and the evolved ammonia was absorbed in 20 mL of 0.1 M HCl solution. The excess of the acid
required 15 mL of 0.1 M NaOH solution for complete neutralization. The percentage of nitrogen in
the compound is
(1) 59.0 (2) 47.4 (3) 23.7 (4) 29.5
10. (3)
Sol : Moles of HCl reacting with
ammonia = (moles of HCl absorbed ) – (moles of NaOH solution required)
–3 –3
= (20 x 0.1 x 10 ) – (15 x 0.1 x 10 )
= moles of NH3 evolved.
= moles of nitrogen in organic compound
∴ wt. of nitrogen in org. comp = 0.5 x 10–3 x 14
–3
= 7 x 10 g
7 × 10−3
% wt = = 23.7%
29.5 × 10 −3
–1
11. The energy required to break one mole of Cl–Cl bonds in Cl2 is 242 kJ mol . The longest
wavelength of light capable of breaking a single Cl – Cl bond is
8 –1 23 –1
(c = 3 x 10 ms and NA = 6.02 x 10 mol )
(1) 594 nm (2) 640 nm (3) 700 nm (4) 494 nm
11. (4)
242 × 103
Sol : Energy required for 1 Cl2 molecule = Joules.
NA
This energy is contained in photon of wavelength ‘λ’.
hc 6.626 × 10−34 × 3 × 108 242 × 103
=E =
λ λ 6.022 × 1023
0
λ = 4947 A ≈ 494 nm
+ –18 –1 2+
12. Ionisation energy of He is 19.6 x 10 J atom . The energy of the first stationary state (n = 1) of Li
is
–16 –1 –17 –1
(1) 4.41 x 10 J atom (2) –4.41 x 10 J atom
–15 –1 –17 –1
(3) –2.2 x 10 J atom (4) 8.82 x 10 J atom
12. (2)
Sol :
He He
1
1 ∞
1
IE + = 13.6 Z 2 + 2 − 2 = 13.6Z2 + where Z + = 2
He He
( )
–1
Hence 13.6 × Z2 = 19.6 × 10 −18 J atom .
He+

1 Z 2 +2 9
(E1 )Li+2 = −13.6 Z 2
Li+2
× 2 = −13.6 Z + × Li2
2
= –19.6 x 10
–18
x = −4.41× 10−17 J / atom
1 He Z + 4
He

13. Consider the following bromides :

(Hyderabad Classes) Limited. 5-9-14/B, Saifabad, (Opp. Secretariat) Hyderabad. 500 063. Phone: 040-66777000 – 03 Fax: 040-66777004
−2010−
AIEEE− −5

Me Me
Me Br Me

Br Br
(A) (B) (C)
The correct order of SN1 reactivity is
(1) B > C > A (2) B > A > C (3) C > B > A (4) A > B > C
13. (1)
Sol : SN1 proceeds via carbocation intermediate, the most stable one forming the product faster. Hence
reactivity order for A, B, C depends on stability of carbocation created.
Me Me
> >
Me Me

14. Which one of the following has an optical isomer ?


2+ 3+ 3+ 2+
(1) Zn ( en )(NH3 )2 (2) Co ( en )3 (3) Co (H2 O )4 ( en ) (4) Zn ( en )2
(en = ethylenediamine)
14. (2)
Sol : Only option (2) is having non–super imposable mirror image & hence one optical isomer.
en en
( 1) ( 2)
en
+2
Zn
en +3 +3
NH3 Co Co en
NH3
no optical isomer. It is
Tetrahedral with a plane of symmetry

en en

optical isomer

H2O
3) 4) en
H 2O +2
Zn
+3
Co en
H2O en
H2O
no optical isomer, it is
Horizontal plane is plane of symmtry tetrahedral with a plane of symmetry

15. On mixing, heptane and octane form an ideal solution. At 373 K, the vapour pressures of the two
liquid components (heptane and octane) are 105 kPa and 45 kPa respectively. Vapour pressure of
the solution obtained by mixing 25.0g of heptane and 35 g of octane will be (molar mass of heptane
–1 –1
= 100 g mol an dof octane = 114 g mol ).
(1) 72.0 kPa (2) 36.1 kPa (3) 96.2 kPa (4) 144.5 kPa
15. (1)

(Hyderabad Classes) Limited. 5-9-14/B, Saifabad, (Opp. Secretariat) Hyderabad. 500 063. Phone: 040-66777000 – 03 Fax: 040-66777004
−2010−
AIEEE− −6

25 /100 0.25
Sol : Mole fraction of Heptane = = = 0.45
25 35 0.557
+
100 114
XHep tan e = 0.45 .
∴ Mole fraction of octane = 0.55 = Xoctane
Total pressure = XiPi0
= (105 x 0.45) + (45 x 0.55) kPa
= 72.0 KPa

16. The main product of the following reaction is C6H5CH2CH(OH)CH(CH3)2 


conc. H2 SO4
→ ?
H5C 6 H C6H5CH2 CH3
(1) C C (2) C C
H CH(CH3)2 H CH3
C6H 5 CH(CH3)2 H5C6CH2CH2
(3) C C (4) C CH2
H H H3C

16. (1)
Sol :
CH2 CH CH CH3
OH CH3

conc. H2SO4

CH2 CH CH CH3
CH3

loss of proton

CH3
CH CH HC (conjugated system)
CH3

Trans isomers is more stable & main product here


H CH(CH3) 2
C C (trans isomer)
H

17. Three reactions involving H2PO −4 are given below :


+ +
(i) H3PO4 + H2O → H3O + H2PO −4 (ii) H2PO −4 + H2O → HPO24− + H3O
(iii) H2PO −4 + OH− → H3PO4 + O2 −

(Hyderabad Classes) Limited. 5-9-14/B, Saifabad, (Opp. Secretariat) Hyderabad. 500 063. Phone: 040-66777000 – 03 Fax: 040-66777004
−2010−
AIEEE− −7

In which of the above does H2PO −4 act as an acid ?


(1) (ii) only (2) (i) and (ii) (3) (iii) only (4) (i) only
17. (1)
Sol : (i) H3PO 4 + H2 → H3 O + + H2PO −4
acid conjugate base

(ii) H2PO −
4
+ H2O → HPO −2
4 + H3 O+
acid conjugate base
–2
(iii) H2PO−4 + OH− → H3PO 4 +O
acid acid conjugate acid

Only in reaction (ii) H2PO −4 acids as ‘acid’.

18. In aqueous solution the ionization constants for carbonic acid are
–7 –11
K1 = 4.2 x 10 and K2 = 4.8 x 10
Select the correct statement for a saturated 0.034 M solution of the carbonic acid.
(1) The concentration of CO32 − is 0.034 M.
(2) The concentration of CO32− is greater than that of HCO3− .
+
(3) The concentration of H and HCO3− are approximately equal.
+
(4) The concentration of H is double that of CO32 − .
18. (3)
+ –7
Sol : A→ H2CO3 H + HCO3− K1 = 4.2 x 10
+ –11
B→ HCO3− H + CO3−2 K2 = 4.8 x 10
As K2 << K1
All major H+ total
≈ H+ A

and from I equilibrium, H+ A


≈ HCO3− ≈ H+ total

CO3−2 is negligible compared to HCO3− or H+ total

19. The edge length of a face centered cubic cell of an ionic substance is 508 pm. If the radius of the
cation is 110 pm, the radius of the anion is
(1) 288 pm (2) 398 pm (3) 618 pm (4) 144 pm
19. (4)
Sol : For an ionic substance in FCC arrangement,
2 (r + + r − ) = edge length
2 (110 + r − ) = 508

r = 144 pm

20. The correct order of increasing basicity of the given conjugate bases (R = CH3) is
(1) RCOO < HC = C < R < NH2 (2) R < HC ≡ C < RCOO < NH2
(3) RCOO < NH2 < HC ≡ C < R (4) RCOO < HC ≡ C < NH2 < R
20. (4)
Sol : Correct order of increasing basic strength is
R–COO < CH≡C < NH(2−) < R
(–) (–) (–)

21. The correct sequence which shows decreasing order of the ionic radii of the elements is
3+ 2+ + – 2– + 2+ 3+ 2– –
(1) Al > Mg > Na > F > O (2) Na > Mg > Al > O > F
+ – 2+ 2– 3+ 2– – + 2+ 3+
(3) Na > F > Mg > O > Al (4) O > F > Na > Mg > Al
21. (4)
Z
Sol : For isoelectronic species higher the ratio , smaller the ionic radius
e

(Hyderabad Classes) Limited. 5-9-14/B, Saifabad, (Opp. Secretariat) Hyderabad. 500 063. Phone: 040-66777000 – 03 Fax: 040-66777004
−2010−
AIEEE− −8

z 8
for O2 − = = 0.8
e 10
9
F− = = 0.9
10
11
Na + = = 1.1
10
12
Mg2 + = = 1.2
10
13
Al3 + = = 1.3
10
–13
22. Solubility product of silver bromide is 5.0 x 10 . The quantity of potassium bromide (molar mass
–1
taken as 120 g of mol ) to be added to 1 litre of 0.05 M solution of silver nitrate to start the
precipitation of AgBr is
–10 –9 –5 –8
(1) 1.2 x 10 g (2) 1.2 x 10 g (3) 6.2 x 10 g (4) 5.0 x 10 g
22. (2)
+ –
Sol : Ag + Br AgBr
Precipitation starts when ionic product just exceeds solubility product
K sp = Ag+ Br −
K sp 5 × 10−13
Br − = = = 10 −11
Ag +
0.05
–11
i.e., precipitation just starts when 10 moles of KBr is added to 1L of AgNO3 solution.
No. of moles of KBr to be added = 10–11
–11
∴ weight of KBr to be added = 10 x 120
–9
= 1.2 x 10 g

23. The Gibbs energy for the decomposition of Al2O3 at 500°C is as follows :
2 4 –1
Al2O3 → Al + O2, ∆rG = + 966 kJ mol
3 3
The potential difference needed for electrolytic reduction of Al2O3 at 500°C is at least
(1) 4.5 V (2) 3.0 V (3) 2.5 V (4) 5.0 V
23. (3)
−∆G
Sol : ∆G = – nFE E=
nF
966 × 103
E=−
4 × 96500
= –2.5 V
∴ The potential difference needed for the reduction = 2.5 V
–11 2+
24. At 25°C, the solubility product of Mg(OH)2 is 1.0 x 10 . At which pH, will Mg ions start precipitating
2+
in the form of Mg(OH)2 from a solution of 0.001 M Mg ions ?
(1) 9 (2) 10 (3) 11 (4) 8
24. (2)
Sol : Mg2 + + 2OH− Mg(OH)2
2
K sp = Mg2 + OH−
K sp
OH− = = 10 −4
Mg2 +
∴ pOH = 4 and pH = 10

(Hyderabad Classes) Limited. 5-9-14/B, Saifabad, (Opp. Secretariat) Hyderabad. 500 063. Phone: 040-66777000 – 03 Fax: 040-66777004
−2010−
AIEEE− −9

25. Percentage of free space in cubic close packed structure and in body centred packed structure are
respectively
(1) 30% and 26% (2) 26% and 32% (3) 32% and 48% (4) 48% and 26%
25. (2)
Sol : packing fraction of cubic close packing and body centred packing are 0.74 and 0.68 respectively.

26. Out of the following, the alkene that exhibits optical isomerism is
(1) 3–methyl–2–pentene (2) 4–methyl–1–pentene
(3) 3–methyl–1–pentene (4) 2–methyl–2–pentene
26. (3)
Sol :
H
H2C=HC C2 H5 only 3–methyl–1–pentene has a chiral carbon
CH3

27. Biuret test is not given by


(1) carbohydrates (2) polypeptides (3) urea (4) proteins
27. (1)
Sol : It is a test characteristic of amide linkage. Urea also has amide linkage like proteins.

28. The correct order of E0 2 + values with negative sign for the four successive elements Cr, Mn, Fe
M /M

and Co is
(1) Mn > Cr > Fe > Co (2) Cr > Fe > Mn > Co (3) Fe > Mn > Cr > Co (4) Cr > Mn > Fe > Co
28. (1)

29. The polymer containing strong intermolecular forces e.g. hydrogen bonding, is
(1) teflon (2) nylon 6,6 (3) polystyrene (4) natural rubber
29. (2)
Sol : nylon 6,6 is a polymer of adipic acid and hexamethylene diamine
O O
C (CH2)4 C NH (CH2)6 NH
n

30. For a particular reversible reaction at temperature T, ∆H and ∆S were found to be both +ve. If Te is
the temperature at equilibrium, the reaction would be spontaneous when
(1) Te > T (2) T > Te (3) Te is 5 times T (4) T = Te
30. (2)
Sol : ∆G = ∆H − T∆S
at equilibrium, ∆G = 0
for a reaction to be spontaneous ∆G should be negative
∴ T > Te

31. A rectangular loop has a sliding connector PQ of length and resistance R Ω and it is moving with a
speed v as shown. The set-up is placed in a uniform magnetic field going into the plane of the paper.
The three currents I1, I2 and I are

(Hyderabad Classes) Limited. 5-9-14/B, Saifabad, (Opp. Secretariat) Hyderabad. 500 063. Phone: 040-66777000 – 03 Fax: 040-66777004
−2010−
AIEEE− −10

B v 2B v
(1) I1 = −I2 = , I=
R R
B v 2B v
(2) I1 = I2 = , I=
3R 3R
B v
(3) I1 = I2 = I =
R
B v B v
(4) I1 = I2 = , I=
6R 3R
31. 2
Sol. A moving conductor is equivalent to a battery of emf = v B (motion emf)
Equivalent circuit
I = I1 + I2
applying Kirchoff’s law
R
I1R + IR − v B = 0 ……………(1) R
I2R + IR − v B = 0 ……………(2)
adding (1) & (2) I1 I2
2IR + IR = 2v B
2vB
I=
3R
vB
I1 = I2 =
3R

32. Let C be the capacitance of a capacitor discharging through a resistor R. Suppose t1 is the time
taken for the energy stored in the capacitor to reduce to half its initial value and t2 is the time taken
for the charge to reduce to one-fourth its initial value. Then the ratio t1/t2 will be
1 1
(1) 1 (2) (3) (4) 2
2 4
32. 3
1 q2 1 q2
Sol. U= = (q0 e− t / T )2 = 0 e−2t / T (where τ = CR )
2 C 2C 2C
U = Ui e −2t / τ
1
Ui = Ui e−2t 1 /τ

2
1 T
= e −2t / τ
1
t1 = ln 2
2 2
Now q = q0 e − t / T
1
q0 = q0 e − t / 2T
4
t 2 = T ln 4 = 2T ln 2
t1 1
∴ =
t2 4

Directions: Questions number 33 – 34 contain Statement-1 and Statement-2. Of the four choices given after
the statements, choose the one that best describes the two statements.

33. Statement-1 : Two particles moving in the same direction do not lose all their energy in a
completely inelastic collision.
Statement-2 : Principle of conservation of momentum holds true for all kinds of collisions.

(Hyderabad Classes) Limited. 5-9-14/B, Saifabad, (Opp. Secretariat) Hyderabad. 500 063. Phone: 040-66777000 – 03 Fax: 040-66777004
−2010−
AIEEE− −11

(1) Statement-1 is true, Statement-2 is true; Statement-2 is the correct explanation of Statement-1.
(2) Statement-1 is true, Statement-2 is true; Statement-2 is not the correct explanation of Statement-
1
(3) Statement-1 is false, Statement-2 is true.
(4) Statement-1 is true, Statement-2 is false.
33. 1
Sol. m1 m2
v1 v2
If it is a completely inelastic collision then
m1v1 + m2 v 2 = m1v + m2 v
m1v1 + m2 v 2
v=
m1 + m2
p12 p2
K.E = + 2
2m1 2m2
as p1 and p2 both simultaneously cannot be zero
therefore total KE cannot be lost.

34. Statement-1 : When ultraviolet light is incident on a photocell, its stopping potential is V0 and the
maximum kinetic energy of the photoelectrons is Kmax. When the ultraviolet light is replaced by X-
rays, both V0 and Kmax increase.
Statement-2 : Photoelectrons are emitted with speeds ranging from zero to a maximum value
because of the range of frequencies present in the incident light.
(1) Statement-1 is true, Statement-2 is true; Statement-2 is the correct explanation of Statement-1.
(2) Statement-1 is true, Statement-2 is true; Statement-2 is not the correct explanation of Statement-
1.
(3) Statement-1 is false, Statement-2 is true.
(4) Statement-1 is true, Statement-2 is false.
34. 4
Sol. Since the frequency of ultraviolet light is less than the frequency of X–rays, the energy of each
incident photon will be more for X–rays
K.E photoelectron = hν − ϕ
Stopping potential is to stop the fastest photoelectron
hν ϕ
V0 = −
e e
so, K.Emax and V0 both increases.
But K.E ranges from zero to K.Emax because of loss of energy due to subsequent collisions before
getting ejected and not due to range of frequencies in the incident light.

35. A ball is made of a material of density ρ where ρoil < ρ < ρwater with ρoil and ρwater representing the
densities of oil and water, respectively. The oil and water are immiscible. If the above ball is in
equilibrium in a mixture of this oil and water, which of the following pictures represents its equilibrium
position ?
(1) (2)

(Hyderabad Classes) Limited. 5-9-14/B, Saifabad, (Opp. Secretariat) Hyderabad. 500 063. Phone: 040-66777000 – 03 Fax: 040-66777004
−2010−
AIEEE− −12

(3) (4)

35. 2
Sol. ρoil < ρ < ρwater
Oil is the least dense of them so it should settle at the top with water at the base. Now the ball is
denser than oil but less denser than water. So, it will sink through oil but will not sink in water. So it
will stay at the oil–water interface.

36. A particle is moving with velocity v = K(y ˆi + x ˆj) , where K is a constant. The general equation for its
path is
2 2 2 2
(1) y = x + constant (2) y = x + constant (3) xy = constant (4) y = x + constant
36. 4
Sol. v = Ky ˆi + Kx ˆj
dx dy
= Ky, = Kx
dt dt
dy dy dt Kx
= × =
dx dt dx Ky
y dy = x dx
2 2
y = x + c.

37. Two long parallel wires are at a distance 2d apart. They carry steady equal current flowing out of the
plane of the paper as shown. The variation of the magnetic field along the line XX 'is given by
(1) (2)

(3) (4)

37. 1
Sol. The magnetic field in between because of each will be in opposite direction
µ i µ0i
Bin between = 0 ˆj − ( −ˆj)
2πx 2π(2d − x)
µi 1 1
= 0 − ( ˆj)
2π x 2d − x
at x = d, Bin between = 0
for x < d, Bin between = ( ˆj)
for x > d, Bin between = ( − ˆj)
towards x net magnetic field will add up and direction will be ( −ˆj)
towards x 'net magnetic field will add up and direction will be ( ˆj)

(Hyderabad Classes) Limited. 5-9-14/B, Saifabad, (Opp. Secretariat) Hyderabad. 500 063. Phone: 040-66777000 – 03 Fax: 040-66777004
−2010−
AIEEE− −13

38. In the circuit shown below, the key K is closed at t = 0. The current through the battery is
V R1 R2 V
(1) at t = 0 and at t = ∞
R1 + R2
2 2 R2
V V (R1 + R2 )
(2) at t = 0 and at t = ∞
R2 R1 R 2
V V R1 R2
(3) at t = 0 and at t = ∞
R2 R12 + R22
V (R1 + R2 ) V
(4) at t = 0 and at t = ∞
R1 R 2 R2
38. 2
Sol. At t = 0, inductor behaves like an infinite resistance
V
So at t = 0, i =
R2
and at t = ∞ , inductor behaves like a conducting wire
V V(R1 + R 2 )
i= =
Re q R1 R 2

39. The figure shows the position – time (x – t)


graph of one-dimensional motion of a body
of mass 0.4 kg. The magnitude of each
impulse is
(1) 0.4 Ns (2) 0.8 Ns
(3) 1.6 Ns (4) 0.2 Ns

39. 2
Sol. From the graph, it is a straight line so, uniform motion. Because of impulse direction of velocity
changes as can be seen from the slope of the graph.
2
Initial velocity = = 1 m / s
2
2
Final velocity = − = −1 m / s
2
Pi = 0.4 N – s
Pji = −0.4 N – s
J = Pf − Pi = – 0.4 – 0.4 = – 0.8 N – s ( J = impulse)
J = 0.8 N–s

Directions : Questions number 40 – 41 are based on the following paragraph.


M
A nucleus of mass M + ∆m is at rest and decays into two daughter nuclei of equal mass each.
2
Speed of light is c.

40. The binding energy per nucleon for the parent nucleus is E1 and that for the daughter nuclei is E2.
Then
(1) E2 = 2E1 (2) E1 > E2 (3) E2 > E1 (4) E1 = 2E2
40. 3
Sol. After decay, the daughter nuclei will be more stable hence binding energy per nucleon will be more
than that of their parent nucleus.

(Hyderabad Classes) Limited. 5-9-14/B, Saifabad, (Opp. Secretariat) Hyderabad. 500 063. Phone: 040-66777000 – 03 Fax: 040-66777004
−2010−
AIEEE− −14

41. The speed of daughter nuclei is


∆m 2∆m ∆m ∆m
(1) c (2) c (3) c (4) c
M + ∆m M M M + ∆m
41. 2
Sol. Conserving the momentum
M M
0 = V1 − V2
2 2
V1 = V2 …………….(1)
1 M 1 M
∆mc 2 = . V12 + . .V22 …………….(2)
2 2 2 2
M 2
∆mc = V1
2

2
2∆mc 2
= V12
M
2∆m
V1 = c
M

42. A radioactive nucleus (initial mass number A and atomic number Z) emits 3 α-particles and 2
positrons. The ratio of number of neutrons to that of protons in the final nucleus will be
A −Z−8 A−Z−4 A − Z − 12 A−Z−4
(1) (2) (3) (4)
Z−4 Z−8 Z−4 Z−2
42. 2
Sol. In positive beta decay a proton is transformed into a neutron and a positron is emitted.
p + 
→ n0 + e +
no. of neutrons initially was A – Z
no. of neutrons after decay (A – Z) – 3 x 2 (due to alpha particles) + 2 x 1 (due to positive beta
decay)
The no. of proton will reduce by 8. [as 3 x 2 (due to alpha particles) + 2(due to positive beta decay)]
Hence atomic number reduces by 8.

43. A thin semi-circular ring of radius r has a positive charge q


distributed uniformly over it. The net field E at the centre O is
q q
(1) ĵ (2) – ĵ
4 π2 ε0 r 2 4 π2 ε 0 r 2
q q
(3) – ĵ (4) ĵ
2 π2 ε0 r 2 2 π2 ε 0 r 2
43. 3
q y
Sol. Linear charge density λ =
πr
K.dq
E = dE sin θ( −ˆj) = sin θ( −ˆj) dθ
r2
θ x
K qr θ
E= 2 d θ sin θ( −ˆj)
r πr

K qπ
= sin θ( − ˆj)
r2 π 0
q
= ( −ˆj)
2π ε 0 r 2
2

(Hyderabad Classes) Limited. 5-9-14/B, Saifabad, (Opp. Secretariat) Hyderabad. 500 063. Phone: 040-66777000 – 03 Fax: 040-66777004
−2010−
AIEEE− −15

44. The combination of gates shown below yields


(1) OR gate (2) NOT gate
(3) XOR gate (4) NAND gate

44. 1
Sol. Truth table for given combination is
A B X
0 0 0
0 1 1
1 0 1
1 1 1
This comes out to be truth table of OR gate

45. A diatomic ideal gas is used in a Car engine as the working substance. If during the adiabatic
expansion part of the cycle, volume of the gas increases from V to 32V the efficiency of the engine is
(1) 0.5 (2) 0.75 (3) 0.99 (4) 0.25
45. 2
Sol. The efficiency of cycle is
T
η = 1− 2
T1
for adiabatic process
TVγ–1 = constant
7
For diatomic gas γ =
5
γ −1 γ −1
T1V1 = T2 V2
γ −1
V2
T1 = T2
V1
7
−1
T1 = T2 (32)5
= T2 (25 )2 / 5
= T2 x 4
T1 = 4T2.
1 3
η = 1− = = 0.75
4 4
20
46. If a source of power 4 kW produces 10 photons/second, the radiation belong to a part of the
spectrum called
(1) X–rays (2) ultraviolet rays (3) microwaves (4) γ–rays
46. 1
3 20
Sol. 4 x 10 = 10 x hf
4 × 103
f = 20
10 × 6.023 × 10−34
16
f = 6.03 x 10 Hz
The obtained frequency lies in the band of X–rays.

47. The respective number of significant figures for the numbers 23.023, 0.0003 and 2.1 x 10–3 are
(1) 5, 1, 2 (2) 5, 1, 5 (3) 5, 5, 2 (4) 4, 4, 2
47. 1

(Hyderabad Classes) Limited. 5-9-14/B, Saifabad, (Opp. Secretariat) Hyderabad. 500 063. Phone: 040-66777000 – 03 Fax: 040-66777004
−2010−
AIEEE− −16

48. In a series LCR circuit R = 200 Ω and the voltage and the frequency of the main supply is 220 V and
50 Hz respectively. On taking out the capacitance from the circuit the current lags behind the voltage
o
by 30°. On taking out the inductor from the circuit the current leads the voltage by 30 . The power
dissipated in the LCR circuit is
(1) 305 W (2) 210 W (3) Zero W (4) 242 W
48. 4
Sol. The given circuit is under resonance as XL = XC
Hence power dissipated in the circuit is
V2
P= = 242 W
R

49. Let there be a spherically symmetric charge distribution with charge density varying as
5 r
ρ(r) = ρ0 − upto r = R, and ρ(r) = 0 for r > R, where r is the distance from the origin. The
4 R
electric field at a distance r(r < R) from the origin is given by
4πρ0 r 5 r ρ r 5 r 4 ρ0 r 5 r ρ r 5 r
(1) − (2) 0 − (3) − (4) 0 −
3 ε0 3 R 4 ε0 3 R 3 ε0 4 R 3 ε0 4 R
49. 2
Sol. Apply shell theorem the total charge upto distance r can be calculated as followed
dq = 4 πr 2 .dr.ρ
5 r
= 4πr 2 .dr.ρ0 −
4 R
5 2 r3
= 4πρ0 r dr − dr
4 R
r
5 2 r3
dq = q = 4πρ0 r dr − dr
0 4 R
5 r3 1 r4
= 4πρ0 −
43 R 4
kq
E=
r2
1 1 5 r3 r4
= .4πρ0 −
4πε0 r 2
4 3 4R
ρ0 r 5 r
E= −
4ε 0 3 R

50. The potential energy function for the force between two atoms in a diatomic molecule is
a b
approximately given by U(x) = 12 − 6 , where a and b are constants and x is the distance between
x x
the atoms. If the dissociation energy of the molecule is D = [U(x = ∞) – Uat equilibrium], D is
b2 b2 b2 b2
(1) (2) (3) (4)
2a 12a 4a 6a
50. 3
a b
Sol. U(x) = 12 − 6
x x
U(x = ∞) = 0
dU 12a 6b
as, F=− = − 13 + 7
dx x x
at equilibrium, F = 0

(Hyderabad Classes) Limited. 5-9-14/B, Saifabad, (Opp. Secretariat) Hyderabad. 500 063. Phone: 040-66777000 – 03 Fax: 040-66777004
−2010−
AIEEE− −17

2a
∴ x6 =
b
a b −b2
∴ Uat equilibrium = 2
− =
2a 2a 4a
b b
b2
∴ D = U(x = ∞) − Uat equilibrium =
4a

51. Two identical charged spheres are suspended by strings of equal lengths. The strings make an
–3
angle of 30° with each other. When suspended in a liquid of density 0.8 g cm , the angle remains
–3
the same. If density of the material of the sphere is 16 g cm , the dielectric constant of the liquid is
(1) 4 (2) 3 (3) 2 (4) 1
51. 3
Sol. From F.B.D of sphere, using Lami’s theorem
F
= tan θ ………………(i) T θ
mg
when suspended in liquid, as θ remains same, F
F'
∴ = tan θ ………………(ii)
ρ
mg 1 − mg
d
using (i) and (ii)
F F' F
= where, F'=
mg ρ K
mg 1 −
d
F F'
∴ =
mg ρ
mg K 1 −
d
1
or K= =2
ρ
1−
d

52. Two conductors have the same resistance at 0oC but their temperature coefficients of resistance are
α1 and α2. The respective temperature coefficients of their series and parallel combinations are
nearly
α + α2 α + α2 α1 α 2 α + α 2 α1 + α 2
(1) 1 , α1 + α 2 (2) α1 + α 2 , 1 (3) α1 + α 2 , (4) 1 ,
2 2 α1 + α 2 2 2
52. 4
Sol. Let R0 be the initial resistance of both conductors
∴ At temperature θ their resistance will be,
R1 = R0 (1 + α1θ) and R2 = R0 (1 + α 2 θ)
for, series combination, Rs = R1 + R2
R s 0 (1 + α s θ) = R0 (1 + α1θ) + R0 (1 + α 2 θ)
where R s 0 = R 0 + R0 = 2R0
∴ 2R0 (1 + α s θ) = 2R0 + R0 θ(α1 + α 2 )
α1 + α 2
or αs =
2
R1 R 2
for parallel combination, Rp =
R1 + R 2

(Hyderabad Classes) Limited. 5-9-14/B, Saifabad, (Opp. Secretariat) Hyderabad. 500 063. Phone: 040-66777000 – 03 Fax: 040-66777004
−2010−
AIEEE− −18

R0 (1 + α1θ)R0 (1 + α 2 θ)
Rp0 (1 + αp θ) =
R0 (1 + α1θ) + R0 (1 + α 2 θ)
R0 R0 R
where, Rp0 = = 0
R0 + R 0 2
R0 R2 (1 + α1θ + α 2 θ + α1α 2 θ)
∴ (1 + αp θ) = 0
2 R 0 (2 + α1θ + α 2 θ)
as α1 and α 2 are small quantities
∴ α1 α 2 is negligible
α1 + α 2 α + α2
or αp = = 1 [1 − (α1 + α 2 )θ]
2 + (α1 + α 2 )θ 2
as ( α1 + α 2 )2 is negligible
α1 + α 2
∴ αp =
2

53. A point P moves in counter-clockwise direction on a circular path


as shown in the figure. The movement of ‘P’ is such that it
3
sweeps out a length s = t + 5, where s is in metres and t is in
seconds. The radius of the path is 20 m. The acceleration of ‘P’
when t = 2 s is nearly
2 2
(1) 13 m/s (2) 12 m/s
2 2
(3) 7.2 m/s (4) 14 m/s

53. 4
Sol. S = t3 + 5
ds
∴ speed, v = = 3t 2
dt
dv
and rate of change of speed = = 6t
dt
2
∴ tangential acceleration at t = 2s, at = 6 x 2 = 12 m/s
2
at t = 2s, v = 3(2) = 12 m/s
v 2 144
∴ centripetal acceleration, ac = = m / s2
R 20
∴ net acceleration = a2t + ai2
≈ 14 m / s2

o
54. Two fixed frictionless inclined plane making an angle 30
and 60o with the vertical are shown in the figure. Two
block A and B are placed on the two planes. What is the
relative vertical acceleration of A with respect to B ?
–2
(1) 4.9 ms in horizontal direction
–2
(2) 9.8 ms in vertical direction
(3) zero
–2
(4) 4.9 ms in vertical direction

54. 4
Sol. mg sin θ = ma
∴ a = g sin θ
where a is along the inclined plane
2
∴ vertical component of acceleration is g sin θ
∴ relative vertical acceleration of A with respect to B is

(Hyderabad Classes) Limited. 5-9-14/B, Saifabad, (Opp. Secretariat) Hyderabad. 500 063. Phone: 040-66777000 – 03 Fax: 040-66777004
−2010−
AIEEE− −19

g
g[sin2 60 − sin2 30] = = 4.9 m / s2 in vertical direction.
2

55. For a particle in uniform circular motion the acceleration a at a point P(R, θ) on the circle of radius R
is (here θ is measured from the x–axis)
v2 v2 v2 v2
(1) − cos θ ˆi + sin θ ˆj (2) − sin θ ˆi + cos θ ˆj
R R R R
2 2 2 2
v v v ˆ v ˆ
(3) − cos θ ˆi − sin θ ˆj (4) i+ j
R R R R
55. 3
Sol. For a particle in uniform circular motion, y
v2
a= towards centre of circle
R P (R, θ)
v2 ac
∴ a= ( − cos θ ˆi − sin θ ˆj)
R x
v2 v2 ac
or ˆ
a = − cos θ i − sin θ j ˆ
R R

Directions: Questions number 56 – 58 are based on the following paragraph.


An initially parallel cylindrical beam travels in a medium of refractive index µ(I) = µ0 + µ 2I , where µ0
and µ2 are positive constants and I is the intensity of the light beam. The intensity of the beam is
decreasing with increasing radius.

56. As the beam enters the medium, it will


(1) diverge
(2) converge
(3) diverge near the axis and converge near the periphery
(4) travel as a cylindrical beam
56. 2
Sol. As intensity is maximum at axis,
∴ µ will be maximum and speed will be minimum on the axis of the beam.
∴ beam will converge.

57. The initial shape of the wave front of the beam is


(1) convex
(2) concave
(3) convex near the axis and concave near the periphery
(4) planar
57. 4
Sol. For a parallel cylinderical beam, wavefront will be planar.

58. The speed of light in the medium is


(1) minimum on the axis of the beam (2) the same everywhere in the beam
(3) directly proportional to the intensity I (4) maximum on the axis of the beam
58. 1

(Hyderabad Classes) Limited. 5-9-14/B, Saifabad, (Opp. Secretariat) Hyderabad. 500 063. Phone: 040-66777000 – 03 Fax: 040-66777004
−2010−
AIEEE− −20

59. A small particle of mass m is projected at an angle θ with the


x-axis with an initial velocity v0 in the x-y plane as shown in the
v sin θ
figure. At a time t < 0 , the angular momentum of the
g
particle is
(1) −mgv 0 t 2 cos θ ˆj (2) mgv 0 t cos θ kˆ
1 1
(3) − mgv 0 t 2 cos θ kˆ (4) mgv 0 t 2 cos θ ˆi
2 2
where ˆi, ˆj and kˆ are unit vectors along x, y and z–axis respectively.
59. 3
Sol. L = m(r × v)
1
L = m v 0 cos θt ˆi + (v 0 sin θt − gt 2 )jˆ × v 0 cos θ ˆi + (v 0 sin θ − gt)jˆ
2
1
= mv 0 cos θt − gt kˆ
2
1
= − mgv 0 t 2 cos θkˆ
2
–1
60. The equation of a wave on a string of linear mass density 0.04 kg m is given by
t x
y = 0.02(m)sin 2π − . The tension in the string is
0.04(s) 0.50(m)
(1) 4.0 N (2) 12.5 N (3) 0.5 N (4) 6.25 N
60. 4
ω2 (2π / 0.004)2
Sol. T = µv 2 = µ = 0.04 = 6.25 N
k2 (2π / 0.50)2

4 5 π
61. Let cos(α + β) = and let sin(α – β) = , where 0 ≤ α, β ≤ , then tan 2α =
5 13 4
56 19 20 25
(1) (2) (3) (4)
33 12 7 16
61. 1
4 3
cos (α + β) = tan(α + β) =
5 4
5 5
sin(α – β) = tan(α – β) =
13 12
3 5
+
56
tan 2α = tan(α + β + α – β) = 4 12 =
3 5 33
1−
4 12

62. Let S be a non-empty subset of R. Consider the following statement:


P: There is a rational number x ∈ S such that x > 0.
Which of the following statements is the negation of the statement P ?
(1) There is no rational number x ∈ S such that x ≤ 0
(2) Every rational number x ∈ S satisfies x ≤ 0

(Hyderabad Classes) Limited. 5-9-14/B, Saifabad, (Opp. Secretariat) Hyderabad. 500 063. Phone: 040-66777000 – 03 Fax: 040-66777004
−2010−
AIEEE− −21

(3) x ∈ S and x ≤ 0 x is not rational


(4) There is a rational number x ∈ S such that x ≤ 0
62. 2
P: there is a rational number x ∈ S such that x > 0
~P: Every rational number x ∈ S satisfies x ≤ 0

63. Let a = ˆj − kˆ and c = ˆi − ˆj − kˆ . Then vector b satisfying a × b + c = 0 and a ⋅ b = 3 is


(1) 2iˆ − ˆj + 2kˆ (2) ˆi − ˆj − 2kˆ (3) ˆi + ˆj − 2kˆ (4) −ˆi + ˆj − 2kˆ
63. 4
c = b×a
b⋅c = 0
( )( )
b1ˆi + b2 ˆj + b3kˆ ⋅ ˆi − ˆj − kˆ = 0
b1 – b2 – b3 = 0
and a ⋅ b = 3
b2 – b3 = 3
b1 = b2 + b3 = 3 + 2b3
b = ( 3 + 2b3 ) ˆi + ( 3 + b3 ) ˆj + b3kˆ .

4
64. The equation of the tangent to the curve y = x + , that is parallel to the x-axis, is
x2
(1) y = 1 (2) y = 2 (3) y = 3 (4) y = 0
64. 3
dy 8
Parallel to x-axis =0 1− =0
dx x3
x=2 y=3
Equation of tangent is y – 3 = 0(x – 2) y–3=0

π
65. Solution of the differential equation cos x dy = y(sin x – y) dx, 0 < x < is
2
(1) y sec x = tan x + c (2) y tan x = sec x + c (3) tan x = (sec x + c)y (4) sec x = (tan x + c)y
65. 4
cos x dy = y(sin x – y) dx
dy
= y tan x − y 2 sec x
dx
1 dy 1
− tan x = − sec x
y 2 dx y
1
Let =t
y
1 dy dt
− =
y 2 dx dx
dy dt
− – t tan x = –sec x + (tan x) t = sec x.
dx dx
tan x dx
I.F. = e = sec x
Solution is t(I.F) = (I.F) sec x dx

(Hyderabad Classes) Limited. 5-9-14/B, Saifabad, (Opp. Secretariat) Hyderabad. 500 063. Phone: 040-66777000 – 03 Fax: 040-66777004
−2010−
AIEEE− −22

1
sec x = tan x + c
y


66. The area bounded by the curves y = cos x and y = sin x between the ordinates x = 0 and x = is
2
(1) 4 2 + 2 (2) 4 2 – 1 (3) 4 2 + 1 (4) 4 2 – 2
66. 4
π 5π 3π
4 4 2
( cos x − sin x ) dx + ( sin x − cos x ) dx + ( cos x − sin x ) = 4 2 −2
0 π 5π
4 4

cos x sin x

5π 3π
π 4 2
0 π 2π
4

2
67. If two tangents drawn from a point P to the parabola y = 4x are at right angles, then the locus of P is
(1) 2x + 1 = 0 (2) x = –1 (3) 2x – 1 = 0 (4) x = 1
67. 2
The locus of perpendicular tangents is directrix
i.e, x = –a; x = –1

68. If the vectors a = ˆi − ˆj + 2k,


ˆ b = 2iˆ + 4ˆj + kˆ and c = λ ˆi + ˆj + µkˆ are mutually orthogonal, then (λ, µ) =
(1) (2, –3) (2) (–2, 3) (3) (3, –2) (4) (–3, 2)
68. 4
a ⋅ b = 0, b ⋅ c = 0, c ⋅a = 0
2λ + 4 + µ = 0 λ – 1 + 2µ = 0
Solving we get: λ = –3, µ = 2

69. Consider the following relations:


R = {(x, y) | x, y are real numbers and x = wy for some rational number w};
m p
S= , m, n, p and q are integers such that n, q ≠ 0 and qm = pn . Then
n q
(1) neither R nor S is an equivalence relation
(2) S is an equivalence relation but R is not an equivalence relation
(3) R and S both are equivalence relations
(4) R is an equivalence relation but S is not an equivalence relation
69. 2
xRy need not implies yRx
m p
S: s ⇔ qm = pn
n q
m m
s reflexive
n n

(Hyderabad Classes) Limited. 5-9-14/B, Saifabad, (Opp. Secretariat) Hyderabad. 500 063. Phone: 040-66777000 – 03 Fax: 040-66777004
AIEEE−2010−23

m p p m
s s symmetric
n q q n
m p p r
s , s qm = pn, ps = rq ms = rn transitive.
n q q s
S is an equivalence relation.

k − 2x, if x ≤ −1
70. Let f: R → R be defined by f(x) = . If f has a local minimum at x = –1, then a
2x + 3, if x > −1
possible value of k is
1
(1) 0 (2) − (3) –1 (4) 1
2
70. 3
f(x) = k – 2x if x ≤ –1
= 2x + 3 if x > –1

2x + 3
k – 2x
1

–1
This is true
lim f(x) ≤ –1 where k = –1
x →−1−

71. The number of 3 × 3 non-singular matrices, with four entries as 1 and all other entries as 0, is
(1) 5 (2) 6 (3) at least 7 (4) less than 4
71. 3
First row with exactly one zero; total number of cases = 6
First row 2 zeros we get more cases
Total we get more than 7.

Directions: Questions Number 72 to 76 are Assertion – Reason type questions. Each of these questions
contains two statements.
Statement-1: (Assertion) and Statement-2: (Reason)
Each of these questions also has four alternative choices, only one of which is the correct answer.
You have to select the correct choice.

72. Four numbers are chosen at random (without replacement) from the set {1, 2, 3, ....., 20}.
Statement-1: The probability that the chosen numbers when arranged in some order will form an AP
1
is .
85
Statement-2: If the four chosen numbers from an AP, then the set of all possible values of common
difference is {±1, ±2, ±3, ±4, ±5}.
(1) Statement-1 is true, Statement-2 is true; Statement-2 is not the correct explanation for Statement-1
(2) Statement-1 is true, Statement-2 is false
(3) Statement-1 is false, Statement-2 is true
(4) Statement-1 is true, Statement-2 is true; Statement-2 is the correct explanation for Statement-1
72. 2

(Hyderabad Classes) Limited. 5-9-14/B, Saifabad, (Opp. Secretariat) Hyderabad. 500 063. Phone: 040-66777000 – 03 Fax: 040-66777004
AIEEE−2010−24

20
N(S) = C4
Statement-1: common difference is 1; total number of cases = 17
common difference is 2; total number of cases = 14
common difference is 3; total number of cases = 11
common difference is 4; total number of cases = 8
common difference is 5; total number of cases = 5
common difference is 6; total number of cases = 2
17 + 14 + 11 + 8 + 5 + 2 1
Prob. = 20
= .
C4 85

73. Statement-1: The point A(3, 1, 6) is the mirror image of the point B(1, 3, 4) in the plane x – y + z = 5.
Statement-2: The plane x – y + z = 5 bisects the line segment joining A(3, 1, 6) and B(1, 3, 4).
(1) Statement-1 is true, Statement-2 is true; Statement-2 is not the correct explanation for Statement-1
(2) Statement-1 is true, Statement-2 is false
(3) Statement-1 is false, Statement-2 is true
(4) Statement-1 is true, Statement-2 is true; Statement-2 is the correct explanation for Statement-1
73. 1
A(3, 1, 6); B = (1, 3, 4)
Mid-point of AB = (2, 2, 5) lies on the plane.
and d.r’s of AB = (2, –2, 2)
d.r’s Of normal to plane = (1, –1, 1).
AB is perpendicular bisector
∴ A is image of B
Statement-2 is correct but it is not correct explanation.

10 10 10
74. Let S1 = j ( j − 1) 10 C j , S2 = j 10
C j and S3 = j2 10
Cj .
j =1 j =1 j =1
9
Statement-1: S3 = 55 × 2
8 8
Statement-2: S1 = 90 × 2 and S2 = 10 × 2 .
(1) Statement-1 is true, Statement-2 is true; Statement-2 is not the correct explanation for Statement-1
(2) Statement-1 is true, Statement-2 is false
(3) Statement-1 is false, Statement-2 is true
(4) Statement-1 is true, Statement-2 is true; Statement-2 is the correct explanation for Statement-1
74. 2
10
10! 10
8!
S1 = j ( j − 1) = 90 = 90 ⋅ 28 .
j =1 j ( )( ) (
j − 1 j − 2 ! 10 − j ) ! j= 2 ( j − 2 ) ( ( ))
! 8 − j − 2 !
10
10! 10
9!
S2 = j = 10 = 10 ⋅ 29 .
j =1 j ( j − 1)! ( 9 − ( j − 1) )! j =1 ( j − 1) ( ( ))
! 9 − j − 1 !
10
10! 10 10
j ( j − 1) + j j ( j − 1) 10 C j =
8 9
S3 = = j 10 C j = 90 . 2 + 10 . 2
j =1 j! (10 − j )! j =1 j =1
8 8 8 9
= 90 . 2 + 20 . 2 = 110 . 2 = 55 . 2 .

2
75. Let A be a 2 × 2 matrix with non-zero entries and let A = I, where I is 2 × 2 identity matrix. Define
Tr(A) = sum of diagonal elements of A and |A| = determinant of matrix A.
Statement-1: Tr(A) = 0
Statement-2: |A| = 1
(1) Statement-1 is true, Statement-2 is true; Statement-2 is not the correct explanation for Statement-1
(2) Statement-1 is true, Statement-2 is false

(Hyderabad Classes) Limited. 5-9-14/B, Saifabad, (Opp. Secretariat) Hyderabad. 500 063. Phone: 040-66777000 – 03 Fax: 040-66777004
AIEEE−2010−25

(3) Statement-1 is false, Statement-2 is true


(4) Statement-1 is true, Statement-2 is true; Statement-2 is the correct explanation for Statement-1
75. 2
a b
Let A = , abcd ≠ 0
c d
2 a b a b
A = ⋅
c d c d
2 a2 + bc ab + bd
A =
ac + cd bc + d2
a2 + bc = 1, bc + d2 = 1
ab + bd = ac + cd = 0
c ≠ 0 and b ≠ 0 a+d=0
Trace A = a + d = 0
|A| = ad – bc = –a2 – bc = –1.

1
76. Let f: R → R be a continuous function defined by f(x) = .
e + 2e − x
x

1
Statement-1: f(c) = , for some c ∈ R.
3
1
Statement-2: 0 < f(x) ≤ , for all x ∈ R
2 2
(1) Statement-1 is true, Statement-2 is true; Statement-2 is not the correct explanation for Statement-1
(2) Statement-1 is true, Statement-2 is false
(3) Statement-1 is false, Statement-2 is true
(4) Statement-1 is true, Statement-2 is true; Statement-2 is the correct explanation for Statement-1
76. 4
1 ex
f(x) = x =
e + 2e− x e2x + 2

f′(x) =
(e 2x
)
+ 2 e x − 2e2x ⋅ e x

(e )2x + 2 2

2x 2x
f′(x) = 0 e + 2 = 2e
2x x
e =2 e = 2
2 1
maximum f(x) = =
4 2 2
1
0 < f(x) ≤ ∀x∈R
2 2
1 1
Since 0 < < for some c ∈ R
3 2 2
1
f(c) =
3

77. For a regular polygon, let r and R be the radii of the inscribed and the circumscribed circles. A false
statement among the following is
r 1 r 2
(1) There is a regular polygon with = (2) There is a regular polygon with =
R 2 R 3

(Hyderabad Classes) Limited. 5-9-14/B, Saifabad, (Opp. Secretariat) Hyderabad. 500 063. Phone: 040-66777000 – 03 Fax: 040-66777004
AIEEE−2010−26

r 3 r 1
(3) There is a regular polygon with = (4) There is a regular polygon with =
R 2 R 2
77. 2
a π
r= cot
2 n
‘a’ is side of polygon.
a π
R = cosec
2 n
π
cot
r
= n = cos π
R π n
cosec
n
π 2
cos ≠ for any n ∈ N.
n 3

2 2009 2009
78. If α and β are the roots of the equation x – x + 1 = 0, then α +β =
(1) –1 (2) 1 (3) 2 (4) –2
78. 2
2 1± 1− 4
x –x+1=0 x=
2
1± 3 i
x=
2
1 3 1 i 3
α = +i , − β=
2 2 2 2
π π π π
α = cos + isin , β = cos − isin
3 3 3 3
2009 2009 π
α +β = 2cos2009
3
2π 2π
= 2cos 668π + π + = 2cos π +
3 3
2π 1
= −2cos = −2 − =1
3 2

79. The number of complex numbers z such that |z – 1| = |z + 1| = |z – i| equals


(1) 1 (2) 2 (3) ∞ (4) 0
79. 1
Let z = x + iy
|z – 1| = |z + 1| Re z = 0 x=0
|z – 1| = |z – i| x=y
|z + 1| = |z – i| y = –x
Only (0, 0) will satisfy all conditions.
Number of complex number z = 1

80. A line AB in three-dimensional space makes angles 45° and 120° with the positive x-axis and the
positive y-axis respectively. If AB makes an acute angle θ with the positive z-axis, then θ equals
(1) 45° (2) 60° (3) 75° (4) 30°
80. 2

(Hyderabad Classes) Limited. 5-9-14/B, Saifabad, (Opp. Secretariat) Hyderabad. 500 063. Phone: 040-66777000 – 03 Fax: 040-66777004
AIEEE−2010−27

1
= cos 45° =
2
1
m = cos 120° = −
2
n = cos θ
where θ is the angle which line makes with positive z-axis.
2 2 2
Now + m + n = 1
1 1 2
+ + cos θ = 1
2 4
2 1
cos θ =
4
1
cos θ = (θ Being acute)
2
π
θ= .
3

x y
81. The line L given by + = 1 passes through the point (13, 32). The line K is parallel to L and has
5 b
x y
the equation + = 1. Then the distance between L and K is
c 3
17 23 23
(1) 17 (2) (3) (4)
15 17 15
81. 3
b
Slope of line L = −
5
3
Slope of line K = −
c
Line L is parallel to line k.
b 3
= bc = 15
5 c
(13, 32) is a point on L.
13 32 32 8
+ =1 =−
5 b b 5
3
b = –20 c= −
4
Equation of K: y – 4x = 3
52 − 32 + 3 23
Distance between L and K = =
17 17

th
82. A person is to count 4500 currency notes. Let an denote the number of notes he counts in the n
minute. If a1 = a2 = ...... = a10 = 150 and a10, a11, ...... are in A.P. with common difference –2, then the
time taken by him to count all notes is
(1) 34 minutes (2) 125 minutes (3) 135 minutes (4) 24 minutes
82. 1
th
Till 10 minute number of counted notes = 1500
n
3000 = [2 × 148 + (n – 1)(–2)] = n[148 – n + 1]
2

(Hyderabad Classes) Limited. 5-9-14/B, Saifabad, (Opp. Secretariat) Hyderabad. 500 063. Phone: 040-66777000 – 03 Fax: 040-66777004
AIEEE−2010−28

2
n – 149n + 3000 = 0
n = 125, 24
n = 125 is not possible.
Total time = 24 + 10 = 34 minutes.

f(3x) f(2x)
83. Let f: R → R be a positive increasing function with lim = 1. Then lim =
x →∞ f(x) x →∞ f(x)

2 3
(1) (2) (3) 3 (4) 1
3 2
83. 4
f(x) is a positive increasing function
0 < f(x) < f(2x) < f(3x)
f(2x) f(3x)
0<1< <
f(x) f(x)
f(2x) f(3x)
lim 1 ≤ lim ≤ lim
x →∞ x →∞ f(x) x →∞ f(x)

By sandwich theorem.
f(2x)
lim =1
x →∞ f(x)

84. Let p(x) be a function defined on R such that p′(x) = p′(1 – x), for all x ∈ [0, 1], p(0) = 1 and p(1) = 41.
1
Then p(x) dx equals
0

(1) 21 (2) 41 (3) 42 (4) 41


84. 1
p′(x) = p′(1 – x)
p(x) = –p(1 – x) + c
at x = 0
p(0) = –p(1) + c 42 = c
now p(x) = –p(1 – x) + 42
p(x) + p(1 – x) = 42
1 1
I= p(x) dx = p(1 − x) dx
0 0
1
2I= (42) dx I = 21.
0

2
85. Let f: (–1, 1) → R be a differentiable function with f(0) = –1 and f′(0) = 1. Let g(x) = [f(2f(x) + 2)] .
Then g′(0) =
(1) –4 (2) 0 (3) –2 (4) 4
85. 1
d
g′(x) = 2(f(2f(x) + 2))
dx
( f ( 2f(x) + 2)) = 2f(2f(x) + 2) f′(2f(x) + 2) . (2f′(x))
g′(0) = 2f(2f(0) + 2) . f′(2f(0) + 2) . 2(f′(0) = 4f(0) f′(0)
= 4(–1) (1) = –4

(Hyderabad Classes) Limited. 5-9-14/B, Saifabad, (Opp. Secretariat) Hyderabad. 500 063. Phone: 040-66777000 – 03 Fax: 040-66777004
AIEEE−2010−29

86. There are two urns. Urn A has 3 distinct red balls and urn B has 9 distinct blue balls. From each urn
two balls are taken out at random and then transferred to the other. The number of ways in which
this can be done is
(1) 36 (2) 66 (3) 108 (4) 3
86. 3
3 9
Total number of ways = C2 × C2
9×8
=3× = 3 × 36 = 108
2

87. Consider the system of linear equations:


x1 + 2x2 + x3 = 3
2x1 + 3x2 + x3 = 3
3x1 + 5x2 + 2x3 = 1
The system has
(1) exactly 3 solutions (2) a unique solution
(3) no solution (4) infinite number of solutions
87. 3
1 2 1
D= 2 3 1=0
3 5 2
3 2 1
D1 = 3 3 1 ≠ 0
1 5 2
Given system, does not have any solution.
No solution.

88. An urn contains nine balls of which three are red, four are blue and two are green. Three balls are
drawn at random without replacement from the urn. The probability that the three balls have different
colour is
2 1 2 1
(1) (2) (3) (4)
7 21 23 3
88. 1
9
n(S) = C3
3 4 2
n(E) = C1 × C1 × C1
3 × 4 × 2 24 × 3! 24 × 6 2
Probability = 9 = × 6! = = .
C3 9! 9×8×7 7

89. For two data sets, each of size 5, the variances are given to be 4 and 5 and the corresponding
means are given to be 2 and 4, respectively. The variance of the combined data set is
11 13 5
(1) (2) 6 (3) (4)
2 2 2
89. 1
2
σx = 4
2
σy = 5
x= 2
y= 4

(Hyderabad Classes) Limited. 5-9-14/B, Saifabad, (Opp. Secretariat) Hyderabad. 500 063. Phone: 040-66777000 – 03 Fax: 040-66777004
AIEEE−2010−30

xi
=2 xi = 10; yi = 20
5
1 1
2
σx =
2
xi2 − ( x ) =
2

5
( yi2 ) − 16
2
xi = 40
2
yi = 105
2
1 x+y 1 145 − 90 55 11
2
σz =
10
( xi 2 + )
y i2 −
2
=
10
( 40 + 105 ) − 9 =
10
= =
10 2

2 2
90. The circle x + y = 4x + 8y + 5 intersects the line 3x – 4y = m at two distinct points if
(1) –35 < m < 15 (2) 15 < m < 65 (3) 35 < m < 85 (4) –85 < m < –35
90. 1
2 2
Circle x + y – 4x – 8y – 5 = 0
Centre = (2, 4), Radius = 4 + 16 + 5 = 5
If circle is intersecting line 3x – 4y = m
at two distinct points.
length of perpendicular from centre < radius
6 − 16 − m
<5
5
|10 + m| < 25
–25 < m + 10 < 25
–35 < m < 15.

***

(Hyderabad Classes) Limited. 5-9-14/B, Saifabad, (Opp. Secretariat) Hyderabad. 500 063. Phone: 040-66777000 – 03 Fax: 040-66777004
AIEEE−2010−31

READ THE FOLLOWING INSTRUCTIONS CAREFULLY:

( ! .! , /

2 ! > ! #

$ 4 1 . ) /
!

% 3 < ( #

* 2 # > . %/ (
1 # < #
!

9 7 ! # . )
- ! - /# <

; !
<
# @! 4 " A < %
. ' , $/

8 3 # < ! <
4 7 7 < #

& 5 - <

' 1 # ! < # <

< 5) 7 < !
< ! -
! < < !
( 7 :
< !

5 : - 5 #

$ < 4 4
5) 7 4 4

% 1 !

* - ) # # # #
# < ) - )

(Hyderabad Classes) Limited. 5-9-14/B, Saifabad, (Opp. Secretariat) Hyderabad. 500 063. Phone: 040-66777000 – 03 Fax: 040-66777004
®

PAPER-1: PHYSICS, MATHEMATICS & CHEMISTRY

Code:
Q
SOLUTIONS TO AIEEE - 2011
Time: 3 hours Maximum Marks: 360
IMPORTANT INSTRUCTIONS:

1. Immediately fill in the particulars on this page of the Test Booklet with Blue/Black Ball Point Pen. Use
of pencil is strictly prohibited.
2. The Answer Sheet is kept inside this Test Booklet. When you are directed to open the Test Booklet,
take out the Answer Sheet and fill in the particulars carefully.
3. The test is of 3 hours duration.
4. The Test Booklet consists of 90 questions. The maximum marks are 360.
5. There are three parts in the questions paper A, B, C consisting of Physics, Mathematics and
Chemistry having 30 questions in each part of equal weightage. Each question is allotted 4(four)
marks for each correct response.
6. Candidates will be awarded marks as stated above in instruction No. 5 for correct response of each
question. ¼ (one fourth) marks will be deducted for indicating incorrect response of each question. No
deduction from the total score will be made if no response is indicated for an item in the answer sheet.
7. There is only one correct response for each question. Filling up more than one response in each
question will be treated as wrong response and marks for wrong response will be deducted accordingly
as per instruction 6 above.
8. Use Blue/Black Ball Point Pen only for writing particulars / marking response on side-1 and
Side – 2 of the Answer Sheet. Use of pencil is strictly prohibited.
9. No candidate is allowed to carry any textual material printed or written, bits of papers, pager, mobile
phone, any electronic device etc; except the Admit Card inside the examination hall/room.
10. Rough work is to be done on the space provided for this purpose in the Test Booklet only. This space
is given at the bottom of each page and in 3 pages at the end of the booklet.
11. On completion of the test, the candidate must hand over the Answer Sheet to the invigilator on duty in
the Room/Hall. However, the candidates are allowed to take away this Test Booklet with them.
12. The CODE for this Booklet is Q. Make sure that the CODE printed on side – 2 of the Answer Sheet is
the same as that on this booklet. In case of discrepancy the candidate should immediately report the
matter to the invigilator for replacement of both the Test Booklet and the Answer Sheet.
13. Do not fold make any stray marks on the Answer Sheet.

Name of the Candidate (in Capital letters):______________________________________________________________

Roll Number: in figures

: in words ______________________________________________________________________________

Examination Centre Number:

Name of Examination Centre (in Capital letters):_______________________________________________________

Candidate’s Signature: _______________________ Invigilator’s Signature: ___________________


®

AIEEE 2011 SOLUTIONS 2

SOLUTIONS TO AIEEE 2011


PHYSICS: (CODE: Q)
PART – A

Note: Questions with (*) mark are from syllabus of class XI.
1. The transverse displacement y(x, t) of a wave on a string is given by y (x, t ) = e − (ax
2
+bt 2 + 2 ab xt ) . This represents a :
b
(1) wave moving in –x direction with speed (2) standing wave of frequency b
a

1 a
(3) standing wave of frequency (4) wave moving in +x direction with speed
b b
2

Sol.: y (x, t ) = e

( a x + bt ) , it is a function of type y = f (x + vt )

b
⇒ Speed of wave =
a

®
Correct choice: (1)

2. A screw gauge gives the following reading when used to measure the diameter of a wire.
Main scale reading : 0 mm
Circular scale reading : 52 divisions
Given that 1 mm on main scale corresponds to 100 divisions of the circular scale.
The diameter of wire from the above data is :
(1) 0.052 cm (2) 0.026 cm (3) 0.005 cm (4) 0.52 cm
1
Sol.: Diameter of wire = MSR + CSR × leastcount = 0 + × 52 = 0.052 cm
100
Correct choice: (1)

3. A mass m hangs with the help of a string wrapped around a pulley on a frictionless bearing. The pulley has mass m and radius
R. Assuming pulley to be a perfect uniform circular disc, the acceleration of the mass m, if the string does not slip on the
pulley, is :
2 g 3
(1) g (2) g (3) (4) g
3 3 2
mg mg 2mg 2 g
Sol.: a= = = =
 I  mR 2 3m 3
m + 2  m +
 R  2R 2
Correct choice: (2)

4. Work done in increasing the size of a soap bubble from a radius of 3 cm to 5 cm is nearly (Surface tension of soap
solution = 0.03 Nm–1)
(1) 0.2 π mJ (2) 2π mJ (3) 0.4π mJ (4) 4π mJ

Sol.: [( ) ]
W = 2T 4π 52 − (3) 10−4 = 2 × 0.03 × 4π[25 − 9]× 10 −4 J = 0.4π mJ
2

Correct choice: (3)

5. A thin horizontal circular disc is rotating about a vertical axis passing through its centre. An insect is at rest at a point near the
rim of the disc. The insect now moves along a diameter of the disc to reach its other end. During the journey of the insect, the
angular speed of the disc :
(1) continuously decreases (2) continuously increases
(3) first increases and then decreases (4) remains unchanged

Sol.: Since M.I. is first decreasing then increasing so from principle of conservation of angular momentum, angular speed, first
increase then decreases
Correct choice: (3)

Brilliant Tutorials Pvt. Ltd. Head Office: 12, Masilamani Street, T. Nagar, Chennai-600 017
Delhi Office: 50-C, Kalu Sarai (Behind Azad Apartments), New Delhi-110 016 Ph.: 2653 7392/93/94/95 Fax: 2653 7396
®

AIEEE 2011 SOLUTIONS 3

6. Two particles are executing simple harmonic motion of the same amplitude A and frequency ω along the x-axis. Their mean
position is separated by distance X0(X0 > A). If the maximum separation between them is (X0 + A), the phase difference
between their motion is :
π π π π
(1) (2) (3) (4)
3 4 6 2
Sol.: Let, x1 = A sin ωt and x 2 = A sin (ωt + φ )

 φ φ
x2 − x1 = 2 A cos ωt +  sin
 2  2
φ π
But 2 A sin = A ⇒ φ=
2 3
Correct choice: (1)

7. Two bodies of masses m and 4 m are placed at a distance r. The gravitational potential at a point on the line joining them
where the gravitational field is zero is :
4Gm 6Gm 9Gm
− − −

®
(1) (2) (3) (4) zero
r r r
Gm 4Gm 1 2
Sol.: = ⇒ = ∴ r − x = 2x
x2 (r − x )2 x r−x
r
x=
3
Gm 4Gm 9Gm
Gravitational potential V = − − =−
r 2r r
3 3
Correct choice: (3)

8. Two identical charged spheres suspended from a common point by two massless strings of length l are initially a distance
d(d << l) apart because of their mutual repulsion. The charge begins to leak from both the spheres at a constant rate. As a
result charges approach each other with a velocity v. Then as a function of distance x between them,
(1) v ∝ x–1 (2) v ∝ x1 / 2 (3) v ∝ x (4) v ∝ x −1 / 2

kq 2
Sol.: At any instant cos θ = mg sin θ
x2
θ
 x l l
⇒ q 2 ∝ x 2 tan θ  tan θ ≈ 
 2l 

⇒ q 2 ∝ x3
T
dq dx
⇒ q ∝ x2 Fe
dt dt x
⇒ v ∝ x −1/ 2 [Q q 2 ∝ x3 ]
Correct choice: (4) mg

9. A boat is moving due east in a region where the earth’s magnetic field is 5.0 × 10–5 NA–1 m–1 due north and horizontal. The
boat carries a vertical aerial 2 m long. If the speed of the boat is 1.50 ms–1, the magnitude of the induced emf in the wire of
aerial is :
(1) 0.75 mV (2) 0.50 mV (3) 0.15 mV (4) 1 mV

Sol.: emf = vBH l = 1.5 × 5 × 10 −5 × 2 = 15 × 10 −5 = 0.15 mV


Correct choice: (3)

Brilliant Tutorials Pvt. Ltd. Head Office: 12, Masilamani Street, T. Nagar, Chennai-600 017
Delhi Office: 50-C, Kalu Sarai (Behind Azad Apartments), New Delhi-110 016 Ph.: 2653 7392/93/94/95 Fax: 2653 7396
®

AIEEE 2011 SOLUTIONS 4

10. An object, moving with a speed of 6.25 m/s, is decelerated at a rate given by :
dv
= −2.5 v where v is the instantaneous speed. The time taken by the object, to come to rest, would be :
dt
(1) 2 s (2) 4 s (3) 8 s (4) 1 s
dv
Sol.: = −2.5 v
dt
0 t

∫ ∫
v −1 / 2 dv = −2.5 dt
6.25 0

t = 2 sec
Correct choice: (1)

11. A fully charged capacitor C with initial charge q0 is connected to a coil of self inductance L at t = 0. The time at which the
energy is stored equally between the electric and the magnetic fields is :
π
(1) LC (2) 2π LC (3) LC (4) π LC

®
4

1 2 1 q2 1
Sol.: Li = also q = q0 cos ωt and ω =
2 2 C LC
π
On solving t = LC
4
Correct choice: (1)

12. Let the x-z plane be the boundary between two transparent media. Medium 1 in z ≥ 0 has a refractive index of 2 and
r
medium 2 with z < 0 has a refractive index of 3 . A ray of light in medium 1 given by the vector A = 6 3iˆ + 8 3 ˆj − 10kˆ is
incident on the plane of separation. The angle of refraction in medium 2 is :
(1) 45o (2) 60o (3) 75o (4) 30o

Sol.: Angle of incidence is given by z

cos (π − i ) =
(
6 3iˆ + 8 3 ˆj − 10kˆ .kˆ )
20
1 i
− cos i = − µ= 2
2
∠i = 60 0 x

From Snell’s law, 2 sin i = 3 sin r µ= 3


∠ r = 45 0 r

Correct choice: (1)

13. A current I flows in an infinitely long wire with cross section in the form of a semi-circular ring of radius R. The magnitude
of the magnetic induction along its axis is :
µ0 I µ0 I µ0 I µ0 I
(1) (2) (3) (4)
2
2π R 2 π R 4 πR π2 R
dθ µ 0 2dI
Sol.: dI = I , dB =
π 4π R
π
µ0 I µ0 I dθ
Bnet =
∫ dB sin θ =
2π2 R ∫ sin θdθ =
π2 R
0 θ
x
Correct choice: (4)
dB

Brilliant Tutorials Pvt. Ltd. Head Office: 12, Masilamani Street, T. Nagar, Chennai-600 017
Delhi Office: 50-C, Kalu Sarai (Behind Azad Apartments), New Delhi-110 016 Ph.: 2653 7392/93/94/95 Fax: 2653 7396
®

AIEEE 2011 SOLUTIONS 5

14. A thermally insulated vessel contains an ideal gas of molecular mass M and ratio of specific heats γ. It is moving with speed v
and its suddenly brought to rest. Assuming no heat is lost to the surroundings, its temperature increases by :

(1)
(γ − 1) Mv 2 K (2)
γMv 2
K (3)
(γ − 1) Mv 2 K (4)
(γ − 1) Mv 2 K
2 γR 2R 2R 2(γ + 1)R
Sol.: Work done is zero
So loss in kinetic energy = change in internal energy of gas
1 2 R
mv = nCv ∆T = n ∆T
2 γ −1

1 2 m R
mv = ∆T
2 M γ −1

Mv 2 (γ − 1)
∴ ∆T = K
2R

®
Correct choice: (3)

15. A mass M, attached to a horizontal spring, executes S.H.M. with amplitude A1. When the mass M passes through its mean
A 
position then a smaller mass m is placed over it and both of them move together with amplitude A2. The ratio of  1  is :
 A2 
12 12
M +m  M  M +m M
(1) (2)   (3)   (4)
M M +m  M  M +m

Sol.: At mean point Fnet = 0, so linear momentum must be conserved


Mv1 = (M + m )v2

K K
MA1 = (M + m )A2
M m+M

A1 M = A2 M + m

A1 m+M
∴ =
A2 M

Correct choice: (3)

16. Water is flowing continuously from a tap having an internal diameter 8 × 10–3 m. The water velocity as it leaves the tap is
0.4 ms–1. The diameter of the water stream at a distance 2 × 10–1 m below the tap is close to :
(1) 7.5 × 10–3 m (2) 9.6 × 10–3 m (3) 3.6 × 10–3 m (4) 5.0 × 10–3 m
Sol.: From Bernoulli’s theorem
1 2 1
P0 + ρv1 + ρgh = P0 + ρv22 + 0
2 2

v2 = v12 + 2 gh = 0.16 + 2 × 10 × 0.2 = 2.03 m/s

From continuity equation A2 v2 = A1v1

D22 D2
π × v2 = π 1 v1
4 4

v1
⇒ D2 = D1 = 3.55 × 10 −3 m
v2

Correct choice: (3)

Brilliant Tutorials Pvt. Ltd. Head Office: 12, Masilamani Street, T. Nagar, Chennai-600 017
Delhi Office: 50-C, Kalu Sarai (Behind Azad Apartments), New Delhi-110 016 Ph.: 2653 7392/93/94/95 Fax: 2653 7396
®

AIEEE 2011 SOLUTIONS 6

17. This question has Statement – 1 and Statement – 2. Of the four choices given after the statements, choose the one that best
describes the two statements.
Statement – 1 :
Sky wave signals are used for long distance radio communication. These signals are in general, less stable than ground wave
signals.
Statement – 2 :
The state of ionosphere varies from hour to hour, day to day and season to season.
(1) Statement – 1 is true, Statement – 2 is true, Statement – 2 is the correct explanation of Statement – 1.
(2) Statement – 1 is true, Statement – 2 is true, Statement – 2 is not the correct explanation of Statement – 1.
(3) Statement – 1 is false, Statement – 2 is true.
(4) Statement – 1 is true, Statement – 2 is false.
Sol.: Correct choice: (2)

18. Three perfect gases at absolute temperatures T1, T2 and T3 are mixed. The masses of molecules are m1, m2 and m3 and the
number of molecules are n1, n2 and n3 respectively. Assuming no loss of energy, the final temperature of the mixture is :

(1)
n1T1 + n2T2 + n3T3
(2)
n1T12 + n2T22 + n3T32
(3)
n 12T12 + n22T22 + n32T32
(4)
(T1 + T2 + T3 )
n1 + n2 + n3 n1T1 + n2T2 + n3T3 n1T1 + n2T2 + n3T3 3

®
Sol.: If atomicity of gas is same
f
So (n1 + n2 + n3 )kT = f n1kT1 + f n2 kT2 + f n3kT3
2 2 2 2
n1T1 + n2T2 + n3T3
∴ T=
n1 + n2 + n3
Correct choice: (1)

19. A pulley of radius 2 m is rotated about its axis by a force F = (20 t – 5t2) Newton (where t is measured in seconds) applied
tangentially. It the moment of inertia of the pulley about its axis of rotation is 10 kg-m2 the number of rotations made by the
pulley before its direction of motion if reserved, is:
(1) more than 3 but less than 6 (2) more than 6 but less than 9
(3) more than 9 (4) less than 3
Sol.: F = 20t − 5t 2
ω t

∫ (4t − t )dt
FR dω

2
∴ α= = 4t − t 2 ⇒ = 4t − t 2 ⇒ dω =
I dt
0 0
3
t
⇒ ω = 2t 2 − (as ω = 0 at t = 0, 6s)
3
θ 6
 t 3  36
∫ ∫  2t
2
dθ = − dt ⇒ θ = 36 rad ⇒ n= <6
3  2π
0 0
Correct choice: (1)

20. A resistor ‘R’ and 2µF capacitor in series in connected through a switch to 200 V direct supply. Across the capacitor is a
neon bulb that lights up at 120 V. Calculate the value of R to make the bulb light up 5 s after the switch has been closed.
(log10 2.5 = 0.4)
(1) 1.7 × 105 Ω (2) 2.7 × 106 Ω (3) 3.3 × 107 Ω (4) 1.3 × 104 Ω

Sol.: (
Vc = Vmax 1 − e −t / RC ; ) ( )
120 = 200 1 − e −t / RC ⇒ t = RC ln(2.5) ⇒ R = 2.71 × 10 6 Ω
Correct choice: (2)
1
21. A Carnot engine operating between temperatures T1 and T2 has efficiency . When T2 is lowered by 62 K its efficiency
6
1
increases to . Then T1 and T2 are, respectively:
3
(1) 372 K and 330 K (2) 330 K and 268 K (3) 310 K and 248 K (4) 372 K and 310 K

Brilliant Tutorials Pvt. Ltd. Head Office: 12, Masilamani Street, T. Nagar, Chennai-600 017
Delhi Office: 50-C, Kalu Sarai (Behind Azad Apartments), New Delhi-110 016 Ph.: 2653 7392/93/94/95 Fax: 2653 7396
®

AIEEE 2011 SOLUTIONS 7

T2
Sol.: Efficiency of engine η = 1 −
T1
T2 5
⇒ = … (i)
T1 6
T2 − 62 1
η2 = 1 − = … (ii)
T1 3
5
Solving we get, T1 = 372 K and T2 = × 372 = 310 K
6
Correct choice: (4)

22. If a wire is stretched to make it 0.1% longer, its resistance will :


(1) increase by 0.2% (2) decrease by 0.2% (3) decrease by 0.05% (4) increase by 0.05%

®
ρl ρl 2
Sol.: Resistance of wire R = = (volume constant or Al = C )
A C
∆R ∆l
∴ Fractional change in resistance =2
R l
Correct choice: (1)

23. This question has a paragraph followed by two statements, Statement – 1 and Statement – 2. Of the given four alternatives
after the statements, choose the one that describes the statements.
A thin air film is formed by putting the convex surface of a plane-convex lens over a plane glass plate. With monochromatic
light, this film gives an interference pattern due to light reflected from the top (convex) surface and the bottom (glass plate)
surface of the film.
Statement – 1 : When light reflects from the air-glass plate interface, the reflected wave suffers a phase change of π.
Statement – 2 : The centre of the interference pattern is dark.
(1) Statement – 1 is true, Statement – 2 is true, Statement – 2 is the correct explanation of Statement – 1.
(2) Statement – 1 is true, Statement – 2 is true, Statement – 2 is not the correct explanation of Statement – 1.
(3) Statement – 1 is false, Statement – 2 is true.
(4) Statement – 1 is true, Statement – 2 is false.
Sol.: Correct choice: (2)

24. A car is fitted with a convex side-view mirror of focal length 20 cm. A second car 2.8 m behind the first car is overtaking the
first car at a relative speed of 15 m/s. The speed of the image of the second car as seen in the mirror of the first one is :
1 1
(1) m/s (2) 10 m/s (3) 15 m/s (4) m/s
15 10
Sol.: From mirror formulae
2
1 1 1 dv v 2  du  dv  f  du dv 1
+ = so, =− 2  ⇒ = −  ⇒ = m/s
v u f dt u  dt  dt  u − f  dt dt 15
Correct choice: (1)

25. Energy required for the electron excitation in Li++ from the first to the third Bohr orbit is :
(1) 36.3 eV (2) 108.8 eV (3) 122.4 eV (4) 12.1 eV
2 1 1 
Sol.: ∆E = 13.6(3)  2 − 2  = 108.8 eV
1 3 
Correct choice: (2)

Brilliant Tutorials Pvt. Ltd. Head Office: 12, Masilamani Street, T. Nagar, Chennai-600 017
Delhi Office: 50-C, Kalu Sarai (Behind Azad Apartments), New Delhi-110 016 Ph.: 2653 7392/93/94/95 Fax: 2653 7396
®

AIEEE 2011 SOLUTIONS 8

26. The electrostatic potential inside a charged spherical ball is given by φ = ar2 + b where r is the distance from the centre a, b
are constants. Then the charge density inside the ball is :
(1) –6aε0r (2) –24πaε0 (3) –6aε0 (4) –24πaε0r

Sol.: Electric field E = − = −2ar … (i)
dr
By Gauss’s theorem
1 q
E= … (ii)
4πε0 r 2
From (i) and (ii) q = −8πε0 ar 3 , dq = −24πε 0 ar 2 dr
dq
Charge density ρ = = −6ε 0 a
4πr 2 dr
Correct choice: (3)

27. A water fountain on the ground sprinkles water all around it. If the speed of water coming out of the fountain is v, the total
area around the fountain that gets wet is :
v4 π v4 v2 v2
(1) π (2) (3) π (4) π

®
g2 2 g2 g2 g
2
Sol.: Total area around fountain A = πRmax
v 2 sin 90 0 v 2 v4
Where Rmax = = ∴ A=π
g g g2
Correct choice: (1)

28. 100g of water is heated from 30oC to 50oC. Ignoring the slight expansion of the water, the change in its internal energy is
(specific heat of water is 4184 J/kg/K):
(1) 8.4 kJ (2) 84 kJ (3) 2.1 kJ (4) 4.2 kJ
Sol.: ∆U = ∆Q = mc∆T = 100 × 10 −3 × 4184(50 − 30 ) ≈ 8.4 kJ
Correct choice: (1)

2
29. The half life of a radioactive substance is 20 minutes. The approximate time interval (t2 – t1) between the time t2 when of it
3
1
has decayed and time t1 when of it had decayed is :
3
(1) 14 min (2) 20 min (3) 28 min (4) 7 min
N0 − λt 2
Sol.: No of undecayed atom after time t2 ; = N 0e … (i)
3
2N0
Number of undecayed atom after time t1 ; = N 0 e −λt1 … (ii)
3
Solving (i) and (ii) t 2 − t1 = 20 min
Correct choice: (2)

30. This question has Statement – 1 and Statement – 2. Of the four choices given after the statements, choose the one that best
describes the two statements.
Statement – 1:
A metallic surface is irradiated by a monochromatic light of frequency v > v0 (the threshold frequency). The maximum
kinetic energy and the stopping potential are Kmax and V0 respectively. If the frequency incident on the surface is doubled,
both the Kmax and V0 are also doubled.
Statement – 2 :
The maximum kinetic energy and the stopping potential of photoelectrons emitted from a surface are linearly dependent on
the frequency of incident light.
(1) Statement – 1 is true, Statement – 2 is true, Statement – 2 is the correct explanation of Statement – 1.
(2) Statement – 1 is true, Statement – 2 is true, Statement – 2 is not the correct explanation of Statement – 1.
(3) Statement – 1 is false, Statement – 2 is true.
(4) Statement – 1 is true, Statement – 2 is false.

Sol.: Kmax = eV0 = hv – hv0


When v is doubled, Kmax and V0 become more that double.
Correct choice: (3)

Brilliant Tutorials Pvt. Ltd. Head Office: 12, Masilamani Street, T. Nagar, Chennai-600 017
Delhi Office: 50-C, Kalu Sarai (Behind Azad Apartments), New Delhi-110 016 Ph.: 2653 7392/93/94/95 Fax: 2653 7396
®

AIEEE 2011 SOLUTIONS 9

SOLUTIONS TO AIEEE 2011


MATHEMATICS (CODE: Q)
PART – B

Note: Questions with (*) mark are from syllabus of class XI.
*31. The lines L1 : y − x = 0 and L2 : 2 x + y = 0 intersect the line L3 : y + 2 = 0 at P and Q respectively. The bisector of the acute
angle between L1 and L2 intersects L3 at R.

Statement-1: The ratio PR : RQ equals 2 2 : 5


Statement-2: In any triangle, bisector of an angle divides the triangle into two similar triangles.
(1) Statement-1 is true, Statement-2 is true; Statement-2 is not a correct explanation for Statement-1.
(2) Statement-1 is true, Statement-2 is false.
(3) Statement-1 is false, Statement-2 is true.
(4) Statement-1 is true, Statement-2 is true; Statement-2 is a correct explanation for Statement-1.

P = (− 2, − 2 )

®
Sol.:
y=x
Q = (1, − 2 )

PR AP 2 2 A(0, 0)
= =
RQ AQ 5 y=–2
P R Q
But statement 2 is not always true
y = – 2x
Correct choice: (2)

*32. If A = sin 2 x + cos 4 x, then for all real x :


13 3 13 3
(1) ≤ A ≤1 (2) 1 ≤ A ≤ 2 (3) ≤ A≤ (4) ≤ A ≤1
16 4 16 4

Sol.: (
A = 1 − sin 2 x )
2
+ sin 2 x
= sin 4 x − sin 2 x + 1
2
 1 3
=  sin 2 x −  +
 2 4

0 ≤ sin 2 x ≤ 1
2
 1 1
0 ≤  sin 2 x −  ≤
 2  4
3
≤ A ≤1 .
4
Correct choice: (4)

*33. The coefficient of x 7 in the expansion of (1 − x − x 2 + x 3 ) 6 is


(1) –132 (2) –144 (3) 132 (4) 144

Sol.: (1 − x − x 2
+x 3 6
) 6
(
= (1 − x ) 1 − x )
2 6

 6 
=
 ∑ 6 r

(
C r (− x )  1 − 6 x 2 + 15 x 4 − 20 x 6 + ... )
 r =0 

= −6 × 6 C 5 × (− 1)5 + 15 × 6 C 3 × (− 1)3 − 20 × 6 C1 × (− 1)1

= 36 − 300 + 120 = −144


Correct choice: (2)

Brilliant Tutorials Pvt. Ltd. Head Office: 12, Masilamani Street, T. Nagar, Chennai-600 017
Delhi Office: 50-C, Kalu Sarai (Behind Azad Apartments), New Delhi-110 016 Ph.: 2653 7392/93/94/95 Fax: 2653 7396
®

AIEEE 2011 SOLUTIONS 10

 1 − cos{2( x − 2)} 
34. lim  
x → 2 x−2 
 
1
(1) equals 2 (2) equals – 2 (3) equals (4) does not exist
2

1 − cos 2(x − 2) 2 | sin (x − 2) |


Sol.: lim = lim
x →2 x−2 x→2 x−2

LHL = − 2
RHL = 2
Correct choice: (4)

*35. Statement-1: The number of ways of distributing 10 identical balls in 4 distinct boxes such that no box is empty is 9 C 3 .

Statement-2: The number of ways of choosing any 3 places from 9 different places is 9 C 3 .

®
(1) Statement-1 is true, Statement-2 is true; Statement-2 is not a correct explanation for Statement-1.
(2) Statement-1 is true, Statement-2 is false.
(3) Statement-1 is false, Statement-2 is true.
(4) Statement-1 is true, Statement-2 is true; Statement-2 is a correct explanation for Statement-1.
Sol.: x1 + x 2 + x3 + x 4 = 10, xi ≥ 1, i = 1, 2, 3, 4
10 −1
Number of ways C 4 −1 = 9 C 3
Correct choice: (1)

d 2x
36. equals :
dy 2
−1 −1
 d 2 y   dy  −3  d 2 y  dy  −2  d 2 y  dy  −3 d2y
(1) −  2    (2)  
 dx 2  dx  (3) −  2   (4)  
 dx 2 
 dx   dx     dx  dx   
d 2x d  dx  d  dx  dx
Sol.: =  =  
dy 2 dy  dy  dx  dy  dy

d  1  dx
=  
dx  dy / dx  dy

1 d2y
=− 32
 dy  dx
 
 dx 
Correct choice: (3)

dy
37. If = y + 3 > 0 and y (0 ) = 2, then y (ln 2 ) is equal to :
dx
(1) 5 (2) 13 (3) –2 (4) 7
dy
Sol.: = y+3
dx
dy
∫ y+3 ∫
= dx ⇒ ln ( y + 3) = x + c

ln 5 = 0 + c
∴ ln ( y + 3) = x + ln 5
ln ( y + 3) = ln 2 + ln 5 ⇒ y + 3 = 10 ⇒ y = 7
Correct choice: (4)

Brilliant Tutorials Pvt. Ltd. Head Office: 12, Masilamani Street, T. Nagar, Chennai-600 017
Delhi Office: 50-C, Kalu Sarai (Behind Azad Apartments), New Delhi-110 016 Ph.: 2653 7392/93/94/95 Fax: 2653 7396
®

AIEEE 2011 SOLUTIONS 11

*38. Let R be the set of real nubmers.


Statement-1: A = {(x, y ) ∈ R × R : y − x is an integer} is an equivalence relation on R.
Statement-2: B = {(x, y )∈ R × R : x = αy for some rational number α} is an equivalence relation on R.
(1) Statement-1 is true, Statement-2 is true; Statement-2 is not a correct explanation for Statement-1.
(2) Statement-1 is true, Statement-2 is false.
(3) Statement-1 is false, Statement-2 is true.
(4) Statement-1 is true, Statement-2 is true; Statement-2 is a correct explanation for Statement-1.

Sol.: Let for statement 1 xRy = x − y ∈ I . As xRx is an integer and yRx as well as xRz (for xRy and yRz ) is also an integer
Hence equivalence.
1
Similarly as x = αy hence α = 1 for reflexive and being a rational for symmetric for some non zero α and product of
α
rationals also being rational ⇒ equivalence
But not symmetric because of α = 0 case

®
Both relations are equivalence but not the correct explanation
Correct choice: (1)

8 log(1 + x )
1

39. The value of



0
1+ x2
dx is

π π
(1) log 2 (2) log 2 (3) log 2 (4) π log 2
8 2
log(1 + x )
1

Sol.: I =8

0
1+ x2
dx

Put x = tan θ
π/4

I =8
∫ log(1 + tan θ)dθ
0
π
=8× log 2 = π log 2
8
Correct choice: (4)

*40. Let α, β be real and z be a complex number. If z 2 + αz + β = 0 has two distinct roots on the line Re z = 1, then it is
necessary that :
(1) β ∈ (−1, 0) (2) | β | = 1 (3) β ∈ (1, ∞ ) (4) β ∈ (0, 1)
Sol.: z 2 + αz + β = 0
Since the coefficients are real, complex roots must occur in conjugate pairs.
Let the roots be 1 + ix, 1 − ix, x ≠ 0 . β = (1 + ix )(1 − ix ) = 1 + x 2 > 1
Correct choice: (3)

41. Consider 5 independent Bernoulli’s trials each with probability of success p. If the probability of at least one failure is
31
greater than or equal to , then p lies in the interval
32
 3 11   1  11   1 3
(1)  ,  (2) 0,  (3)  , 1 (4)  , 
 4 12   2  12   2 4
Sol.: P ( X = r ) = 5 C r p r q 5− r
31 1
1 − P ( X = 5) ≥ ⇒ p5 ≤
32 32
1
p≤
2
Correct choice: (2)

Brilliant Tutorials Pvt. Ltd. Head Office: 12, Masilamani Street, T. Nagar, Chennai-600 017
Delhi Office: 50-C, Kalu Sarai (Behind Azad Apartments), New Delhi-110 016 Ph.: 2653 7392/93/94/95 Fax: 2653 7396
®

AIEEE 2011 SOLUTIONS 12

*42. A man saves Rs. 200 in each of the first three months of his service. In each of the subsequent months his saving increases by
Rs. 40 more than the saving of immediately previous month. His total saving from the start of service will be Rs. 11040 after
(1) 19 months (2) 20 months (3) 21 months (4) 18 months
Sol.: 200 + 200 + 200 + (240 + 280 + ...n terms ) = 11040
n = 18
∴ 21 months
Correct choice: (3)

1
43. The domain of the function f ( x) = is
| x| − x
(1) (0, ∞ ) (2) (− ∞, 0) (3) (− ∞, ∞ ) − {0} (4) (− ∞, ∞ )
Sol.: | x | −x > 0
| x| > x
⇒ x<0
Correct choice: (2)

®
y −1 z − 3  5 
44. If the angle between the line x = = and the plane x + 2 y + 3z = 4 is cos −1  , then λ equals
2 λ  14 
 
3 2 5 2
(1) (2) (3) (4)
2 5 3 3
1×1 + 2 × 2 + λ × 3
Sol.: sin θ =
5 + λ2 14
5 5 + 3λ
sin cos −1 =
14 14 5 + λ2
2
λ=
3
Correct choice: (4)

45.
r
If a =
1
10
( r 1
) ( ) r r r r
( r r
) [(
3iˆ + kˆ and b = 2iˆ + 3 ˆj − 6kˆ , then the value of 2a − b . a × b × a + 2b is
7
)( )]
(1) –3 (2) 5 (3) 3 (4) –5
Sol.: ( r r r r r
)[( ) ( )]
2a − b . a × b × a + 2b
r
r
( r r
)[( ) ( )]
r r r
= b − 2a . a + 2b × a × b
r
( r r r r
)[ ( ) ( )]
r r r
= b − 2a . a × a × b + 2b × a × b
r
( r rrr rr r
)[( ) ( ) ( )]
rrr rrr
= b − 2a . a.b a − (a.a )b + 2 b .b a − 2 b .a b
r
( r r
)(r
) ( )( )
r r r r
= b − 2a . − b + 2a = − 2 a − b . 2a − b
( r r
= − 4 + 1 − 4a.b = −5 )
Correct choice: (4)

*46. Equation of the ellipse whose axes are the axes of coordinates and which passes through the point (–3, 1) and has eccentricity
2
is
5
(1) 5 x 2 + 3 y 2 − 48 = 0 (2) 3 x 2 + 5 y 2 − 15 = 0 (3) 5 x 2 + 3 y 2 − 32 = 0 (4) 3 x 2 + 5 y 2 − 32 = 0
x2 y2
Sol.: 2
+ =1
a b2
 2
As b 2 = a 2 1 −  …(i)
 5
9 1
+ =1 …(ii)
a2 b2
9 5
+ 2 =1
a 2
a (3)

Brilliant Tutorials Pvt. Ltd. Head Office: 12, Masilamani Street, T. Nagar, Chennai-600 017
Delhi Office: 50-C, Kalu Sarai (Behind Azad Apartments), New Delhi-110 016 Ph.: 2653 7392/93/94/95 Fax: 2653 7396
®

AIEEE 2011 SOLUTIONS 13

1  5 2 32
9 +  = 1 ⇒ a =
a2  3 3
32 3 32
b2 = × =
3 5 5
⇒ 3 x 2 + 5 y 2 − 32 = 0 .
There is one more case of vertical ellipse which gives 5 x 2 + 3 y 2 − 48 = 0 . So two options are correct.
Correct choice: (1) and (4)

47. Let I be the purchase value of an equipment and V (t ) be the value after it has been used for t years. The value V (t )
dV (t )
depreciates at a rate given by differential equation = −k (T − t ), where k > 0 is a constant and T is the total life in
dt
years of the equipment. Then the scrap value V (T ) of the equipment is
kT 2 k (T − t )2 I
(1) I− (2) I− (3) e − kT (4) T 2 −
2 2 k
dV (t )
Sol.: + kT − kt = 0

®
dt
dV (t )
− kt = −kT ⇒ LDE (at t = 0, V = I )
dt
kT 2
⇒ V (T ) = I − (scrap value obtained by putting t = T )
2
Correct choice: (1)

r r r r r r r r r r
48. The vectors a and b are not perpendicular and c and d are two vectors satisfying b × c = b × d and a . d = 0. Then the
r
vector d is equal to
rr r r
r  a.c  r r  b .cr  r rr
r  a.c  r r  b .cr  r
(1) c +  r r b 
(2) b + r r c  (3) c −  r r b (4) b − r r c

 a.b   a.b   a.b   a.b 
   
rr
Sol.: a.b ≠ 0
r r r r rr
b × c = b × d and a.d = 0
r
( ) ( )
r r r r r
⇒ a× b ×c =a× b ×d
( ) ( ) ( )
rr r rrr r r r rr r
⇒ (a.c )b − a.b c = a.d b − a.b d
( )
rr r
( )rr r rrr
⇒ a.b d = −(a.c )b + a.b c
r r  ar.cr  r
d = c −  r r b
 a.b 
Correct choice: (3)

*49. The two circles x 2 + y 2 = ax and x 2 + y 2 = c 2 (c > 0 ) touch each other if


(1) | a | = c (2) a = 2c (3) | a | = 2c (4) 2 | a | = c
Sol.:
y

(0, 0)
x
(–|a|/2, 0) (|a|/2, 0)

⇒ a =C
Correct choice: (1)

Brilliant Tutorials Pvt. Ltd. Head Office: 12, Masilamani Street, T. Nagar, Chennai-600 017
Delhi Office: 50-C, Kalu Sarai (Behind Azad Apartments), New Delhi-110 016 Ph.: 2653 7392/93/94/95 Fax: 2653 7396
®

AIEEE 2011 SOLUTIONS 14

50. If C and D are two events such that C ⊂ D and P(D ) ≠ 0, then the correct statement among the following is
P (D )
(1) P(C | D ) ≥ P(C ) (2) P(C | D ) < P(C ) (3) P(C | D ) = (4) P(C | D ) = P(C )
P(C )
Sol.: In this case
 C  P(C ∩ D ) P(C )
P  = =
D P (D ) P (D )

C
where P(D) ≤ 1 hence P  ≥ P (C )
D
Correct choice: (1)

51. The number of values of k for which the linear equations 4 x + ky + 2 z = 0 , kx + 4 y + z = 0 and 2 x + 2 y + z = 0 possess a
non-zero solution is
(1) 2 (2) 1 (3) zero (4) 3
Sol.: ∆=0

®
4 k 2
⇒ k 4 1 = 0 ⇒ 4(4 – 2) – k (k – 2) + 2(2k – 8) = 0
2 2 1
⇒ 8 – k2 + 2k + 4k – 16 = 0
k2 – 6k + 8 = 0 ⇒ (k – 4)(k – 2) = 0, k = 4, 2
Correct choice: (1)

*52. Consider the following statements


P : Suman is brilliant
Q : Suman is rich
R : Suman is honest
The negation of the statement “Suman is brilliant and dishonest if and only if Suman is rich” can be expressed as
(1) ~ (Q ↔ (P ^ ~ R )) (2) ~ Q ↔~ P ^ R (3) ~ (P ^ ~ R ) ↔ Q (4) ~ P ^ (Q ↔ ~ R )
Sol.: Correct choice: (3)

*53. The shortest distance between line y − x = 1 and curve x = y 2 is

3 2 8 4 3
(1) (2) (3) (4)
8 3 2 3 4
Sol.: y = mx – 2am – am3 as m = –1
1
y= −x−2 (− 1) − 1 (− 1)3
4 4
1 1 3
y = −x ++ = −x
2 4 4
Common normal
3  1 7 1 1
y= − x ⇒ AB ⇒ A =  − ,  ; B =  , 
4  8 8  4 2
3
= 2
8
Correct choice: (1)

*54. If the mean deviation about the median of the numbers a, 2a, .......,50a is 50, then | a | equals
(1) 3 (2) 4 (3) 5 (4) 2
25a + 26a 51a | xi − M |
Sol.: Median (M) =
2
=
2
and Now M .D. = ∑ n
Correct choice: (2)

Brilliant Tutorials Pvt. Ltd. Head Office: 12, Masilamani Street, T. Nagar, Chennai-600 017
Delhi Office: 50-C, Kalu Sarai (Behind Azad Apartments), New Delhi-110 016 Ph.: 2653 7392/93/94/95 Fax: 2653 7396
®

AIEEE 2011 SOLUTIONS 15

x y −1 z − 2
55. Statement-1: The point A(1, 0, 7) ) is the mirror image of the point B(1, 6, 3) in the line : = = .
1 2 3
x y −1 z − 2
Statement-2: The line = = bisects the line segment joining A(1, 0, 7 ) and B(1, 6, 3) .
1 2 3
(1) Statement-1 is true, Statement-2 is true; Statement-2 is not a correct explanation for Statement-1.
(2) Statement-1 is true, Statement-2 is false.
(3) Statement-1 is false, Statement-2 is true.
(4) Statement-1 is true, Statement-2 is true; Statement-2 is a correct explanation for Statement-1.
x y −1 z − 2
Sol.: As the line = = is the right bisector of line segment joining (1, 0, 7) with (1, 6, 3).
1 2 3
But right bisector is not mentioned in statement 2.
Correct choice: (1)

®
56. Let A and B be two symmetric matrices of order 3.
Statement-1: A(BA) and ( AB )A are symmetric matrices.
Statement-2: AB is symmetric matrix if matrix multiplication of A with B is commutative.
(1) Statement-1 is true, Statement-2 is true; Statement-2 is not a correct explanation for Statement-1.
(2) Statement-1 is true, Statement-2 is false.
(3) Statement-1 is false, Statement-2 is true.
(4) Statement-1 is true, Statement-2 is true; Statement-2 is a correct explanation for Statement-1.
Sol.: ( A(BA))T = (BA)T AT ( )
= AT B T AT = ( AB )A = A(BA)
Correct choice: (1)

If ω(≠ 1) is a cube root of unity, and (1 + ω) = A + Bω. Then ( A, B ) equals


7
*57.

(1) (1, 1) (2) (1, 0) (3) (–1, 1) (4) (0, 1)


7
Sol.: ⇒ (1 + ω) = A + Bω

(− ω ) 2 7
=1+ ω
A = 1, B = 1
Correct choice: (1)

 sin ( p + 1)x + sin x


 , x<0
 x
58. The values of p and q for which the function f ( x) =  q , x = 0 is continuous for all x in R, are

 x + x2 − x
, x>0
 x3/ 2
5 1 3 1 1 3 1 3
(1) p= , q= (2) p=− , q= (3) p= , q= (4) p = , q=−
2 2 2 2 2 2 2 2
Sol.: LHL:-
sin{( p + 1)(− h )} − sinh
lim f (x ) = lim = p +1+1= p + 2
x →0 − h →0 −h
RHL:-
1 1
lim f (x ) = lim = = (by rationalization)
x →0 + h →0 1+1 2
3 1
f(0) = q ⇒ p = − , q =
2 2
Correct choice: (2)

Brilliant Tutorials Pvt. Ltd. Head Office: 12, Masilamani Street, T. Nagar, Chennai-600 017
Delhi Office: 50-C, Kalu Sarai (Behind Azad Apartments), New Delhi-110 016 Ph.: 2653 7392/93/94/95 Fax: 2653 7396
®

AIEEE 2011 SOLUTIONS 16

1
59. The area of the region enclosed by the curves y = x, x = e, y = and the positive x-axis is
x
3 5 1
(1) 1 square unit (2) square units (3) square units (4) square unit
2 2 2

Sol.: Required region AOBC:- x=e


y=x
1 e
1 y
∫0
xdx +

1
x
dx

1 3 (1, 1)
= +1 =
2 2
Correct choice: (2)  1
B  e, 
e
O 
C x

®
x
 5π 
60. For x ∈  0,

 , define f (x ) =
2  ∫
0
t sin t dt . Then f has

(1) local minimum at π and 2π


(2) local minimum at π and local maximum at 2π
(3) local maximum at π and local minimum at 2π
(4) local maximum at π and 2π

Sol.: f ′(x ) = x sin x


f ′(x ) = 0
⇒ x = 0 or sinx = 0
⇒ x = 2π, π
1
f ′′(x ) = x cos x + sin x
2 x
1
⇒ (2 x cos x + sin x )
2 x
at x = π ⇒ f ′′(x ) < 0 ⇒ maxima

at x = 2π ⇒ f ′′(x ) > 0 ⇒ minima


Correct choice: (3)

Brilliant Tutorials Pvt. Ltd. Head Office: 12, Masilamani Street, T. Nagar, Chennai-600 017
Delhi Office: 50-C, Kalu Sarai (Behind Azad Apartments), New Delhi-110 016 Ph.: 2653 7392/93/94/95 Fax: 2653 7396
®

AIEEE 2011 SOLUTIONS 17

SOLUTIONS TO AIEEE 2011


CHEMISTRY: (CODE: Q)
PART – C

Note: Questions with (*) mark are from syllabus of class XI.
*61. Among the following the maximum covalent character is shown by the compound:
(1) SnCl2 (2) AlCl3 (3) MgCl2 (4) FeCl2
Sol.: Higher charge density of cation favours covalency.
Correct choice: (2)

62. The presence or absence of hydroxy group on which carbon atom of sugar differentiates RNA and DNA?
(1) 2nd (2) 3rd (3) 4th (4) 1st
nd
Sol.: RNA has hydroxy group attached at the 2 C-atom while DNA does not.
Correct choice: (1)

63. Trichloroacetaldehyde was subjected to Cannizzaro’s reaction by using NaOH. The mixture of the products contains sodium

®
trichloroacetate and another compound. The other compound is:
(1) Trichloromethanol (2) 2,2,2-Trichloropropanol
(3) Chloroform (4) 2,2,2- Trichloroethanol
Cl Cl Cl
Sol.: Cl–C–C–H Cl–C–COONa + Cl–C–CH2OH
NaOH
Cl O Cl Cl
Correct choice: (4)

64. Sodium ethoxide has reacted with ethanoyl chloride. The compound that is produced in the above reaction is:
(1) 2-Butanone (2) Ethyl chloride (3) Ethyl ethanoate (4) Diethyl ether
.. +
Sol.: CH3–C–Cl + C2H5ONa .. CH3–C–OC2H5 + NaCl
O O
Correct choice: (3)

65. The reduction potential of hydrogen half-cell will be negative if:


(1) p(H2) = 1 atm and [H+] = 1.0 M (2) p(H2) = 2 atm and [H+] = 1.0 M
(3) p(H2) = 2 atm and [H+] = 2.0 M (4) p(H2) = 1 atm and [H+] = 2.0 M
Sol.: Pt | H2(g) | H+(aq)
H+ + e– → ½H2(g)
1/ 2
0.059 (PH 2 ) 0.059 21/ 2
E = E° – log ; E=0– log = –ve.
1 [H + ] 1 1
Correct choice: (2)
*66. The strongest acid amongst the following compounds is:
(1) HCOOH (2) CH3CH2CH(Cl)CO2H (3) ClCH2CH2CH2COOH (4) CH3COOH
Sol.: CH3CH2CH(Cl)CO2H is the strongest acid due to –I effect of Cl.
Correct choice: (2)
67. The degree of dissociation (α) of a weak electrolyte, AxBy is related to van’t Hoff factor (i) by the expression:
i −1 x + y −1 x + y +1 i −1
(1) α = (2) α = (3) α = (4) α =
x + y +1 i −1 i −1 ( x + y − 1)
Sol.: AxBy xAy+ + yBx–
Moles at
equilibrium 1–α xα yα
i = 1 – α + xα + yα
(i – 1) = α(x + y – 1)
i −1
α=
x + y −1
Correct choice: (4)

Brilliant Tutorials Pvt. Ltd. Head Office: 12, Masilamani Street, T. Nagar, Chennai-600 017
Delhi Office: 50-C, Kalu Sarai (Behind Azad Apartments), New Delhi-110 016 Ph.: 2653 7392/93/94/95 Fax: 2653 7396
®

AIEEE 2011 SOLUTIONS 18

*68. ‘a’ and ‘b’ are van der Waal’s constants for gases. Chlorine is more easily liquefied than ethane because
(1) a and b for Cl2 < a and b for C2H6. (2) a for Cl2 < a for C2H6 but b for Cl2 > b for C2H6
(3) a for Cl2 > a for C2H6 but b for Cl2 < b for C2H6 (4) a and b for Cl2 > a and b for C2H6
Sol.: van der Waal’s constant ‘a’ is a measure of intermolecular force of attraction while ‘b’ is a measure of size of the gaseous
molecule.
Correct choice: (3)

*69. A vessel at 1000 K contains CO2 with a pressure of 0.5 atm. Some of the CO2 is converted into CO on the addition of
graphite. If the total pressure at equilibrium is 0.8 atm, the value of K is:
(1) 3 atm (2) 0.3 atm (3) 0.18 atm (4) 1.8 atm
Sol.: CO2(g) + C(s) 2CO(g)
0.5 – x 2x
0.5 – x + 2x = 0.8
x = 0.3
0.6 × 0.6
K= = 1.8 atm.

®
0 .2
Correct choice: (4)

*70. Boron cannot form which one of the following anions?



(1) BH −4 (2) B(OH) 4 (3) BO −2 (4) BF63−
Sol.: Boron cannot form BF63− due to non availability of d-orbital.
Correct choice: (4)

71. Which of the following facts about the complex [Cr(NH3)6]Cl3 is wrong?
(1) The complex is paramagnetic.
(2) The complex is an outer orbital complex.
(3) The complex gives white precipitate with silver nitrate solution.
(4) The complex involves d2sp3 hybridisation and is octahedral in shape.
Sol.: The complex involves d2sp3 hybridisation, and hence it is an inner orbital complex.
Correct choice: (2)

72. Ethylene glycol is used as an antifreeze in a cold climate. Mass of ethylene glycol which should be added to 4 kg of water to
prevent it from freezing at –6°C will be: (Kf for water = 1.86 K kg mol–1, and molar mass of ethylene glycol = 62 g mol–1)
(1) 204.30 g (2) 400.00 g (3) 304.60 g (4) 804.32 g
Sol.: ∆Tf = Kf × m
w
6 = 1.86 ×
62 × 4
6 × 62 × 4
w= = 800 g.
1.86
Correct choice: (4)

*73. Which one of the following orders presents the correct sequence of the increasing basic nature of the given oxides?
(1) MgO < K2O < Al2O3 < Na2O (2) Na2O < K2O < MgO < Al2O3
(3) K2O < Na2O < Al2O3 < MgO (4) Al2O3 < MgO < Na2O < K2O
Sol.: The basic character of metal oxide increases with increase in electropositivity of metal.
Correct choice: (4)

74. The rate of a chemical reaction doubles for every 10°C rise of temperature. If the temperature is raised by 50°C, the rate of
the reaction increases by about:
(1) 24 times (2) 32 times (3) 64 times (4) 10 times
Sol.: Increase in reaction rate = 25 = 32 times.
Correct choice: (2)

Brilliant Tutorials Pvt. Ltd. Head Office: 12, Masilamani Street, T. Nagar, Chennai-600 017
Delhi Office: 50-C, Kalu Sarai (Behind Azad Apartments), New Delhi-110 016 Ph.: 2653 7392/93/94/95 Fax: 2653 7396
®

AIEEE 2011 SOLUTIONS 19

75. The magnetic moment (spin only) of [NiCl4]2– is:


(1) 5.46 BM (2) 2.82 BM (3) 1.41 BM (4) 1.82 BM
2+ 8 0
Sol.: Ni 3d 4s
The hybridisation of central metal ion is sp3 in NiCl −4 2 .
Therefore number of unpaired electrons = 2.
µ= n ( n + 2) = 2 × 4 = 2 2 = 2 × 1.41 = 2.82 BM.
Correct choice: (2)
*76. The hybridisation of orbitals of N atom in NO 3− , NO +2 and NH +4 are respectively:
(1) sp2, sp, sp3 (2) sp, sp3, sp2 (3) sp2, sp3, sp (4) sp, sp2, sp3
H
⊕ O ⊕ ⊕

Sol.: O–N , O=N=O , N
O– H H
H
Hybridization of N atom in NO3− , NO +2 and NH +4 are respectively sp2, sp, sp3.

®
Correct choice: (1)
77. In context of the lanthanoids, which of the following statements is not correct?
(1) All the members exhibit +3 oxidation state.
(2) Because of similar properties the separation of lanthanoids is not easy.
(3) Availability of 4f electrons results in the formation of compounds in +4 state for all the members of the series.
(4) There is a gradual decrease in the radii of the members with increasing atomic number in the series.
Sol.: The common oxidation state of lanthanoids is +3. Some of the lanthanoids also show +2 and +4 oxidation state but they
easily revert to the more stable +3 state.
Correct choice: (3)
*78. A 5.2 molal aqueous solution of methyl alcohol, CH3OH, is supplied. What is the mole fraction of methyl alcohol in the
solution?
(1) 0.190 (2) 0.086 (3) 0.050 (4) 0.100
Sol.: n CH 3OH = 5.2 moles

1000
n H 2O = = 55.55 moles
18
5.2
X CH 3OH = = 0.086
5.2 + 55.55
Correct choice: (2)
79. Which of the following statement is wrong?
(1) Nitrogen cannot form dπ-pπ bond.
(2) Single N–N bond is weaker than the single P–P bond.
(3) N2O4 has two resonance structures.
(4) The stability of hydrides increases from NH3 to BiH3 in group 15 of the periodic table.
Sol.: The stability of hydrides decreases from NH3 to BiH3 due to increase in size of the central atom.
Correct choice: (4)
80. The outer electron configuration of Gd (Atomic number: 64) is:
(1) 4f8 5d0 6s2 (2) 4f4 5d4 6s2 (3) 4f7 5d1 6s2 (4) 4f3 5d5 6s2
Sol.: Outer electronic configuration of Gd is 4f7 5d1 6s2
Correct choice: (3)
81. Which of the following statements regarding sulphur is incorrect?
(1) The vapour at 200°C consists mostly of S8 rings.
(2) At 600°C the gas mainly consists of S2 molecules.
(3) The oxidation state of sulphur is never less than +4 in its compounds.
(4) S2 molecule is paramagnetic.
Sol.: The oxidation state of sulphur in its various compounds varies from –2 to +6.
Correct choice: (3)

Brilliant Tutorials Pvt. Ltd. Head Office: 12, Masilamani Street, T. Nagar, Chennai-600 017
Delhi Office: 50-C, Kalu Sarai (Behind Azad Apartments), New Delhi-110 016 Ph.: 2653 7392/93/94/95 Fax: 2653 7396
®

AIEEE 2011 SOLUTIONS 20

*82. The structure of IF7 is:


(1) trigonal bipyramid (2) octahedral (3) pentagonal bipyramid (4) square pyramid
3 3
Sol.: The hybridisation of central atom is sp d and its structure is pentagonal bipyramid.
Correct choice: (3)
*83. Ozonolysis of an organic compound gives formaldehyde as one of the products. This confirms the presence of:
(1) a vinyl group. (2) an isopropyl group.
(3) an acetylenic triple bond. (4) two ethylenic double bonds.
Sol.: Vinyl group on ozonolysis gives formaldehyde as one of the products.
Correct choice: (1)
*84. A gas absorbs a photon of 355 nm and emits at two wavelengths. If one of the emissions is at 680 nm, the other is at:
(1) 325 nm (2) 743 nm (3) 518 nm (4) 1035 nm
1 1 1 1 1 1
Sol.: = + ; = + ; λ2 = 743 nm.
λ λ1 λ2 355 680 λ2
Correct choice: (2)
85. Silver mirror test is given by which one of the following compounds?

®
(1) Acetone (2) Formaldehyde (3) Benzophenone (4) Acetaldehyde
Sol.: Correct choice: (4)

86. Which of the following reagents may be used to distinguish between phenol and benzoic acid?
(1) Tollen’s reagent (2) Molisch reagent (3) Neutral FeCl3 (4) Aqueous NaOH
Sol.: Phenol gives a violet colour with neutral ferric chloride solution.
Correct choice: (3)
87. Phenol is heated with a solution of mixture of KBr and KBrO3. The major product obtained in the above reaction is:
(1) 3-Bromophenol (2) 4-Bromophenol (3) 2,4,6-Tribromophenol (4) 2-Bromophenol
Sol.: 5Br– + BrO 3− + 3H2O 3Br2 + 6OH–
OH OH
Br Br
Br2/HO–

Br
Correct choice: (3)
88. In a face centred cubic lattice, atom A occupies the corner positions and atom B occupies the face centre positions. If one
atom of B is missing from one of the face centred points, the formula of the compound is
(1) AB2 (2) A2B3 (3) A2B5 (4) A2B
1
Sol.: Contribution by A atoms at corners = 8 × = 1.
8
1 5
Contribution by B atoms at face centre = 5 × = .
2 2
So, formula is AB5/2 or A2B5.
Correct choice: (3)
*89. The entropy change involved in the isothermal reversible expansion of 2 moles of an ideal gas from a volume of 10 dm3 to a
volume of 100 dm3 at 27°C is:
(1) 35.8 J mol–1 K–1 (2) 32.3 J mol–1 K–1 (3) 42.3 J mol–1 K–1 (4) 38.3 J mol–1 K–1
Sol.: ∆S for isothermal reversible expansion is given by
V2 100
∆S = nR ln = 2 × 8.314 ln = 2 × 8.314 × 2.303 × 1 = 38.3 J mol–1 K–1.
V1 10
Correct choice: (4)
*90. Identify the compound that exhibits tautomerism.
(1) Lactic acid (2) 2-Pentanone (3) Phenol (4) 2-Butene
O OH
Sol.: CH3–C–CH2–CH2–CH3 CH3–C=CH–CH2–CH3
Correct choice: (2)

Brilliant Tutorials Pvt. Ltd. Head Office: 12, Masilamani Street, T. Nagar, Chennai-600 017
Delhi Office: 50-C, Kalu Sarai (Behind Azad Apartments), New Delhi-110 016 Ph.: 2653 7392/93/94/95 Fax: 2653 7396
®

AIEEE 2011 SOLUTIONS 21

TOPICWISE

Brilliant Tutorials Pvt. Ltd. Head Office: 12, Masilamani Street, T. Nagar, Chennai-600 017
Delhi Office: 50-C, Kalu Sarai (Behind Azad Apartments), New Delhi-110 016 Ph.: 2653 7392/93/94/95 Fax: 2653 7396
®

AIEEE 2011 SOLUTIONS 22

LEVEL OF DIFFICULTY

Brilliant Tutorials Pvt. Ltd. Head Office: 12, Masilamani Street, T. Nagar, Chennai-600 017
Delhi Office: 50-C, Kalu Sarai (Behind Azad Apartments), New Delhi-110 016 Ph.: 2653 7392/93/94/95 Fax: 2653 7396
®

AIEEE 2011 SOLUTIONS 23

CLASSWISE

Brilliant Tutorials Pvt. Ltd. Head Office: 12, Masilamani Street, T. Nagar, Chennai-600 017
Delhi Office: 50-C, Kalu Sarai (Behind Azad Apartments), New Delhi-110 016 Ph.: 2653 7392/93/94/95 Fax: 2653 7396
®

AIEEE 2011 SOLUTIONS 24

Read the following instructions carefully


1. The candidates should fill in the required particulars on the Test Booklet and Answer Sheet (side – 1)
with Blue/Black Ball Point Pen.

2. For writing / marking particulars on side – 2 of the Answer Sheet, use Blue/Black Ball Point Pen
only.

3. The candidate should not write their Roll Numbers anywhere else (except in the specific space) on the
Test Booklet/Answer Sheet.

4. Out of the four options given for each question only one option is the correct answer.

5. For each incorrect response, one-fourth (1/4) of the total marks allotted to the question would be
deducted from the total score. No deduction from the total score, however, will be made if no response
is indicated for an item in the Answer Sheet.

®
6. Handle the Test Booklet and Answer Sheet with care, as under no circumstance (except for
discrepancy in Test Booklet Code and Answer Sheet Code) will another set be provided.

7. The candidates are not allowed to do any rough work or writing work on the Answer Sheet. All
calculations/ writing work are to be done in the space provided for this purpose in the Test Booklet
itself, marked ‘Space for Rough Work’. This space is given at the bottom of each page and in 3 pages
(Pages 21-23) at the end of the booklet.

8. On completion of the test, the candidates must hand over the Answer Sheet to the Invigilator on duty
in the Room/Hall. However, the candidates are allowed to take away this Test Booklet with them.

9. Each candidate must show on demand his/her Admit Card to the Invigilator.

10. No candidate, without special permission of the Superintendent or Invigilator, should leave his/ her
seat.

11. The candidates should not leave the Examination Hall without handing over their Answer Sheet to the
Invigilator on duty and sign the Attendance Sheet again. Cases where a candidate has not signed the
Attendance Sheet a second time will be deemed not to have handed over the Answer Sheet and dealt
with as an unfair means case. The candidates are also required to put their left hand THUMB
impression in the space provided in the Attendance Sheet.

12. Use of electronic/ Manual Calculator and any electronic Item like mobile phone, pager etc. is
prohibited.

13. The candidates are governed by all Rules and Regulations of the Board with regard to their conduct in
the Examination Hall. All cases of unfair means will be dealt with as per Rules and Regulations of the
Board.

14. No part of the Test Booklet and Answer Sheet shall be detached under any circumstances.
Candidates are not allowed to carry any textual material, printed or written, bits of papers, pager,
mobile phone, electronic device or any other material except the Admit Card inside the examination
hall/ room.

Brilliant Tutorials Pvt. Ltd. Head Office: 12, Masilamani Street, T. Nagar, Chennai-600 017
Delhi Office: 50-C, Kalu Sarai (Behind Azad Apartments), New Delhi-110 016 Ph.: 2653 7392/93/94/95 Fax: 2653 7396
No. I 110 5 J07; l
Tlis bootlet conbitu 24 Pnnb'1Pa8€s

Do not oPen this Test BookLt stil vot de


Read c.r€fully the Instsuctios o! the 3dk covq ol this
Inpotant Instfl.tio'c
Tst Bdokl€t
A Pd
tn.,r'. p"*t""- m .ni, prSP ol $eTF| 8oo-er ath Btlel3k \ Bolt Poi"t
'r--*rl
&. af pdcil i. ,bituy Ptohibited
The A,wd Sh.et js lept insid€ thi6 T6t Bootlei Wren you m diected io op€n ihe Test
Bml]d, b*. out ihe A*vd shet md fill in ihe Partiolm caetullv
3. TE hous ddahon
t€sl is of 3
4. The Test Sooklei cotuisls of qi qlesriotu. The ldihum na'k de 360
lhere d-" Itr.. prd- rhe quen'or D"P'r A B, 'ans' rnA d Phvsks Ch€miiiry dnd
Mih.m:ti.! ha\,m J0 'n qu.'do! n e-'ptd '
of 'quJ n'i8h a3€ tdr qu6m s
allofted a (fo!4 n;ks ror ;a.h cor<t €sPome
rtttb'N,d?dw4s6 trrdM"tit' tu han Na rto,w t tyit ' ola'r'qw<tid
cjrd dak
\. tatu hwht w4\ xi! b? Mu ka br' nn 4 tn@d"t d Ptne rta4 qn"han No d"d u nor mn
h2 bA ,art uitt & nodr n no ,cwnr | ,nintar fu n tq n th! thw' 'tud
Ihers -sor vonc.md apoB. rotcdhquabor lJineupm
ouerti.n *'l b. ft, ed x -'o.s rdpon+ dd ml|. for Mong raPo* wiu b' dedlrkd
a..ordingly d per iGtruction 6 above.
A. tj6e Bl@/ Blnck Ball Point Pd d&/ for Miting panicul6/Mkng 6p'Ns on Sid?-l dd
S'dF2 oi the Aswer Shet Us. oJ pdcll is sni.d! Ptuhibited
q No.Jnddare h ilosed o cd_' anv rP\rua. hdhn4 Prinled or wr'lle- bns or Pdpere'
p"eer. mb'1" phme "r el^n.nl dry.e Ptr .*PI hP adnr ftidr l}e Adindbon
'ad
be dom on ihe sPace Provided for Ois Purpose in the Tst B@ldet orny
This
Rolsi vor! i. bo
t.tt- boouet
.r..? ; Ar"r o* *ch pas; md in 3 Pass (?ag€6 21 - 23) at the end or the

* ."-.r.o* or ,r." n. ,lr""l .urd ddh mut tdd ove- rc Atrwd 5hca ro .hc lnqgxalor
"r on
i.*;lru n.-,ru no--e,thP ndddr{ ' atttu d to tar' au'! thtsr6t BoorIPl
t2. he CODI ro- rh. Bool.el F A. Md' " qr'€ lhrr rh" CODL pr'lLcd o' side-2 of rhe
aswersrer F r\Fsre dr rl.r m \b bookler lr.#oldis@prry hP rdidtP hord
.nr "dlhl) tlc ruft' l,o $" InvjSi;b r,o' repra "T"nr ^l boih the Tar BoorlPr rd
'cpor
13. Do noi fold or make any skiy ndks on ihe Affier Shet
**'.*" * **t -t1*,, PALT\' AI
"*t
3. I t)
A-e,al 6L\4^
EEni@don cenft Number:
"l 3 c (
NmeorEiminationcd@(incapihrbss):A odr ^1( \tF ' sc-$ool J AYAfr LIOAt?U(

.-*0"","'s-*. "\, ^J
3-ul Truth tall€ lor sysrem ol for NAND gates
Two electric bulbs narked 2sw-220v
r' ,// dd 1mw'220v ue couected in series r.
a 440V supply. Whicn oI the burbs win

g/',* B
0 0

0)
i! i#, 0
1
.:\
(41

t'.,1 0
a
1

0 1
(21

v A boy 6
helsht oI 10
throw
n
a stone up ro a
J\e Iwjmum holronral
mximm 1

di$a@ rlat the boy.d throw the sde


B
0 0

(1)
I
(3)
tro.tr r
1

\2) f. ? -1\
a
B
g{zo^ 0 0
0 I
1
(4) 20E n I
gPACE FOR ROUGIT WORI(
rL'f P.
rr\fi.
(, vt
r
'
J
'. f,i-u is \5
'=i'' ,,::,.!i"".,+..
i, -r5.5
'r.'

1. I itr-'c '";;|
ln YounS's double slii exPdiment, one of
4./ lhis ttr3tiar has statefltnt 1 nnd
the slit is wider than other, so that lhe
l,/ statenat z of the Jart ihdies iad rfu
Stttentflt!, cltoost thz one thtt
the
des.nb?s the hN sldtmcnts
besl
iffi::*:'i*:i:ilil;:T:
naximum inteNity, $e resdtafrinletultv
I whd they inl4fere at Phase di(eretue
O is given by: f
(r)
IL
+,1-'.*,1' a,. 9i
Statem.nt 1: Davisson _ Cermer
dpe.imenl €stablished tne wave nature of (2) .+ (1+4cd':) ,
a
g, l,'*-''!r 5r. \1
Stat€meni 2 : If elechons have wave t(5*!r *.r)
natlre, they .an interfere an'1 show rn, ! tn*t-'or
LIG II ;J
If a simPle P€ndulum has significant
(1) Siatement t h bue, stalement 2 is amplitude (uP to a lacior oi 1/€ ol onginar)
.nlv in the pdiod hehv&n t = 0s io t = {t
thd r roy L caued u€ avdaee life ol the
pen&ium when the sphe.i.al bob of the
lendulun suilers a retaidation (due to
i2) Statemenr 1 is trrc, Statenenr 2 is visous drag) ProPorrional to its veto'1rv'
tre, Statement 2 is the co'rect with'b as OE costant oI propo'tionaLity'
explamtion lor Statement r $e avera8e Me time of ihe Penddrn is
(assuming danPins is snarl) in saonds:

(1)

(2)
b

(3)
t
1 is I.lF, Stalehent 2 is

SPACE FOR ROUCH WORK

1.. q1tt. .r
Jk' This questian hls Stnt.neat 1 dd
strtmdt 2. of ttz hlt chan6 qiM afd dD.ge Q dd radius & th€ eltrEic fi€ld E
thz Sttt.flents, choaee the one thor b.st is plotted as a fmction of dbtdce ttum
de{ib$ lhe hno slalemdts, the .enrre. The graph which would
.oftBpdd io rhe above wil be :
r two springs 51 dd 52 oI fo(e coturdts
k1 md k! Fspd.tivel, de sheched bY ihe
eme for€. it is folnd dEt mde work n
dore pn sp.ng q than on spring sr'
,l
stat€m€nt I : If stretcled by the same
anout, work don€ dn s1, will b€ nore (1)

gy'$aierud 1 is true sbt€ment 2 rr


kue, statem€nt 2 is the .orrecl
explmtion oI Sbrement 1.
(3) Statenent 1 c rrue, Statement 2 is
kue, state@t 2 is nol the corr4t
€rplamtion 6f stat€rent 1.
(4) Statenent 1 is false statenent 2 is

.4. An obj{r 2.4 m in tront of a letu form a


(3)
shrp image 6n a filrn 12 cn behid th€
l€s. A glass plat€ 1m thick of €fnctive
indd 1.50 is inle.po*d kws les and
iilm witi itl llane faces Paiallel lo firn
At whar distafte (from lere) shodd obj&t
b€ snifkd to be in sharP l@s d 61m ?

(r)
(2) (4)
(3)

(41

SPACE FOR ROUCHI'\IORK

.U*j
"' *r ) t' r'*,
10. A coil s s$pend€d in a unirdn nagnen' J}, a diromc mdNle ir mde ot h'o tuss
ri€14 with the platu of the con PaFuel lo
- n, rd dr whrn e FParated h"v a disi:nce
tlFra$e!g!!rsof iote Wnenacrent r. r ve .alculate ite rotationar eturgv bv
is passed ihrough the coil it starb aPPtying Bohls nne ol anFnd mnmtum

.scillatin& it is vdy dijficdt to sloP But qumtizri@ ry!sry!!!: sv@ bY:


iI an alminium plate is Plred ner to the
coil, il stoF This is dte to :

(1) induction ol elechi.al charS€ on the 0)

(2)
(2) snielding oI tu8fttic riffi ot tote
as aluminium is a Paramagneti'
{3! \.
l) el{tromagner( rndu.lLon 'n the
aluminium Plate giving lis€ to
el{tionagnetic danPrS. tl'(!^)"
*
,,,.,9!_1i;
(4) development of an cudenl wrd the

3 19
25

,:
r ly'A spdtunetd gives the lonowing eadjng
" when used to measure ihe dgl€ ol
Y
a Priqn' !ro
Main hle reading :58.5 deglee - Dl
5!
vemier scale reading, !! dn'isions
50 100 150 200 250 3m
Given thal 1 division on mam scare
codeslonds to 0.5 degree Total drlisiotu
on the wenier scale is 30 and mtch with
The liglre shom an €xPe.imenlal Plot tor
29 divhids ol lhe tuin scale The anSle
discharSing o! a caPacitor in an R'C
of the pnsm hom the above data : .init. The tida .onstant r of Oris cinil

ta/'58.6 d.e.,ee (2) 50 sd and 1m se.


(g,/rm ,* u"a rso *"
(4) 150 sec and 200 sd
SPACE FOR ROUGIIWOR(.
2* ):' '' . /1 irjPa
\7t )l
[.i l(- a ,r

A particl€ of Ims n is at F* at the origin 16l A radar has a power of Lkldrnd i3


at tine r=0. It is subje.t€d to a force
J .p-,rinB ut u r,"q'"^.y orE&- n i"
F (t)=Foe
hin ther direction. Its sp€ed located on a nounrain toP of heiSht
js depicted by whi.l oI rhe fonowin8 s@jrLTlE tuimm distane nPto which
"(t) it cd detct obj{t ldal€d on l}F tuface
ol the eain Radis or eairh - 6,4 x 106 m)
al .'I
(1)

(2)
-!g
\2)
(3)

(r)
! V*^
17,
z A$ure dEttuutrcn bEals into a Proion
a

5 and an eleciron. The ene.gy released


(4) dring this plocess i3:
uol
(Mas d iaul'm- UZ25,10 2'ts

Two uB of q and m, de noving


n6es
" ,tz-
/
in circles of radii ! dd rr, r€sp{tively
Tneii speeds are sn.h rlar they make
.om?Lte cirles ir the sme timef Th€
ntio of their.mtripetal acceldation is :
(1) 7.10 Mev

(1)

12)
(3) 5.4 MeV
(3)

(a) 0.73 M.v


SPACEIOR ROIJGH WORK
'a,r a,1r ! c nt
!"
(0
v This q@nion has Stdt.nent 1 dfld 19rl A liquid in a b€alet hns temP€ntur€
St,t ndt 2. Of tlv hw ehai6 8i@ aJtt " at tim€ t and l]o is temP€rature of
th. Sttt.ne ts, chaose the oke that besl mdings, then a(oidinS ro Newton s
d.sdib6 lE h@ stttm ts law of .ooling the corecl 4aPh berwM
lo&(o-00)andts:
An iEulaiing sgBls-t hdjof Fdis R has
a mifo.nny positive chdg€ d4ity p. a .9-n'l
g. a")
a result of dtis unifm dEge dishibuldn
rher is a 6nite varue of ele.hic Poreniar
ar rhe .mtre oI the sPE !e, at tne surface
of the sPhere and also at a Poinr out side I
tlFspnere. flE eldhic Pot€ntial at iiliDtv

Stat€ment1: Wh€^ a ddge'C is taken


lrom the cenhe ro the sffIace of the sprEre,
chdg6 bY
3
iis potenial energY
;
(2)
st.t€n€n:22lne ekki. rield at a dntane
1

r (Frl ftomlhe.enbeot he spnere E l.o

3
(1) statement 1 is ttue Statement 2 is
(3)
"qd

statement 1 is lalse statenenr 2 is


I
9/

(r) Statemnt 1 is hue, Staterent 2 is


hue, Statement 2 is rhe corr€cl 3
€xplmnd ol Srateneni I

(4) Statement 1 is hle Statem€nl Z is I


h{e; Stat€ment 2 is not th€ coE€.i
expl,fttion of Statemt 1.
SFACE FOR ROUGH I{IORX

II r34 3
z@
ff i,(tB] l
.3L,. ?r 5^ & ;.' i
R.stlNe oI a gt!€n wn€ is obtain€d by 22
L r^''
A cylindri.al tube, open ar both mds, has
measuinS the .ureni liowing in jt and a tun.lamental fr€quemy, f, in an. The
tlE voltag€ .litferene applied aoo$ it. U hrbeh .lipFd v€rricany in wate! so lnai
thePdenlage qrc4 in the meas@ent hau oI it ir in rarer, The tundanental
of ure culmt and dE voLtage differenc€ Irequarl of the air<olllm is now :
are!3-each, rhen e{or in rhe vahe or
'J
resistatr€ of the wire is :
t!%
?, !I (2)

(2) {3) 2f

(3) t%
9/'
Cuo t /'aU-shape.l
thin liquid liln
wire and
fo.n€d betqeen a
light slider supports
a
a w€igrt of 1.5.xlO-2N (seeligue) The
I l6$n of the slider is 30 cn and its w€ight
negligible. nE sufre tesion or rne iiquftl
2l_zzhe lMs of a spacBhip is 1000 kg. Iiisto
I\---l
I b€ landed lron the €dthk turface our
rnto i.ee space. The walue of 'g' and ,g 1r-{ \,- l'!,at
'R' (radius or ea.rh) ale 10 m/s, and
_'
64!0 tm r€spsrivdy. The r€quired €nqgj,
fo dE worl wil b€ ,t ' ';;
i .ti|\\ r
I l:>4.
--^
u r! (3" \ d).: r:'
il) 6.4!1d toubs

e);.ax10,lotrr6
(1) o.1Nn r

(2) 0.05 Nn r

\)rt'n'"t."k' 9)- 0.025 Nn-1

(4) 0.0125 Nn ,

SPACE POR ROUGH WORK

J. !{
P
H€lim s6 s@s duougn a qde*r!qeq{_-
,sl A wooden wheel of ndi$ R is md' or
rwo sdidKuld Pats (sea fiSure) The
(cotuisting of lwo ho.hori' and two
r{o parts de held toSeth€r by a ring md€ isobari. lines) as sho{n in ligure
mbr striP or .rcs sdion t artjrl E6iciency ol rhis .Ycle is nedLy :
ot a
dd leng!\!;L is srighllY les thd 2?R (A!ffie the 8as io be dose ro ideal gas)
To ffi the ing it is heated so
on th€ wheel, d{'r'-
that ib temPeratue ;ss by-Al and it iEt v ye'
s€Ps w€r the wneel As jl c@ls down to
sDnoundins temPerature/ $ Presses me
semi.ircDlar Parts to8ethe. ff rne 0,a'\"n
c@fficiat of tinear e4arson ol tle retal
is q dd its Yodgt nodulus is Y, th€ lorce (1) 9.1%
$at ore parl of the wleel aPPlies on ttu (2) c"a'):-/
.
10.5%
',\r"t
tr (3)
(4)
12.s%
15.4% '| 'q. i
eo'
ilydrog€n alon is er.fted fron glou^d
i vj46t state io anoth€1 state with P'inciPal
qudtum rumbd equal to 4 Then the
nunber of 3p{titl lines in the emission
- i:3 (l
/5JAr gt3
.5
12)

w6
(4) 2

27"u4otor! dd alPha Pdticie 6I the


.g/rr".o, Deuteron
lineb. effigy ft mvmg m or(:rlE
- same
trajatories in a consta mgndtic field'
The radn ot protoD deut€ron and alPha
pa*icle are res?e.iively .p, rd and rd
Whi.h one ol tne lollowing relations 's

&{5'y, rir 1:

SPACE FONROUGIIWORK

lf "1r.
1/
s.
24. A .hdge Q js nndrJy dGhibuted over 29. An el€cbodgneh. have h vacdm has
the su.face of non conducung dis. of
radius R. Ihe disc lotates about an axis
rh€ ere.dic and mgneti. fielde E and 3,
which are always p€rpendicular to each
perpendi.ula. to its llane and passjng

.t ttuou8h its cenhe witl an angllar veldity


o. A3 a resulr of fiis mralion a mSneric
orhe! The dire.t'on of polarrzation is
givo by i and thal oi wave P|opasario
field of induction B is cdee
obtained at the
oftiedis. t we keep both the mount oI
clErge pla.ed on lhe dis. and its angular
rclo.ity to b€ corermt dd vdy L\e !adi$ 0) ilE dd iIEXB
of ih. disc rhen the variation of the
tuaretic indu.tion ai the cenhe ol rhe dis
win be repre*nted by the lig!re |
(r) i ld aa i ix6
(3) i lli i16xE
(1)

. ,;:
(4) ld -d tld.i

30. A Camot engitu, whose eiii.iency is 40*,


(2)
t l,'$rn takes in heat from a source mintained at

dl a lehp.rahne of 500 K
It is desir€d to
have an engine o! eltcien.y 60%. hen,
th€ intake temperature for th€ same
exnad Gink) tenp€htu€ nusr b€ :
I
(3)
(1) 1200 K

a4i zso r
(3) 600 K
(4)
I
(4) erficiency oI cainot ensine.annot
b€ mde larger dEn 50%

SPACE FOR ROIICHWORK


_1
T.
1
PART B _ CSEMISTRY pli" -P**ion amonB the
'*-*"
(1)

(r// LilMr3 wn#bb - nRr hq


(2) PdlBaso4
(3) U Allr4 '
etzAx= RT
(4) rrlH,
Which ol rh€ toUowjtg on the'mats
dMPosition yields a basi' as wer ae m

(1) (Clo3 -!.


(2) cacos Whici bianched cnain isomer or tne
(3) NHINOI hydrocalbon w'rh mole'ular ras' Z?s
eives only one iMe!
of nono substihried
(4) NaNo3
33. Which N ol the folowing stateNnls ir

(1) AU mjno a.ids de oPtiol]Y ftive


(2) A]1 anino a.id;eacePt glYqire a€
olticallY activ€.
(3) AI amino a.ids q.ePt glutanic aod
are oPti.allY a.tive s/ A.coldin' to lreundlich adsorPrton
AU dino acids excePt lYtne arc isothern, whicn oi lh€ lorlowing is

34,,/Th€ detuity ol a solution PrePare'l bv c);"P1


7 dis$lvins 120 e ol fta (nol m$=60u)
in lOoO I 6f water is 115 g/m]- The
hoLantY oi rhn sobtion is l
r' !:-
0) 178 M
(2) 1.02 M

g{ zos u .ld Al e.orEci


the above
rag6 ot Piesw
tor diffdeni

(4) 0.50 M

STACI FOR ROUGH }IORI(


fdl tl $'r'
,r. i ," c'a
5t
# :tl .r
l'
a
1l
09./ln which of the roxowing paiF the h,o 9rl l. th€ Bi'en h".,romah"n whEh,
sPe6 *jgusostructual ?

;z G) lclr* aa soeu 5f
'- .M11""a4-r." ':i /---:z.o{ = oJcOCHr
ta (3) AF!r- dd sFd ,: --
I ; (at COr:- and NOr .-:-" "ov
:%0
'".9,, }low na y chiral conPoun& are .YcH=ctffi,c}I3
pcsibl€ on monodnorimiion of2 - ftthyl
z^-HA/:rIcl !
vt
12) 4 | - t- L- L
(r)
(2) Na, Llq. NH3

(3) NalHa -
\3) 6
al
o
(4) rry4,r-r,"r,ru.,8s
8
,t/
Jy'lhe im€sins o.dd of ihe io.i. ndii
is:
Lithilm lorms bodY cenrred cut
the Erwd boel*tronic aPe.i6
sfuctue. Tte l4grn ol tne side of iis u
0) s':-, cl , caz+,K+ cel is 35] gr Atonic.adi$ of ihe litlw
,py'cS*, <*, cr, s'
E) K+, s'1 , cr'1+, Cl
(4) cl ,ca':+, K+, s'z-
(2) 240 pm
rcne*sibrftl
P, !^.S'
\i),4
*X ,.f-s\"
(1) 1
r) (v-*r). n(t
\ (tl)
49/ l+pb/Rr 45 Kr lor wab b K lg mol r. ffYo
1.86
(3) 1-lb/Rr J
antonobile ladiaior holdsaQf Eor wat
(4) 1+Rr/r'! ,.J e how many grams ol elnyl€ne glY(
(CrH6o, must you add to get dE frezi
tt ./t\4ichamng thefolowingwilbemed
as diblomidobis(erhylene diamine) point of th€ solution ldwd€d to - 2 3"(
.hrolid (m) brcNd€ ?
,v"re
(2J 39 E
{,[: l,tl ;*\
(2) Icl (en) B!r-
(3) 27 B

(4) Icr (€nll Br3 v \4) 729

SPACTI]OR ROUGHWORX

" ''i ct\ a,'t *t Jq


n*
93 t-862 ,,,8o r-
J!

61'
cmposds can be
Which of ine lollowing
fte moldk him8 sdlbsr bdd
argl€
der€.1€d bY
r€'t ?
2r 4d!s43

G) -Ascl3
!g)'/ sbcl3
(3) PC\
(4) Ncl3 ror
sz The ,landard redu(r'on Potenriars
is DDT aNng the lonowng :
- ,^r*lZn, und Ie" / re are
",t*/n
-:a.7b, 0.21 dd ',o 44 v reePe'hv€ry
V.sv (2)
-
Th"de*tion x+!l+ .x'?*+Y
wilL be

sPontaeou6 wlen :
(3)
(1) x-Ni, Y=zn Lt
(2) x=Fe Y=zr1
,/H fj' t?: T:f":iff:,::$il: 192/x=z' v=\i
(a th;
coffidi, of acid F I

(1) 1x1o 3
ut/ rxro-' dl'r1tr,
{3) xlo
T 7

(4) 3x10-'
vl2-l/o
N'FoPheiol'ghows
b€ mde rnkdole.ularE _ bondsg
49- Ver) Pue hvdrosen 1{q%lGf $
b) wtuch oi thE fo o*m8 Prdes*s
r i3l Melti^8 Poinr ol o - NiEoPhenor
.
hydrocarbons ot l.wer than those ol n - ano
(1) Mixing natural
hish molecdar weight P- ieomds
rolabe 6
(?) ltedrorYsL of vater rar o - NitroPiercl s dore
wth steam than ihose ol
m - ano
(3I Reaction or salt like hvdrides

(4) R€action ol nethane


with $eam lrom all
. lodo(orm can be Pre?ared
as
5or. AsPiiin is known :

K. 6y rt "nyt "uri.vlut" (2) 3 - Methyl _ 2 - butdog


(2) Acetyl elnYlard
(3) M€6Yl satiq!' a'id rs6€
(4) EL\YI flebYl
(9,/ Acetyl sali.Ylic a.ld
FacT FoR RoUGH r4IoRK

P.
{ L. r,:
.. i-
cm re
bond angre t1. Whi.h ol the following compouds
Ihe molaule lDvjrg smlest r
2: ' ao"o"a ty !4ou"ut t""

(1) -AsCl3
G/ sncr3
(3) PiinDiY alcohols
(3) PcLr
(4) Nitrc conPouas
(4) NCll
szl Ih. srandard redud'on potenr'als for
is DDT aNrg the lolowing : v znr*/zn, Ni2+/Ni, und re;r/Ee re
V ,ty
(2)
=0%. 023 and
rh-eaction \+11+-\:+ +r R'll
044 v re5Pe'bverY
be

(3) sPont@ls wtun :

(4) (1) x-Ni Y-zn L'.


(2) X=Fe, Y=zt!
4& ofa l mdr soluhonof tJre a''d
The PH
HO El
O

Tbe lalue oi the


{g/x=z' r=ni
'onization {4) x=Ni,Y=F€
cotutant, Ka ol this acid rs :

in wa@
5t1 ortho - Nitro$enol b 166 sluble
"
(?y' 1x\o-5 't,', ihmP-ddm_NiboPhenolsb€cause:
(1) o - NilroPhmor shows
'"i'n,o' Intrrmorduld H bonding

be @de
. r/1- Nrhprrena st'o*s
lt
49 Very Pu'r hldrogen (999%)'d ' rnrenol€crdd bonding
is
by wluch ot the fonows8 PiocsFs (3) M€1ting Point ol o - NitroPhenol
(1) Mixing natural lvdro'arbons or lower than tlose of m - ano
high molecurar weight
(a Ek.trclYsis or wad (4) o _ NitroPhenol is more volaiile
n
Reaction ot sair r&e nvdrjd€s
wiih <i€am than ihos€ of m - ano
(3)r

(4) Rea.tion of nerhane with sted


Iodolorm .an b€ PrePaied fron
all

50l AsPnh is known as l


U( o) r'henyl slicYlate
(2) AcetYl *lirylate (2) 3-M€tnyl-2_buran@
(3) Metlyl salicYli' <id
(a) rthyl medYl k'tone/'
1l9/ AdtYl salFYl( aod
specr ron noucn wonx
rhe species which can best 6erve as 59. lron exhibik +2ad +3 oridation srahes
P d initiator fd. ihe etionic po\Terization wnich of the rollowing sratenents abou

(1) rNo3
(2) AICL (1) Ferrous comlounds ale relarive!
(3) BuLi mole ioni. thd the corespondin!
(4) LL{H4
\S The equilib.ium constanr (K.) for rhe (2) Fsous conpduds de les volaril(
rdadion N,{8) + oz(s)+2Na(8) al ihan th€ co.responding lelrn
rempdatu.e T is4x10 a Thevaheoft!
for *r€ 4actror! No(s)+n Nl8) + r! ols)
at be sam temPeratuie is :
(3) Idous compouds ae norc easilt
(1) 2.5x10r
hydJolysed tld the corslondine

{?l 50.0
TJ ; o"to" t, **"
31fr* -o* uuo"
ayr 0.o2 "

The eledrons id€ntiried by quanrum


997

!!1!& i" : (a) n-4l=1


Jf 3.4ixfi 4 M/ni\
(2) 3.47x10 5 M/min
(3) 1.73 x 10 a M/mi.
(4) 1.73x10 s M/min G) n-3,l=2 q

(d)
5& Wnl.h nethod of purifi.arion js
rep!€hted by the following eqnation: cb be placed in orderoftrreasdssergy
Tj G)+2Ir(B]--!PI+ Tir4(s) ltuox ,
TiG)+2rr(g)
Jrf G). b)
(1) Cupellation (2) (b). (d)

(3) (3) (a) < (.)


van Arkel
{a) zone relining

lrr'
(',rr A'itJ ' ,
"'J
r o,r ,.3!) 3-qr v,i
l--rt
54- Ixt l' I MtsrcF Fih P *Q
PART C_ MATTIEMATICS
' r'4;r{ Pand Q be

""9 llo=9111
ti"^
st-/",x=tt z z + St Th€ nDdbd'l .r.@ni of (r'+ai) is eqd to
c,' d,ffoeni od*d PaiF Cl,-z) u'at 'd r (1) 1
l{m€d flch tha!Ls]' z ex and ^z
Y
(2) 0 e t*, d^je.
(1) 35 t 3:,
(3) _1
(4) -2 f , "-"'
.g/z' i" ;(
(3) 53 B p9z/tt * it
4a
4,r r" 5 '
(4) 5'z -J, t .,
[9!a: d,=, 6
A

r oi a edan
b'."'!*'l 1'
, 6/rhe poPulahon Pltr ai tiF€
" nouse spe''es satisfr"s the
drterenira' (1) 2 (.i".a t'-"*
(2) 1
equation 45o rr .,,.
#=os?(t) sy'z ft;i;p-o
p(O) = 850, then the tim€
at which rhe (4) -1
-$[ fr*t*.-*^' i"
'
6t It s(r)= Jo
cos4t di, lhen g(r+n)
1
t:2) , (I) s (') +

6du 13 (2) I (:) -


14) 2 (3) c (tlc

632- Ir / : R+R is a llnction delined bv

- 16aa-,la;!)-where tll denores


,6?./An equation ol a PLtr parauel to oP Pide
$e 8F;te* htesd nmdion' thdjis :
r-2;+2'-s=o an'l a urrt dstance
"t
(1) disontinuous onlY 49:1Q- ?"Ftr.*ig^i"' -j')7
u) , 2!+22+1'0
(2/-discontiNous onlv at no!-zero
\-/ i Ecral valu€s of r'
(r) conrnuds onlY at l-u
g,//r2v+22 3=0
(4) continuo6 for dery real I
Iracs ron noucs worx

t;{-)@r r
r.l j^'.r.a
ku
A spheri.al balloon is filled with sraknent 1 : An equanon of a c()m(
4500 r dbi. meteis oI helium gas. If a ransen o the paraboh l-16a6x ar
leak in the balloor .aues the gas ro escale
tne euipF 2l+l=a b r=2r+2'6.
at the rate ol72 T obic turers Er niftle
then the rate (in meters per hinrte) at
which the &dius ol the baloor d{reaps
,or*"., r , ,r ,1"t*"r=**4
gmi tes afrer the leahge began is : (m + 0) is a .onmon raneent
l1) 7 /9 the parabola !2=1615: and rl
elli,pse 2,2+!2=4, then m satisfi
t2) 2/e 4: ' r.!,r
\3) e/2
14) 9/7 J9- srabmenr 1 is rroe, statement 2
trn€, Statement 2 is a .orre
gt / Ii rh" ti"" zt +y= t purses tlrorgh the explmtion lor Stalqlqt 1.
point whi.n divides rhe line segnent (2) srarement 1 i3 bue, siarerent 2
joinins rhe pdinrs (1, 1) md (2, 4) in rhe
true, Statem€nt 2 b not a coEe
ntio3:2,tI@keqnals; elPlmrion lor Statemt 1.
(1) 5
(3) srabnent 1 is hue, siatemenr 2
tg/ 6

(3) 11ls
Sratenent 1 is fak, StaleMt 2
(4) 29/ 5

\j9a rf,t a and b berwo dit 72. Three numbers are chosen at rando
r witlout€pla.enent lrom { 1, 2, 3,..., 3
'*t^i=i+:6"'a
are pe+endi.dd to each
The probab y fiat their ninimqn ie
given tbat ihen mjmm is 6, is :
anslebetwenS'"aGis ,s
(1) a lY'
, .l
j'i 12) i 6+ 1

a (rl ? -s
(3) 5

(4) ! (4) n
3
73,/A line il draM droush $e Point.G.alo
/
-*r th€ @ordiMte ax6 at P and Q such
tlEt it a Eiangle OPQ, {hd€ o is
fod
ihe 6dgi!. r the ea oI the hiangle O?Q """ ^=li : ll '""". ""-
en the sloPe of tne Lne PQ i5 |

(1) -4 .",* **"" ""*


*, *, =
"-
(2) l; I
;-2
rd%
$) - '/a ^'[r]
'7 Asuning rh€ balG to b€ identical etePt
I: I :lli
Ior differefte in .olouit tne nun!€r or
ways in which one oi mor€ baus be
'an
(r)
I J T ,JL l"(i C

sel*ted from l!.lrbile Zsl9€n and ? !la'k


balls G
(lt 629
i2) b3O
\)t 8?e ,."i" I
"(. .
_ L"
(2)
il
L'1,
(4) 830 t 1
1
Stat.ment 1 The s!0 of the series
1+ (1+2+4) + (4 +6 +9) + (2112+ 19
1

+ . . . + (361 + 330 + 400) is 8000

sr.r€n*tr: i(Lr -rr


ror a9Y nahrar nmM n
rrr)=nr

l=+
ffi bomded b€twd
and 12=9Y, and
€|-
$e Pebdd
$e straiSht line
I
{1/ statemeni 1 i6 t!u€, stat€nent 2 is
true; StaGm€nt 2 is a cor"ct r=?1".
exFlafttion for $atenenl 1
(2) Slat€nent 1 is truq statemenl 2 is
true; Statement 2 i5 not a .orre't
.grtt*
€rPleiion lor Staternent I (2)
2a
"E
(3) he Shrement 2 is 3
statement 1 is
(3) ro,11 t
(4) staten€nt I is ialse, Sbrenent 2 is
(4) N
"!i
SPACElof,ROUGHWORX

+.
I. j'
i +r-- 3. ,3
(c'n' - o- r'l \kr' r <') i !r -!r rl
u.. md let ao.Zrm um ue roos mor an el wi
.z/. *t 'r, ',
""
,
"t'enatiom' f n.^ zero comon aifr'ren'e eqoals t
t be rheir di$m.tic ned and ql be th€n 150'te
50 tnres ft sos teE! th€n ihe
varian e. r,-;t-Y,. ,,.

Srntdert 1 : Vdian e of 1r, 2l2 '' ztn (1) 150 tiFe6 its 504 ttd
(2) 1s0

Statement 2 : Aiithmetic mean ot


a>/*-
(4) 150
L\, zt! ,, t^\s E.
8rl- The length of ihe drameter ot the 'ir
(1) Statement 1 is de, statement 2 is g which rouches ihe ,_a\is at ihe Po
true, Statement 2 is a coli€'t (1, 0) a d pa$6 tlmugl $e Point {Z
expLanation for Siatemdi 1

\7) 3/5
(2) stat€nent 1 is true, Staterent 2 is
hue, Stalement 2 is not a cof,ect
13, 5/i
exPLanation Io! Statenent 1
g{ tott
(3)/statenent 1 is true shtenent 2 is
1 8y rlt a, b. R be su.hthat^th€tuMuon fgi
" tv I t*)=Ln lrl+bx'+a'' r+u
extrse valu.s dd
at r= -1
(4) Sbtemdt 1 is falF, slatemnt 2 is '=2
Stat.ment 1 : /has local narnur

st tedentr: a-; dd b= 4
/r !-jq l-p9:!!t9- ]]1es:! rh5
- 1va r 4'- 1"e -
o'
';'" v
(ltz statenent 1 is rue, siatenent
true, staremeni 2 is a cor
exPlaEtion lo. Statdent 1'
(1) d odd Posihv€
'neger {2) statement 1 i. true' Statenent
lruei Statenenr 2 is rot a 'o
(2) an even Posiiive
'nreger expldtior ld StaErunt 1'

(3) a ratioGl number othq than Posiliv€


(3) Statffil 1 is true, statemenl

(4) statement 1 is fal6e, statemdn

s?ACE FOR ROUCBWORK

c ,,J -tts-r)-( ji ,E -,)


( l*/.(i rr) ,ft ",
G' '!)
3t lFt ABCD b€ a para elogim su.h that 35r-.dn ellipF F drawn bl taLbg a drdeier
r' of ihe cilcte (r-1)':+r':-1 as fts semi-
aB = ,1, 4p :] and ZBAD be an
hinor axis and a diam€t€r of th€ .nd€
acurc an8le. Ii r is rhe vecror ihar l+tv-2t:=a.5 lts 5e* @jo..n5. rf
.oiftid4 with the altitude diEted from nE dhe of the euipe is at the oriSin d.l
theverter B rdthe sideAD tien r isgiv€n fts de are ihe coordinale axes, thm the
by: .qDtim of the e[ilse is :

(r) ,= i.f+l i .., (ii t')


o)

-r( (2)

v@t
v k
(2)
lppl
+y
{'r)'
pv i= :i* ll P
(4)

o]
[o
36, Th. regation of the statement

"lf I b€.om€ a tea.hdr, tnen I wiD open a


l;;J
34,
'r
*" r'^* f =$=f (r) Eithe! I wil not bdme a teache. o!

i = -- =
i intds<t,0Enkisequl
I
r d" lgf Ne ther I w u beome a t€a.h.r nor

(1) ?
t-c (3) I wi| not b€.on€ a rea.her or I will
(2) z
2
(3)

(4)
0
rfl' (1) I will becore a teacner and I wi]l not

SPACE FOR ROUCHr{'ORX


"
:v /(r)=lHl+r-sl,r€R "y'*.***:: "

stat€F.nt 2 : tis h-l?4:l' l7)


'ontinuous
diftde^tiable in 99 dd I =l €)'
(a
2; (3) *ctly ro$ rear r@u
(1) StateNnt 1 is true' StateMt
truer Siatenenr 2 is a codecr (4) intnite Nnb€r ot realrcots
1'
eaPlanation for statdEnt
2 is
(2) Statement 1 is rrue statedent
trueiSlatement 2 is n6t a cofe'r
exlbMnon ror siatendt 1
9(L tn a APQR, iI3 tn P+a 'oq Q=b
' a sin Q+3 cos P=1 tnen
r}t I
1 is tru'' siat€meni 2 is 'ngte
{rzstatement
2 is
(4) SraFrent 1 is false' Sratercnt
(1) ;

83. Uz +7 4d :- 6 real ihen $€ Ponr

rePre6ented bv the comPlea numbet

ihe ongn
G) on a .irle wfth enEe at
(2) eithd on the real dis G or a dde
not Pa$ing Otrotgh the
ongr' (4)
6

(3) on ore imsinary ds


. {a) atner on the real as
or on a
're€
ongn
r€$ing [vough the
-AcE roR noucH woRx
Il:lr" <+r ) <<n lt
^ri{.1
l(*Qi.) ;| 'r-r - ^'3
f.'-f

)r-t_
FIITJEE Solutions to JEE(MAIN)-2013
XYZ Test Booklet Code
PAPER – 1 : CHEMISTRY, MATHEMATICS & PHYSICS

Read carefully the Instructions on the Back Cover of this Test Booklet. Q
Important Instructions:
1. Immediately fill in the particulars on this page of the Test Booklet with Blue/Black Ball Point Pen. Use of
pencil is strictly prohibited.
2. The test is of 3 hours duration.
3. The Test Booklet consists of 90 questions. The maximum marks are 360.
4. There are three parts in the question paper A, B, C consisting of Chemistry, Mathematics and Physics
having 30 questions in each part of equal weightage. Each question is allotted 4 (four) marks for correct
response.
5. Candidates will be awarded marks as stated above in instruction No. 4 for correct response of each
question. (1/4) (one fourth) marks will be deducted for indicating incorrect response of each question. No
deduction from the total score will be made if no response is indicated for an item in the answer sheet.
6. There is only one correct response for each question. Filling up more than one response in any question
will be treated as wrong response and marks for wrong response will be deducted accordingly as per
instruction 5 above.

FIITJEE Ltd., FIITJEE House, 29-A, Kalu Sarai, Sarvapriya Vihar, New Delhi -110016, Ph 46106000, 26569493, Fax 26513942
website: www.fiitjee.com.
JEE (MAIN)-2013-CMP -2

PART A −CHEMISTRY
1. An unknown alcohol is treated with the “Lucas reagent” to determine whether the alcohol is primary,
secondary or tertiary. Which alcohol reacts fastest and by what mechanism:
(1) tertiary alcohol by SN1 (2) secondary alcohol by SN2
(3) tertiary alcohol by SN2 (4) secondary alcohol by SN1

Sol. (1)
Reaction proceeds through carbocation formation as 30 carbocation is highly stable, hence reaction proceeds
through SN1 with 30 alcohol.

2. The first ionization potential of Na is 5.1 eV. The value of electron gain enthalpy of Na+ will be:
(1) – 5.1 eV (3) –10.2 eV
(3) + 2.55 eV (4) – 2.55 eV

Sol. (1)
∆H =+5.1eV

Na ←
→ Na + + e − , here the backward reaction releases same amount of energy and known as
∆H =−5.1eV
Electron gain enthalpy.

3. Stability of the species Li2, Li −2 and Li +2 increases in the order of:


(1) Li −2 < Li +2 < Li 2 (3) Li 2 < Li 2− < Li 2+
(3) Li −2 < Li 2 < Li +2 (4) Li 2 < Li 2+ < Li 2−

Sol. (1)
Li 2 ( 6 ) = σ1s2 σ* 1s 2 σ2s 2
4−2
B.O. = =1
2
Li +2 ( 5 ) = σ1s2 σ* 1s 2 σ2s1
3− 2
B. O. = = 0.5
2
Li −2 ( 7 ) = σ1s 2 σ* 1s 2 σ2s 2 σ* 2s1
4−3
B.O. = = 0.5
2
Li +2 is more stable than Li −2 because Li −2 has more numbers of antibonding electrons.

4. The molarity of a solution obtained by mixing 750 mL of 0.5 (M) HCl with 250 mL of 2(M)HCl will be:
(1) 1.00 M (2) 1. 75 M
(3) 0.975 M (4) 0.875 M

Sol. (4)
M1V1 + M2V2 = MV
M V + M 2 V2 0.5 × 750 + 2 × 250
M= 1 1 =
V 1000
M = 0. 875

5. Which of the following is the wrong statement?


(1) O3 molecule is bent (2) Ozone is violet-black in solid state
(3) Ozone is diamagnetic gas (4) ONCl and ONO– are not isoelectronic

Sol. (All the options are correct statements)


O
(1) Correct, as O O is bent.

FIITJEE Ltd., FIITJEE House, 29-A, Kalu Sarai, Sarvapriya Vihar, New Delhi -110016, Ph 46106000, 26569493, Fax 26513942
website: www.fiitjee.com.
JEE (MAIN)-2013-CMP-3

(2) Correct, as ozone is violet-black solid.


(3) Correct, as ozone is diamagnetic.
(4) Correct, as ONCl = 32 electrons and ONO− = 24 electron hence are not isoelectronic.
All options are correct statements.
6. Four successive members of the first row transition elements are listed below with atomic numbers. Which
one of them is expected to have the highest E 0M3+ /M2+ value?
(1) Mn(Z = 25) (2) Fe(Z = 26)
(3) Co(Z = 27) (4) Cr(Z = 24)

Sol. (3)
E 0Mn +3 / Mn +2 = 1.57 V
E 0Fe+3 / Fe+2 = 0.77 V
E 0Co+3 / Co+2 = 1.97 V
E 0Cr+3 /Cr +2 = −0.41 V

7. A solution of (–) –1 – chloro –1 – phenylethane is toluene racemises slowly in the presence of a small
amount of SbCl5, due to the formation of :
(1) carbene (2) carbocation
(3) free radical (4) carbanion

Sol. (2)

+ −
CH CH3 
SbCl5
→ [ Ph − CH − CH 3 ] [SbCl6 ]
Planer structure
Cl

8. The coagulating power of electrolytes having ions Na+, Al3+ and Ba2+ for arsenic sulphide sol increases in
the order:
(1) Na + < Ba 2+ < Al3+ (2) Ba 2+ < Na + < Al3+
3+ + 2+
(3) Al < Na < Ba (4) Al3+ < Ba 2+ < Na +

Sol. (1)
As2S3 is an anionic sol (negative sol) hence coagulation will depend upon coagulating power of cation,
which is directly proportional to the valency of cation (Hardy-Schulze rule).

9. How many litres of water must be added to 1 litre of an aqueous solution of HCl with a pH of 1 to create an
aqueous solution with pH of 2?
(1) 0.9 L (2) 2.0 L
(3) 9.0 L (4) 0.1 L

Sol. (3)
Initial pH = 1, i.e. [H+] = 0.1 mole/litre
New pH = 2, i.e. [H+] = 0.01 mole/litre
In case of dilution: M1V1 = M2V2
0.1 ×1 =0.01 × V2
V2 = 10 litre.
Volume of water added = 9 litre.

10. Which one of the following molecules is expected to exhibit diamagnetic behaviour?
(1) N2 (2) O2
(3) S2 (4) C2

Sol. (1) & (4) both are correct answers.


N2 → Diamagnetic
O2 → Paramagnetic
S2 → Paramagnetic

FIITJEE Ltd., FIITJEE House, 29-A, Kalu Sarai, Sarvapriya Vihar, New Delhi -110016, Ph 46106000, 26569493, Fax 26513942
website: www.fiitjee.com.
JEE (MAIN)-2013-CMP -4

C2 → Diamagnetic

11. Which of the following arrangements does not represent the correct order of the property stated against it ?
(1) Ni2+ < Co2+ < Fe2+ < Mn2+ : ionic size
(2) Co3+ < Fe3+ < Cr3+ < Sc3+ : stability in aqueous solution
(3) Sc < Ti < Cr < Mn : number of oxidation states
(4) V2+ < Cr2+ < Mn2+ < Fe2+ : paramagnetic behaviour

Sol. (2) & (4) both are correct answers)


The exothermic hydration enthalpies of the given trivalent cations are:
Sc+3 = 3960 kJ/mole
Fe+3 = 4429 kJ/mole
Co+3 = 4653 kJ/mole
Cr+3 = 4563 kJ/mole
Hence Sc+3 is least hydrated; so least stable (not most stable)
Fe+2 contains 4 unpaired electrons where as Mn+2 contains 5 unpaired electrons hence (4) is incorrect.

12. Experimentally it was found that a metal oxide has formula M0.98O. Metal M, is present as M2+ and M3+ in
its oxide. Fraction of the metal which exists as M3+ would be:
(1) 4.08% (2) 6.05%
(3) 5.08% (4) 7.01%

Sol. (1)
Metal oxide = M0.98O
If ‘x’ ions of M are in +3 state, then
3x + (0.98 – x) × 2 = 2
x = 0.04
0.04
So the percentage of metal in +3 state would be × 100 = 4.08%
0.98

13. A compound with molecular mass 180 is acylated with CH3COCl to get a compound with molecular mass
390. The number of amino groups present per molecule of the former compound is:
(1) 5 (2) 4
(3) 6 (4) 2

Sol. (1)
O O
|| ||
R − NH 2 + CH 3 − C − Cl 
→ R − NH − C − CH 3 + HCl
O
||
Each CH 3 − C addition increases the molecular wt. by 42.
Total increase in m.wt. = 390 – 180 = 210
210
Then number of NH2 groups = =5
42

14. Given
E 0Cr3+ /Cr = −0.74 V; E 0MnO− /Mn 2+ = 1.51 V
4

E 0Cr O2− /Cr3+ = 1.33 V; E 0Cl/ Cl− = 1.36 V


2 7

Based on the data given above, strongest oxidising agent will be:
(1) Cr3+ (2) Mn2+

(3) MnO 4 (4) Cl−

Sol. (3)
As per data mentioned
MnO −4 is strongest oxidising agent as it has maximum SRP value.

FIITJEE Ltd., FIITJEE House, 29-A, Kalu Sarai, Sarvapriya Vihar, New Delhi -110016, Ph 46106000, 26569493, Fax 26513942
website: www.fiitjee.com.
JEE (MAIN)-2013-CMP-5

15. Arrange the following compounds in order of decreasing acidity:


OH OH OH OH

Cl CH3 NO2 OCH3


(I) (II) (III) (IV)
(1) I > II > III > IV (2) III > I > II > IV
(3) IV > III > I > II (4) II > IV > I > III
Sol. (2)
Correct order of acidic strength is III > I > II > IV

16. The rate of a reaction doubles when its temperature changes from 300K to 310K. Activation energy of such
a reaction will be:
(R = 8.314 JK–1 mol–1 and log 2 = 0.301)
(1) 48.6 kJ mol–1 (2) 58.5 kJ mol–1
–1
(3) 60.5 kJ mol (4) 53.6 kJ mol–1

Sol. (4)
As per Arrhenius equation:
k E æ1 1 ö÷
ln 2 = - a ççç - ÷
÷
k1 R èçT2 T1 ø÷
Ea æ ö
çç 1 - 1 ÷
2.303 log 2 = - ÷
÷
ç
8.314 è310 300 ø
⇒ Ea = 53.6 kJ/mole

17. Synthesis of each molecule of glucose in photosynthesis involves:


(1) 10 molecules of ATP (2) 8 molecules of ATP
(3) 6 molecules of ATP (4) 18 molecules of ATP
Sol. (4)
12H 2 O + 12NADP + 18ADP 
Light reaction
→ 6O 2 + 18ATP + 12NADPH
6CO2 + 12NADPH + 18ATP 
Dark reaction
→ C6 H12 O6 + 12NADP + 18ADP + 6H 2 O
Net reaction: 6CO 2 + 6H 2 O 
→ C6 H12 O6 + 6O2

18. Which of the following complex species is not expected to exhibit optical isomerism?
+
(1) Co ( en )2 Cl2  (2) Co ( NH 3 )3 Cl3 
+ 3+
(3) Co ( en ) ( NH3 )2 Cl2  (4) Co ( en )3 

Sol. (2)
[Co(NH3)3Cl3] exists in two forms (facial and meridonial)

FIITJEE Ltd., FIITJEE House, 29-A, Kalu Sarai, Sarvapriya Vihar, New Delhi -110016, Ph 46106000, 26569493, Fax 26513942
website: www.fiitjee.com.
JEE (MAIN)-2013-CMP -6

NH3 NH3

H3 N Cl H3N Cl

Co Co

H3 N Cl Cl NH3

Cl Cl
facial meridonial
Both of these forms are achiral. Hence, [Co(NH3)3Cl3] does not show optical isomerism.

19. A piston filled with 0.04 mol of an ideal gas expands reversibly from 50.0 mL to 375 mL at a constant
temperature of 37.00C. As it does so, it absorbs 208J of heat. The values of q and w for the process will be:
(R = 8.314 J/mol K) ( l n 7.5 = 2.01)
(1) q = – 208 J, w = – 208 J (2) q = – 208 J, w = + 208 J
(3) q = + 208 J, w = + 208 J (4) q = + 208 J, w = – 208 J

Sol. (4)
Process is isothermal reversible expansion, hence ∆U = 0.
∴ q = −W
As q = +208 J
Hence W = −208 J

20. A gaseous hydrocarbon gives upon combustion 0.72 g of water and 3.08 g of CO2. The empirical formula of
the hydrocarbon is:
(1) C3H4 (2) C6H5
(3) C7H8 (4) C2H4

Sol. (3)
æ yö y
C x H y + ççx + ÷ ÷O2 ¾ ¾
® xCO2 + H 2 O
çè 4 ø÷ 2
Weight (g ) 3.08 g 0.72 g
moles 0.07 0.04
x 0.07
=
y / 2 0.04
x 7
Þ =
y 8

21. The order of stability of the following carbocations:

CH2

H2C CH CH2 ; H3C CH2 CH2 ;


I II
III
is:
(1) II > III > I (2) I > II > III
(3) III > I > II (4) III > II > I

Sol. (3)
Order of stability is III > I > II.

FIITJEE Ltd., FIITJEE House, 29-A, Kalu Sarai, Sarvapriya Vihar, New Delhi -110016, Ph 46106000, 26569493, Fax 26513942
website: www.fiitjee.com.
JEE (MAIN)-2013-CMP-7

(Stability ∝ extent of delocalization)

22. Which of the following represents the correct order of increasing first ionization enthalpy for Ca, Ba, S, Se
and Ar?
(1) S < Se < Ca < Ba < Ar (2) Ba < Ca < Se < S < Ar
(3) Ca < Ba < S < Se < Ar (4) Ca < S < Ba < Se < Ar

Sol. (2)
Increasing order of first ionization enthalpy is
Ba < Ca < Se < S < Ar

23. For gaseous state, if most probable speed is denoated by C*, average speed by C and mean square speed by
C, then for a large number of molecules the ratios of these speeds are:
(1) C* : C : C = 1.128 :1.225 :1 (2) C* : C : C = 1:1.128 :1.225
(3) C* : C : C = 1:1.125 :1.128 (4) C* : C : C = 1.225 :1.128 :1

Sol. (2)
2RT 8RT 3RT
C* = , C= , C=
M πM M

24. The gas leaked from a storage tank of the Union Carbide plant in Bhopal gas tragedy was:
(1) Methylamine (2) Ammonia
(3) Phosgene (4) Methylisocyanate

Sol. (4)
It was methyl isocyanate (CH3NCO)

25. Consider the following reaction:


z
xMnO-4 + yC2 O 24- + zH+ ¾ ¾
® xMn 2+ + 2yCO 2 + H2O
2
The values of x, y and z in the reaction are, respectively:
(1) 2, 5 and 8 (2) 2, 5 and 16
(3) 5, 2 and 8 (4) 5, 2 and 16

Sol. (2)
2MnO −4 + 5C2O4−2 + 16H + 
→ 2Mn +2 + 10CO 2 + 8H 2O
x = 2, y = 5, z = 16

26. Which of the following exists as covalent crystals in the solid state?
(1) Silicon (2) Sulphur
(3) Phosphorous (4) Iodine

Sol. (1)
Silicon (Si) – covalent solid
Sulphur (S8) – molecular solid
Phosphorous (P4) – Molecular solid
Iodine (I2) – Molecular solid

27. Compound (A), C8H9Br, gives a white precipitate when warmed with alcoholic AgNO3. Oxidation of (A)
gives a acid (B), C8H6O4. (B) easily forms anhydride on heating. Identify the compound (A).

FIITJEE Ltd., FIITJEE House, 29-A, Kalu Sarai, Sarvapriya Vihar, New Delhi -110016, Ph 46106000, 26569493, Fax 26513942
website: www.fiitjee.com.
JEE (MAIN)-2013-CMP -8

(1) C2 H5 (2) CH2 Br

Br

CH3
(3) CH2 Br (4) CH2 Br

CH3
CH3

Sol. (3)
+
CH2Br CH2
3( )
AgNO alc.
+ AgBr ↓
 →
CH3 ( pale yellow )
(A) CH3
O
[O] COOH


−H O
→ O
2
COOH
(B) O
pthalic anhydride

æZ2 ö÷
28. Energy of an electron is given by E = - 2.178´ 10- 18 J çç 2 ÷ . Wavelength of light required to excite an
çè n ÷
÷
ø
electron in an hydrogen atom from level n = 1 to n = 2 will be
(h = 6.62 × 10−34 Js and c = 3.0 × 108 ms−1)
(1) 2.816 × 10−7 m (2) 6.500 × 10−7 m
−7
(3) 8.500 × 10 m (4) 1.214 × 10−7 m

Sol. (4)
hc 1 1 
E= = 2.178 × 10−18 × Z2  2 − 2 
λ 1 2 
⇒ λ = 1.214 × 10−7 m

29. An organic compound A upon reacting with NH3 gives B. On heating B gives C. C in presence of KOH
reacts with Br2 to give CH3CH2NH2. A is
(1) CH3CH2CH2COOH (2) H3C CH COOH

CH3
(3) CH3CH2COOH (4) CH3COOH

Sol. (3)

FIITJEE Ltd., FIITJEE House, 29-A, Kalu Sarai, Sarvapriya Vihar, New Delhi -110016, Ph 46106000, 26569493, Fax 26513942
website: www.fiitjee.com.
JEE (MAIN)-2013-CMP-9

O O
NH 3
CH3CH2 C OH ¾ ¾ ¾
® CH3CH2 C ONH4
(A) (B)

∆ / (- H 2 O)
O
CH3CH2 C NH2
(C)
Br2 / KOH

CH3CH2 NH2

30. In which of the following pairs of molecules/ions, both the species are not likely to exist?
(1) H-2 , He22- (2) H 22+ , He 2
(3) H-2 , He22+ (4) H +2 , He 22-

Sol. (2)
Bond order of H 22+ and He2 is zero, thus their existence is not possible.

FIITJEE Ltd., FIITJEE House, 29-A, Kalu Sarai, Sarvapriya Vihar, New Delhi -110016, Ph 46106000, 26569493, Fax 26513942
website: www.fiitjee.com.
JEE (MAIN)-2013-CMP -10

PART B − MATHEMATICS
31. The circle passing through (1, −2) and touching the axis of x at (3, 0) also passes through the point
(1) (2, −5) (2) (5, −2)
(3) (−2, 5) (4) (−5, 2)

Sol. (2)
(x − 3)2 + y2 + λy = 0
The circle passes through (1, − 2)
⇒ 4 + 4 − 2λ = 0 ⇒ λ = 4
(x − 3)2 + y2 + 4y = 0 ⇒ Clearly (5, − 2) satisfies.

32. ABCD is a trapezium such that AB and CD are parallel and BC⊥CD. If ∠ADB = θ, BC = p and CD = q,
then AB is equal to
p 2 + q 2 cos θ p2 + q 2
(1) (2) 2
p cos θ + q sin θ p cos θ + q 2 sin θ

(3)
(p 2
+ q 2 ) sin θ
(4)
(p 2
+ q 2 ) sin θ
( p cos θ + q sin θ ) p cos θ + q sin θ
2

Sol. (4) A B
Using sine rule in triangle ABD α
π −(θ + α)
AB BD
=
sin θ sin ( θ + α ) p
p2 + q 2
p + q sin θ
2 2
p + q sin θ
2 2

⇒ AB = = θ
sin θ cos α + cos θ sin α sin θ ⋅ q cos θ ⋅ p α
+ D q C
p +q
2 2
p2 + q 2

⇒ AB =
(p 2
+ q 2 ) sin θ
.
( p cos θ + q sin θ )

33. Given : A circle, 2x2 + 2y2 = 5 and a parabola, y2 = 4 5 x.


Statement − I : An equation of a common tangent to these curves is y = x + 5 .
5
Statement − II : If the line, y = mx + (m ≠ 0) is their common tangent, then m satisfies m4 − 3m2 + 2 =
m
0.
(1) Statement - I is True; Statement -II is true; Statement-II is not a correct explanation for Statement-I
(2) Statement -I is True; Statement -II is False.
(3) Statement -I is False; Statement -II is True
(4) Statement -I is True; Statement -II is True; Statement-II is a correct explanation for Statement-I

Sol. (1)
5
Let the tangent to the parabola be y = mx + (m ≠ 0).
m
Now, its distance from the centre of the circle must be equal to the radius of the circle.
5 5
So, = 1 + m 2 ⇒ (1 + m2) m2 = 2 ⇒ m4 + m2 − 2 = 0.
m 2
⇒ (m2 − 1) (m2 + 2) = 0 ⇒ m = ± 1
So, the common tangents are y = x + 5 and y = − x − 5 .

34. A ray of light along x + 3y = 3 gets reflected upon reaching x-axis, the equation of the reflected rays is

FIITJEE Ltd., FIITJEE House, 29-A, Kalu Sarai, Sarvapriya Vihar, New Delhi -110016, Ph 46106000, 26569493, Fax 26513942
website: www.fiitjee.com.
JEE (MAIN)-2013-CMP-11

(1) 3y = x − 3 (2) y = 3x − 3
(3) 3y = x − 1 (4) y = x + 3

Sol. (1)
1
Slope of the incident ray is − .
3
1
So, the slope of the reflected ray must be .
3

The point of incidence is ( )


3, 0 . So, the equation of reflected ray is y =
1
3
(x − 3) .
35. All the students of a class performed poorly in Mathematics. The teacher decided to give grace marks of 10
to each of the students. Which of the following statistical measures will not change even after the grace
marks were given ?
(1) median (2) mode
(3) variance (4) mean

Sol. (3)
Variance is not changed by the change of origin.
Alternate Solution:
∑ x−x
2

σ= for y = x + 10 ⇒ y = x + 10
n

∑ y + 10 − y − 10 ∑ y−y
2 2

σ1 = = =σ.
n n

36. If x, y, z are in A.P. and tan−1x, tan−1y and tan−1z are also in A.P., then
(1) 2x = 3y = 6z (2) 6x = 3y = 2z
(3) 6x = 4y = 3z (4) x = y = z

Sol. (4)
If x, y, z are in A.P.
2y = x + z
and tan−1x, tan−1y, tan−1z are in A.P.
2 tan−1 y = tan−1x + tan−1z ⇒ x = y = z.
Note: If y = 0, then none of the options is appropriate.

∫ f ( x ) dx = Ψ ( x ) , then ∫ x f ( x ) dx
5 3
37. If is equal to
1 3 1 3
(1) x Ψ ( x 3 ) − 3∫ x 3 Ψ ( x 3 ) dx + C (2) x Ψ ( x 3 ) − ∫ x 2 Ψ ( x 3 ) dx + C
3 3
1 1
(3)  x 3 Ψ ( x 3 ) − ∫ x 3 Ψ ( x 3 ) dx  + C (4)  x 3 Ψ ( x 3 ) − ∫ x 2 Ψ ( x 3 ) dx  + C
3  3 

Sol. (2)
∫ f ( x ) dx = ψ ( x )
Let x3 = t
3x2dx = dt
1
∫ x f ( x ) dx 3∫
tf ( t ) dt
5 3
then =

=
1
{ 1
}
t f ( t )dt − ∫ 1 ⋅ ∫ f ( t ) dt dt  = x 3 ψ ( x 3 ) − ∫ x 2 ψ ( x 3 ) dx + C .
3 ∫  3

FIITJEE Ltd., FIITJEE House, 29-A, Kalu Sarai, Sarvapriya Vihar, New Delhi -110016, Ph 46106000, 26569493, Fax 26513942
website: www.fiitjee.com.
JEE (MAIN)-2013-CMP -12

x 2 y2
38. The equation of the circle passing through the foci of the ellipse + = 1 , and having centre at (0, 3) is
16 9
(1) x 2 + y 2 − 6y + 7 = 0 (2) x 2 + y 2 − 6y − 5 = 0
(3) x 2 + y 2 − 6y + 5 = 0 (4) x 2 + y 2 − 6y − 7 = 0

Sol. (4)
foci ≡ (± ae, 0)
We have a2e2 = a2 − b2 = 7
( )
2
+ ( 0 − 3)
2
Equation of circle (x − 0)2 + (y − 3)2 = 7 −0
⇒ x2 + y2 − 6y − 7 = 0.

39. The x-coordinate of the incentre of the triangle that has the coordinates of mid points of its sides as (0, 1)
(1, 1) and (1, 0) is
(1) 2 − 2 (2) 1 + 2
(3) 1 − 2 (4) 2 + 2

Sol. (1) C(0, 2)


ax + bx 2 + cx 3
x-coordinate = 1
a+b+c
(1, 1)
2× 2 + 2 2 ×0 + 2× 0 (0, 1)
=
2+2+2 2
B(2, 0)
4 2
= = = 2− 2 . A(0, 0) (1, 0)
4+2 2 2+ 2
Alternate Solution:
x-coordinate = r = (s − a) tan A/2
 4+2 2  π
= 
 − 2 2  tan = 2 − 2 .
 2  4

x
40. The intercepts on x-axis made by tangents to the curve, y = ∫ t dt , x ∈ R, which are parallel to the line
0
y = 2x, are equal to
(1) ± 2 (2) ± 3
(3) ± 4 (4) ± 1

Sol. (4)
2
dy
dx
= x =2 ⇒x=±2⇒y= ∫ t dt = 2 for x = 2
0
−2
and y = ∫
0
t dt = −2 for x = − 2

∴ tangents are y − 2 = 2 (x − 2) ⇒ y = 2x − 2
and y + 2 = 2 (x + 2) ⇒ y = 2x + 2
Putting y = 0, we get x = 1 and − 1.

41. The sum of first 20 terms of the sequence 0.7, 0.77, 0.777, ….. , is
7 7
(1) ( 99 − 10−20 ) (2) (179 + 10−20 )
9 81
7 7
(3) ( 99 + 10−20 ) (4) (179 − 10−20 )
9 81

Sol. (2)
tr = 0.777 ….. r times

FIITJEE Ltd., FIITJEE House, 29-A, Kalu Sarai, Sarvapriya Vihar, New Delhi -110016, Ph 46106000, 26569493, Fax 26513942
website: www.fiitjee.com.
JEE (MAIN)-2013-CMP-13

= 7 (10–1 + 10–2 + 10–3 + ….. + 10–r)


7
= (1 − 10− r )
9
7  7
20 20

∑ ∑
1 7(
S20 = t r =  20 − 10− r  =  20 − (1 − 10−20 )  = 179 + 10−20 )
9   9 9  81
r =1  r =1 

42. Consider :
Statement − I : (p ∧ ~ q) ∧ (~ p ∧ q) is a fallacy.
Statement − II : (p → q) ↔ (~ q → ~ p) is a tautology.
(1) Statement - I is True; Statement -II is true; Statement-II is not a correct explanation for Statement-I
(2) Statement -I is True; Statement -II is False.
(3) Statement -I is False; Statement -II is True
(4) Statement -I is True; Statement -II is True; Statement-II is a correct explanation for Statement-I

Sol. (1)
S1:
p q ~p ~q p^~q ~p^q (p^~q)^(~p^q)
T T F F F F F
T F F T T F F
F T T F F T F
F F T T F F F
Fallacy
S2:
p q ~p ~q p⇒q ~q⇒~p (p ⇒ q) ⇔ (~ q ⇒ ~ p)
T T F F T T T
T F F T F F T
F T T F T T T
F F T T T T T
Tautology
S2 is not an explanation of S1

43. The area (in square units) bounded by the curves y = x , 2y − x + 3 = 0 , x-axis, and lying in the first
quadrant is
(1) 36 (2) 18
27
(3) (4) 9
4

Sol. (4)
2 x = x −3 3
4x = x2 – 6x + 9
x2 – 10x + 9
x = 9, x = 1 (3, 0)
3 9
 3
∫ ( 2y + 3) − y  dy
2
 −0, 
0
 x
3
 2 y3 
 y + 3y −  = 9 + 9 – 9 = 9
 3 0

tan A cot A
44. The expression + can be written as
1 − cot A 1 − tan A
(1) secA cosecA + 1 (2) tanA + cotA
(3) secA + cosecA (4) sinA cosA + 1

Sol. (1)

FIITJEE Ltd., FIITJEE House, 29-A, Kalu Sarai, Sarvapriya Vihar, New Delhi -110016, Ph 46106000, 26569493, Fax 26513942
website: www.fiitjee.com.
JEE (MAIN)-2013-CMP -14

1 cot 2 A 1 − cot 3 A cos ec 2 A + cot A


− = = = 1 + sec A cosec A
cot A (1 − cot A ) (1 − cot A ) cot A (1 − cot A ) cot A

45. The real number k for which the equation, 2x3 + 3x + k = 0 has two distinct real roots in [0, 1]
(1) lies between 2 and 3 (2) lies between −1 and 0
(3) does not exist (4) lies between 1 and 2

Sol. (3)
If 2x3 + 3x + k = 0 has 2 distinct real roots in [0, 1], then f′ (x) will change sign
but f′(x) = 6x2 + 3 > 0
So no value of k exists.

(1 − cos 2x ) ( 3 + cos x )
46. lim is equal to
x →0 x tan 4x
1
(1) (2) 1
2
1
(3) 2 (4) −
4

Sol. (3)
(1 − cos 2x )
lim ( 3 + cos x )
x →0 x ( tan 4x )
2
 sin x  1  4x  1
lim 2   ⋅   ( 3 + cos x ) = 2 ×1× × 1× ( 3 + 1) = 2 .
x →0
 x  4  tan 4x  4

47. Let Tn be the number of all possible triangles formed by joining vertices of an n-sided regular polygon. If
Tn+1 − Tn = 10, then the value of n is
(1) 5 (2) 10
(3) 8 (4) 7

Sol. (1)
n +1
C3 − n C3 = 10 ⇒ nC2 = 10 ⇒ n = 5.

48. At present, a firm is manufacturing 2000 items. It is estimated that the rate of change of production P w.r.t.
dP
additional number of workers x is given by = 100 − 12 x . If the firm employs 25 more workers, then
dx
the new level of production of items is
(1) 3000 (2) 3500
(3) 4500 (4) 2500

Sol. (2)
P 25


2000
dP = ∫ (100 − 12
0
x ) dx

12 × 2
(P – 2000) = 25 × 100 – ( 25 )3/ 2
3
P = 3500.

π /3
dx π
49. Statement − I : The value of the integral ∫ 1+
π /6 tan x
is equal to
6
.
b b
Statement − II : ∫ f ( x ) dx = ∫ f ( a + b − x ) dx .
a a
(1) Statement - I is True; Statement -II is true; Statement-II is not a correct explanation for Statement-I
(2) Statement -I is True; Statement -II is False.

FIITJEE Ltd., FIITJEE House, 29-A, Kalu Sarai, Sarvapriya Vihar, New Delhi -110016, Ph 46106000, 26569493, Fax 26513942
website: www.fiitjee.com.
JEE (MAIN)-2013-CMP-15

(3) Statement -I is False; Statement -II is True


(4) Statement -I is True; Statement -II is True; Statement-II is a correct explanation for Statement-I

Sol. (3)
π/3
dx
I= ∫ 1+
π/6
tan x
π/3
tan x
I= ∫ 1+
π/6
tan x
dx

π
2I =
6
π
I= .
12

1 α 3 
50. If P = 1 3 3  is the adjoint of a 3 × 3 matrix A and |A| = 4, then α is equal to
 2 4 4 
(1) 11 (2) 5
(3) 0 (4) 4

Sol. (1)
1 α 3 
P = 1 3 3 
 2 4 4 
|Adj A| = |A|2
|Adj A| = 16
1 (12 − 12) − α (4 − 6) + 3 (4 − 6) = 16.
2α − 6 = 16.
2α = 22.
α = 11.

51. The number of values of k, for which the system of equations


(k + 1)x + 8y = 4k
kx + (k + 3)y = 3k − 1
has no solution, is
(1) 1 (2) 2
(3) 3 (4) infinite

Sol. (1)
For no solution
k +1 8 4k
= ≠ … (1)
k k + 3 3k − 1
⇒ (k + 1) (k + 3) − 8k = 0
or k2 − 4k + 3 = 0 ⇒ k = 1, 3
But for k = 1, equation (1) is not satisfied
Hence k = 3.

dy
52. If y = sec(tan−1x), then at x = 1 is equal to
dx
1
(1) (2) 1
2
1
(3) 2 (4)
2

FIITJEE Ltd., FIITJEE House, 29-A, Kalu Sarai, Sarvapriya Vihar, New Delhi -110016, Ph 46106000, 26569493, Fax 26513942
website: www.fiitjee.com.
JEE (MAIN)-2013-CMP -16

Sol. (4)
y = sec (tan−1x)
dy 1
= sec ( tan −1 x ) tan ( tan −1 x ) ⋅
dx 1+ x2
dy 1 1
= 2 × 1× = .
dx x =1 2 2

x −2 y −3 z −4 x −1 y − 4 z − 5
53. If the lines = = and = = are coplanar, then k can have
1 1 −k k 2 1
(1) exactly one value (2) exactly two values
(3) exactly three values (4) any value

Sol. (2)
1 −1 −1
1 1 −k = 0
k 2 1
1 (1 + 2k) + 1 (1 + k2) − 1 (2 − k) = 0
k2 + 1 + 2k + 1 − 2 + k = 0
k2 + 3k = 0
(k) (k + 3) = 0
2 values of k.

54. Let A and B be two sets containing 2 elements and 4 elements respectively. The number of subsets of A × B
having 3 or more elements is
(1) 220 (2) 219
(3) 211 (4) 256

Sol. (2)
A × B will have 8 elements.
28 − 8C0 − 8C1 − 8C2 = 256 − 1 − 8 − 28 = 219.
uuur uuur
55. If the vectors AB = 3iˆ + 4kˆ and AC = 5iˆ − 2ˆj + 4kˆ are the sides of a triangle ABC, then the length of the
median through A is
(1) 72 (2) 33
(3) 45 (4) 18

Sol. (2) C
uuur uuur
uuuur AB + AC
AM =
2 M
uuuur
AM = 4iˆ − ˆj + 4kˆ
uuuur
AM = 16 + 16 + 1 = 33
A B

56. A multiple choice examination has 5 questions. Each question has three alternative answers of which
exactly one is correct. The probability that a student will get 4 or more correct answers just by guessing is
13 11
(1) 5 (2) 5
3 3
10 17
(3) 5 (4) 5
3 3

Sol. (2)
P (correct answer) = 1/3

FIITJEE Ltd., FIITJEE House, 29-A, Kalu Sarai, Sarvapriya Vihar, New Delhi -110016, Ph 46106000, 26569493, Fax 26513942
website: www.fiitjee.com.
JEE (MAIN)-2013-CMP-17

4 1 5
1  2 1
5
C 4     + 5 C5  
3  3 3
5× 2 1 11
+ 5 = 5 .
( 3) ( 3) 3
5

 1+ z 
57. If z is a complex number of unit modulus and argument θ, then arg   equals
 1+ z 
π
(1) − θ (2) θ
2
(3) π − θ (4) − θ

Sol. (2)
|z| = 1 ⇒ zz = 1
1+ z 1+ z
= =z.
1+ z 1
1+
z

58. If the equations x2 + 2x + 3 = 0 and ax2 + bx + c = 0, a, b, c ∈ R, have a common root, then


a : b : c is
(1) 3 : 2 : 1 (2) 1 : 3 : 2
(3) 3 : 1 : 2 (4) 1 : 2 : 3

Sol. (4)
For equation x2 + 2x + 3 = 0
both roots are imaginary.
Since a, b, c ∈ R.
If one root is common then both roots are common
a b c
Hence, = =
1 2 3
a : b : c = 1 : 2 : 3.
59. Distance between two parallel planes 2x + y + 2z = 8 and 4x + 2y + 4z + 5 = 0 is
5 7
(1) (2)
2 2
9 3
(3) (4)
2 2
Sol. (2)
4x + 2y + 4z = 16
4x + 2y + 4z = − 5
21 21 7
dmin = = = .
36 6 2
10
 x +1 x −1 
60. The term independent of x in expansion of  2/3 − 1/ 2 
is
 x − x +1 x − x 
1/3

(1) 120 (2) 210


(3) 310 (4) 4
Sol. (2)

( )( )
10
 ( x1/3 + 1)( x 2/3 − x1/3 + 1) x +1 x −1 
 1 
− ⋅ = (x1/3 − x−1/2)10


x − x +1
2/3 1/3
x ( x −1 ) 

20 − 5r
Tr+1 = (− 1)r 10Cr x 6
⇒r=4
10
C4 = 210.

FIITJEE Ltd., FIITJEE House, 29-A, Kalu Sarai, Sarvapriya Vihar, New Delhi -110016, Ph 46106000, 26569493, Fax 26513942
website: www.fiitjee.com.
JEE (MAIN)-2013-CMP -18

PART C − PHYSICS
61. In an LCR circuit as shown below both switches are open initially. Now switch S1 is closed, S2 kept open.
(q is charge on the capacitor and τ = RC is capacitive time constant). Which of the following statement is
correct?

R
S1
C
S2
L
(1) At t = τ, q = CV/2
(2) At t = 2τ, q = CV(1 − e−2)
τ
(3) At t = , q = CV(1 − e−1 )
2
(4) Work done by the battery is half of the energy dissipated in the resistor.

Sol. (2)
Charge on the capacitor at any time ‘t’ is
(
q = CV 1 − e− t / τ )
at t = 2τ
(
q = CV 1 − e −2 )
62. A diode detector is used to detect an amplitude modulated wave of 60% modulation by using a condenser of
capacity 250 pico farad in parallel with a load resistance 100 kilo ohm. Find the maximum modulated
frequency which could be detected by it.
(1) 10.62 kHz (2) 5.31 MHz
(3) 5.31 kHz (4) 10.62 MHz

Sol. (3)
1 1
fC = = = 6.37 kHz
2πRC 2 × 3.14 × 100 × 103 × 250 × 10−12
fC = cut off frequency
As we know that fm fC
∴ (3) is correct
Note: The maximum frequency of modulation must be less than fm, where
1 − m2
f m = fC
m
m ⇒ modulation index

63. The supply voltage to a room is 120 V. The resistance of the lead wires is 6 Ω. A 60 W bulb is already
switched on. What is the decrease of voltage across the bulb, when a 240 W heater is switched on in parallel
to the bulb?
(1) 2.9 Volt (2) 13.3 Volt
(3) 10.04 Volt (4) zero volt

Sol. (3) Heater


120 × 120
Resistance of bulb = = 240Ω
60
120 × 120 6Ω Bulb
Resistance of Heater = = 60Ω
240

120 V

FIITJEE Ltd., FIITJEE House, 29-A, Kalu Sarai, Sarvapriya Vihar, New Delhi -110016, Ph 46106000, 26569493, Fax 26513942
website: www.fiitjee.com.
JEE (MAIN)-2013-CMP-19

120
Voltage across bulb before heater is switched on, V1 = × 240
246
120
Voltage across bulb after heater is switched on, V2 = × 48
54
Decrease in the voltage is V1 − V2 = 10.04 (approximately)
Note: Here supply voltage is taken as rated voltage.

64. A uniform cylinder of length L and mass M having cross-sectional area A is suspended, with its length
vertical, from a fixed point by a massless spring, such that it is half submerged in a liquid of density σ at
equilibrium position. The extension x0 of the spring when it is in equilibrium is:
Mg  LAσ  Mg  LAσ 
(1) 1 −  (2) 1 − 
k  M  k  2M 
Mg  LAσ  Mg
(3) 1 +  (4)
k  M  k
(Here k is spring constant)

Sol. (2)
At equilibrium ΣF = 0 kx0 Buoyant force
 AL 
kx 0 +  σg  − Mg = 0
 2 
 LAσ 
x 0 = Mg 1 −
 2M  Mg

65. Two charges, each equal to q, are kept at x = −a and x = a on the x-axis. A particle of mass m and charge
q
q 0 = is placed at the origin. If charge q0 is given a small displacement (y a) along the y-axis, the net
2
force acting on the particle is proportional to:
1
(1) −y (2)
y
1
(3) − (4) y
y

Sol. (4)
Fnet = 2Fcos θ F θ θ F
k ⋅q ⋅q / 2 y q/2
=2 ⋅
( )
2 y
a 2 + y2 a + y2
2

q a a q
kq 2 y
= (y a)
a3

66. A beam of unpolarised light of intensity I0 is passed through a polaroid A and then through another polaroid
B which is oriented so that its principal plane makes an angle of 45° relative to that of A. The intensity of
the emergent light is:
(1) I0/2 (2) I0/4
(3) I0/8 (4) I0

Sol. (2)

67. The anode voltage of a photocell is kept fixed. The wavelength λ of the light falling on the cathode is
gradually changed. The plate current I of the photocell varies as follows:

FIITJEE Ltd., FIITJEE House, 29-A, Kalu Sarai, Sarvapriya Vihar, New Delhi -110016, Ph 46106000, 26569493, Fax 26513942
website: www.fiitjee.com.
JEE (MAIN)-2013-CMP -20

(1) (2)
I I

O O
λ λ
(3) (4)
I I

O O
λ λ

Sol. (3)
68. Two coherent point sources S1 and S2 are separated by a small distance ‘d’ as d
shown. The fringes obtained on the screen will be: S1 S2 Screen
(1) straight lines (2) semi-circles
D
(3) concentric circles (4) points

Sol. (3)
69. A metallic rod of length ‘l’ is tied to a string of length 2l and made to
rotate with angular speed ω on a horizontal table with one end of the
string fixed. If there is a vertical magnetic field ‘B’ in the region, the
e.m.f. induced across the ends of the rod is:
3Bωl 2 4Bωl 2
(1) (2)
2 2
5Bωl 2 2Bωl 2
(3) (4)
2 2

Sol. (3)
de = B(ωx) ⋅ dx
ω
3L 2L L
e = Bω ∫ xdx x
2L

5BωL2
=
2

70. In a hydrogen like atom electron makes transition from an energy level with quantum number n to another
with quantum number (n – 1). If n >> 1, the frequency of radiation emitted is proportional to
1 1
(1) 2 (2) 3/ 2
n n
1 1
(3) 3 (4)
n n

Sol. (3)
 1 1 
ν∝ 2
− 2
 (n − 1) n 
(2n − 1)
∝ 2
n (n − 1) 2
1
∝ (since n 1)
n3

FIITJEE Ltd., FIITJEE House, 29-A, Kalu Sarai, Sarvapriya Vihar, New Delhi -110016, Ph 46106000, 26569493, Fax 26513942
website: www.fiitjee.com.
JEE (MAIN)-2013-CMP-21

71. Assume that a drop of liquid evaporates by decrease in its surface energy, so that its temperature remains
unchanged. What should be the minimum radius of the drop for this to be possible? The surface tension is
T, density of liquid is ρ and L is its latent heat of vaporization.
(1) T / ρL (2) T/ρL
(3) 2T/ρL (4) ρL/T

Sol. (3)
ρ4πR2∆RL = T 4π[R2 – (R - ∆R)2]
ρR2∆RL = T[R2 – R2 + 2R∆R - ∆R2]
ρR2∆RL = T2R∆R (∆R is very small) R
2T
R= .
ρL

72. The graph between angle of deviation (δ) and angle of incidence (i) for a triangular prism is represented by:

(1) δ (2) δ

O i O i

(3) δ (4) δ

O i O i
Sol. (2)

73. Let [ε0] denote the dimensional formula of the permittivity of vacuum. If M = mass, L = length, T = time
and A = electric current, then:
(1) [ε0] = [M-1L-3T4A2] (2) [ε0] = [M-1L2T-1A-2]
-1 2 -1
(3) [ε0] = [M L T A] (4) [ε0] = [M-1L-3T2A]

Sol. (1)
1 q2
73. =F
4πε0 r 2
[A 2 T 2 ]
ε0= = [M −1L−3 A 2 T 4 ]
[MLT −2 L2 ]

74.
A B
2p0
p
p0 C
D

O v0 2v0
v
The above p-v diagram represents the thermodynamic cycle of an engine, operating with an ideal
monoatomic gas. The amount of heat extracted from the source in a single cycle is
 13   11 
(1)   p0 v 0 (2)   p0 v0
 2 2
(3) 4p0v0 (4) p0v0

Sol. (2)
Heat is extracted from the source in path DA and AB is

FIITJEE Ltd., FIITJEE House, 29-A, Kalu Sarai, Sarvapriya Vihar, New Delhi -110016, Ph 46106000, 26569493, Fax 26513942
website: www.fiitjee.com.
JEE (MAIN)-2013-CMP -22

3  P0 V0  5  2P0 V0  13
∆Q = R + R = P0 V0
2  R  2  R  2

75. A sonometer wire of length 1.5 m is made of steel. The tension in it produces an elastic strain of 1 %.
What is the fundamental frequency of steel if density and elasticity of steel are 7.7 × 103 kg/m3 and
2.2 × 1011N/m2 respectively?
(1) 178.2 Hz (2) 200.5 Hz
(3) 770 Hz (4) 188.5 Hz

Sol. (1)
1 T
Fundamental frequency f =
2l µ
1 T
=
2l Aρ
1 stress 1 2.2 × 1011 ×10−2
= = .
2l ρ 2 × 1.5 7.7 × 103

76. This question has statement I and statement II. Of the four choices given after the statements, choose the
one that best describes the two statements.
Statement- I: Higher the range, greater is the resistance of ammeter.
Statement- II: To increase the range of ammeter, additional shunt needs to be used across it.
(1) Statement – I is true, Statement – II is true, Statement – II is not the correct explanation of Statement–I.
(2) Statement – I is true, statement – II is false.
(3) Statement – I is false, Statement – II is true
(4) Statement – I is true, Statement – II is true, Statement – II is the correct explanation of statement- I .

Sol. (3)
Ig G
For Ammeter, S =
I − Ig
So for I to increase, S should decrease, so additional S can be connected across it.

77. What is the minimum energy required to launch a satellite of mass m from the surface of a planet of mass
M and radius R in a circular orbit at an altitude of 2R?
2GmM GmM
(1) (2)
3R 2R
GmM 5GmM
(3) (4)
3R 6R

Sol. (4)
GMm
T. Ef = −
6R
GMm
T. Ei = −
R
5GMm
∆W = T.Ef – T.Ei =
6R

78. ( )
A projectile is given an initial velocity of iˆ + 2 ˆj m/s, where iˆ is along the ground and ĵ is along the
2
vertical. If g = 10 m/s , the equation of its trajectory is:
(1) y = 2 x − 5 x 2 (2) 4 y = 2 x − 5 x 2
(3) 4 y = 2 x − 25 x 2 (4) y = x − 5 x 2

Sol. (1)

FIITJEE Ltd., FIITJEE House, 29-A, Kalu Sarai, Sarvapriya Vihar, New Delhi -110016, Ph 46106000, 26569493, Fax 26513942
website: www.fiitjee.com.
JEE (MAIN)-2013-CMP-23

x=t
y = 2t − 5t 2
Equation of trajectory is y = 2 x − 5 x 2

79. Two capacitors C1 and C2 are charged to 120 V and 200 V respectively. It is found that by connecting
them together the potential on each one can be made zero. Then :
(1) 3C1 = 5C2 (2) 3C1 + 5C2 = 0
(3) 9C1 = 4C2 (4) 5C1 = 3C2

Sol. (1)
120C1 = 200C2
6C1 = 10C2
3C1 = 5C2

80. A hoop of radius r and mass m rotating with an angular velocity ω0 is placed on a rough horizontal surface.
The initial velocity of the centre of the hoop is zero. What will be the velocity of the centre of the hoop
when it ceases to slip?
rω0 rω0
(1) (2)
3 2
rω0
(3) rω0 (4)
4

Sol. (2)
From conservation of angular momentum about any fix point on the surface
mr 2ω0 = 2mr 2ω
ω0
∴ ω=
2
ω0 r
∴ VCM =
2

81. An ideal gas enclosed in a vertical cylindrical container supports a freely moving piston of mass M. The
piston and cylinder have equal cross sectional area A. When the piston is in equilibrium, the volume of the
gas is V0 and its pressure is P0. The piston is slightly displaced from the equilibrium position and released.
Assuming that the system is completely isolated from its surrounding, the piston executes a simple
harmonic motion with frequency:
1 V0 MP0 1 A2γ P0
(1) (2)
2π A2γ 2π MV0
1 MV0 1 Aγ P0
(3) (4)
2π Aγ P0 2π V0 M

Sol. (2)
Patm A

P0 A
mg
FBD of piston at equilibrium
⇒ Patm A + mg = P0 A ...(1)
Patm A

(P0 + dP) A
mg
FBD of piston when piston is pushed down a distance x

FIITJEE Ltd., FIITJEE House, 29-A, Kalu Sarai, Sarvapriya Vihar, New Delhi -110016, Ph 46106000, 26569493, Fax 26513942
website: www.fiitjee.com.
JEE (MAIN)-2013-CMP -24

d2x
Patm + mg – (P0 +dP) A = m ...(2)
dt 2
γ PdV
Process is adiabatic ⇒ PV γ = C ⇒ − dP =
V
1 A γ P0
2
Using 1, 2, 3 me get f =
2π MV0

82. A charge Q is uniformly distributed over a long rod AB of length L as shown in the figure. The electric
potential at the point O lying at a distance L from the end A is:
O A B
L L
3Q Q
(1) (2)
4πε 0 L 4πε 0 L ln 2
Q ln 2 Q
(3) (4)
4πε 0 L 8πε 0 L

Sol. (3)
O

x dx
x =2 L k Q Q ln 2
V =∫   dx =
x=L x L  4πε 0 L

83. A circular loop of radius 0.3 cm lies parallel to a much bigger circular loop of radius 20 cm. The centre of
the small loop is on the axis of the bigger loop. The distance between their centres is 15 cm. If a current of
2.0 A flows through the smaller loop, then the flux linked with bigger loop is
(1) 6 × 10–11 weber (2) 3.3 × 10–11 weber
–9
(3) 6.6 × 10 weber (4) 9.1 × 10–11 weber

Sol. (4)
2 1
r

R d

Let M12 be the coefficient of mutual induction between loops


φ1 = M12 i2
µ0 i2 R 2
⇒ π r 2 = M 12 i2
2(d 2 + R 2 )3/2
µ 0 R 2π r 2
⇒ M 12 =
2(d 2 + R 2 )3/ 2
φ2 = M 12 i1 ⇒ φ2 = 9.1×10−11 weber

84. If a piece of metal is heated to temperature θ and then allowed to cool in a room which is at temperature
θ 0 the graph between the temperature T of the metal and time t will be closest to :
(1) T (2) T
θ0 θ0

O t O t

FIITJEE Ltd., FIITJEE House, 29-A, Kalu Sarai, Sarvapriya Vihar, New Delhi -110016, Ph 46106000, 26569493, Fax 26513942
website: www.fiitjee.com.
JEE (MAIN)-2013-CMP-25

(3) T (4) T
θ0

O t O t

Sol. (2)
The temperature goes on decreasing with time (non-linearly) The rate of decrease will be more initially
which is depicted in the second graph.

85. The I – V characteristic of an LED is


(1) B (2) I
G
Y
R
O V
O V
(3) V (4)

Yellow
O

Green
Blue
Red
R
(R)(Y)(G)(B)
Y I
G
B
I

O V

Sol. (4)
For LED, in forward bias, intensity increases with voltage.

86. This question has Statement I and Statement II. Of the four choices given after the Statements, choose the
one that best describes the two Statements.
Statement – I : A point particle of mass m moving with speed v collides with stationary point particle of
1   m 
mass M. If the maximum energy loss possible is given as f  mv 2  then f =  .
2  M +m
Statement – II : Maximum energy loss occurs when the particles get stuck together as a result of the
collision.
(1) Statement – I is true, Statement – II is true, Statement – II is not a correct explanation of Statement – I.
(2) Statement – I is true, Statement – II is false.
(3) Statement – I is false, Statement – II is true
(4) Statement – I is true, Statement – II is true, Statement – II is a correct explanation of Statement – I.

Sol. (3)
Loss of energy is maximum when collision is inelastic as in an inelastic collision there will be maximum
deformation.
1  Mm  2
KE in COM frame is   Vrel
2 M +m
1  Mm  2
KEi =  V KE f = 0 (Q Vrel = 0 )
2 M +m
1  Mm  2
Hence loss in energy is  V
2 M +m
M
⇒ f =
M +m

FIITJEE Ltd., FIITJEE House, 29-A, Kalu Sarai, Sarvapriya Vihar, New Delhi -110016, Ph 46106000, 26569493, Fax 26513942
website: www.fiitjee.com.
JEE (MAIN)-2013-CMP -26

87. The amplitude of a damped oscillator decreases to 0.9 times its original magnitude is 5s. In another 10s it
will decrease to α times its original magnitude, where α equals.
(1) 0.81 (2) 0.729
(3) 0.6 (4) 0.7

Sol. (2)
A = A0 e− kt
⇒ 0.9 A0 = A0 e−5 k
and α A0 = A0 e−15 k
solving ⇒ α = 0.729

88. Diameter of plano-convex lens is 6 cm and thickness at the centre is 3 mm. If speed of light in material of
lens is 2 × 108 m/s, the focal length of the lens is :
(1) 20 cm (2) 30 cm
(3) 10 cm (4) 15 cm

Sol. (2)

R
d

R-t t

R 2 = d 2 + ( R − t )2
2
 t
R 2 − d 2 = R 2 1 − 
 R
2
d 2t
1− 2 = 1−
R R
(3)2 90
R= = = 15 cm
2 × (0 ⋅ 3) 6
1  1 1 
= ( µ − 1)  − 
f R
 1 R2 

1  3  1 
=  − 1  
f  2   15 
f = 30 cm

89. The magnetic field in a travelling electromagnetic wave has a peak value of 20 nT. The peak value of
electric field strength is :
(1) 6 V/m (2) 9 V/m
(3) 12 V/m (4) 3 V/m

Sol. (1)
Ε 0 = CB0
= 3 × 108 × 20 × 10–9
= 6 V/m

90. Two short bar magnets of length 1 cm each have magnetic moments 1.20 Am2 and 1.00 Am2 respectively.
They are placed on a horizontal table parallel to each other with their N poles pointing towards the South.
They have a common magnetic equator and are separated by a distance of 20.0 cm. The value of the
resultant horizontal magnetic induction at the mid - point O of the line joining their centres is close to
(Horizontal component of earth’s magnetic induction is 3.6 × 10–5 Wb/m2)

FIITJEE Ltd., FIITJEE House, 29-A, Kalu Sarai, Sarvapriya Vihar, New Delhi -110016, Ph 46106000, 26569493, Fax 26513942
website: www.fiitjee.com.
JEE (MAIN)-2013-CMP-27

(1) 2.56 × 10–4 Wb/m2 (2) 3.50 × 10–4 Wb/m2


(3) 5.80 × 10–4 Wb/m2 (4) 3.6 × 10–5 Wb/m2

Sol. (1)
Bnet = BM1 + BM 2 + BH
µ0 M 1 µ 0 M 2
= + + BH
4π x3 4π x3
µ
= 0 3 ( M 1 + M 2 ) + BH
4π x
10−7
= × 2.2 + 3.6 ×10−5
10−3
= 2.56 × 10−4 Wb/m2

FIITJEE Ltd., FIITJEE House, 29-A, Kalu Sarai, Sarvapriya Vihar, New Delhi -110016, Ph 46106000, 26569493, Fax 26513942
website: www.fiitjee.com.

You might also like